BDT Hinhhoc HoangNgocQuang

You might also like

Download as pdf or txt
Download as pdf or txt
You are on page 1of 120

I HC THI NGUYN

TRNG I HC KHOA HC
Hong Ngc Quang
MT S BT NG THC HNH HC
Chuyn Nghnh: PHNG PHP TON S CP
M S: 60.46.40
LUN VN THC S TON HC
Ngi hng dn khoa hc: TS. Nguyn Vn Ngc
Thi Nguyn - 2011
www.MATHVN.com - HOANG NGOC QUANG, Yen Bai
Cng trnh c hon thnh ti
Trng i hc Khoa hc - i hc Thi Nguyn
Ngi hng dn khoa hc: TS. Nguyn Vn Ngc
Phn bin 1: . . . . . . . . . . . . . . . . . . . . . . . . . . . . . . . . . . . . . . . . . . . . . . . . . . . . . . .
. . . . . . . . . . . . . . . . . . . . . . . . . . . . . . . . . . . . . . . . . . . . . . . . . . . . . . . . . . . . . . . . . . . .
Phn bin 2: . . . . . . . . . . . . . . . . . . . . . . . . . . . . . . . . . . . . . . . . . . . . . . . . . . . . . . .
. . . . . . . . . . . . . . . . . . . . . . . . . . . . . . . . . . . . . . . . . . . . . . . . . . . . . . . . . . . . . . . . . . . .
Lun vn s c bo v trc hi ng chm lun vn hp ti:
Trng i hc Khoa hc - i hc Thi Nguyn
Ngy .... thng .... nm 2011
C th tm hiu ti
Th vin i hc Thi Nguyn
www.MATHVN.com - HOANG NGOC QUANG, Yen Bai
1
Mc lc
Mc lc . . . . . . . . . . . . . . . . . . . . . . . . . . . . . 1
M u . . . . . . . . . . . . . . . . . . . . . . . . . . . . . . 3
Chng 1. Cc bt ng thc trong tam gic v t gic 6
1.1. Cc bt ng thc i s c bn . . . . . . . . . . . . . . 6
1.2. Cc ng thc v bt ng thc c bn trong tam gic . 8
1.2.1. Cc ng thc c bn trong tam gic . . . . . . . 8
1.2.2. Cc bt ng thc c bn trong tam gic . . . . . 10
1.3. Bt ng thc trong tam gic . . . . . . . . . . . . . . . 11
1.3.1. Bt ng thc v di cc cnh . . . . . . . . . 11
1.3.2. Bt ng thc v cc i lng c bit . . . . . 14
1.4. Cc bt ng thc sinh ra t cc cng thc hnh hc . . 17
1.5. Bt ng thc trong cc tam gic c bit . . . . . . . . 23
1.5.1. Cc bt ng thc trong tam gic u . . . . . . 23
1.5.2. Cc bt ng thc trong tam gic vung v tam
gic cn . . . . . . . . . . . . . . . . . . . . . . . 27
1.6. Cc bt ng thc khc trong tam gic . . . . . . . . . . 29
1.7. Cc bt ng thc trong t gic . . . . . . . . . . . . . . 40
1.7.1. Cc bt ng thc c bn trong t gic . . . . . . 41
1.7.2. Cc bt ng thc khc trong t gic . . . . . . . 45
Chng 2. Bt ng thc Ptolemy v cc m rng 48
2.1. nh l Ptolemy . . . . . . . . . . . . . . . . . . . . . . . 48
2.2. Bt ng thc Ptolemy . . . . . . . . . . . . . . . . . . . 53
2.3. nh l Bretschneider . . . . . . . . . . . . . . . . . . . . 63
2.4. nh l Casey . . . . . . . . . . . . . . . . . . . . . . . . 63
2.5. M rng bt ng thc Ptolemy trong khng gian . . . . 68
www.MATHVN.com - HOANG NGOC QUANG, Yen Bai
2
Chng 3. Bt ng thc Erdos-Mordell v cc m rng 70
3.1. Bt ng thc Erdos-Mordell trong tam gic . . . . . . . 70
3.2. Bt ng thc Erdos-Mordell trong tam gic m rng . . 79
3.3. M rng bt ng thc Erdos-Mordell trong t gic . . . 85
3.4. M rng bt ng thc Erdos-Mordell trong a gic . . . 87
3.5. M rng bt ng thc Erdos-Mordell trong t din . . . 90
Chng 4. Cc bt ng thc c trng 92
4.1. Bt ng thc dng Hayashi v cc h qu . . . . . . . . 92
4.1.1. Bt ng thc Hayashi . . . . . . . . . . . . . . . 92
4.1.2. Cc h qu ca bt ng thc hyashi . . . . . . . 94
4.1.3. Bi ton p dng . . . . . . . . . . . . . . . . . . 94
4.2. Bt ng thc Weizenbock suy rng v cc h qu . . . 96
4.2.1. Bt ng thc Weizenbock suy rng . . . . . . . 96
4.2.2. Cc h qu ca bt ng thc Weizenbock suy rng101
4.3. Bt ng thc Klamkin v cc h qu . . . . . . . . . . 105
4.3.1. Bt ng thc Klamkin . . . . . . . . . . . . . . 105
4.3.2. Cc h qu ca bt ng thc Klamkin . . . . . . 106
4.4. Bt ng thc Jian Liu v cc h qu . . . . . . . . . . 108
4.4.1. Bt ng thc Jian Liu . . . . . . . . . . . . . . . 108
4.4.2. Cc h qu ca bt ng thc Jian Liu . . . . . . 110
Kt lun . . . . . . . . . . . . . . . . . . . . . . . . . . . . . 116
Ti liu tham kho . . . . . . . . . . . . . . . . . . . . . . . 117
www.MATHVN.com - HOANG NGOC QUANG, Yen Bai
3
M u
Cc bi ton v bt ng thc v cc tr hnh hc thuc loi nhng
bi ton kh, lm cho hc sinh ph thng, nht l ph thng c s k c
hc sinh gii lng tng khi gp cc bi ton loi ny. Thc s n l mt
phn rt quan trng ca hnh hc v nhng kin thc v bt ng thc
trong hnh hc cng lm phong ph hn phm vi ng dng ca ton
hc. So vi cc bt ng thc i s, cc bt ng thc hnh hc cha
c quan tm nhiu. Mt trong nhng nguyn nhn gy kh gii quyt
vn ny l v phng php tip cn khng phi l cc phng php
thng thng hay c p dng trong hnh hc v cng khng phi l
phng php i s thun ty. gii mt bi ton v bt ng thc
hnh hc cn thit phi bit vn dng cc kin thc hnh hc v i s
mt cch thch hp v nhy bn.
Lun vn ny gii thiu mt s bt ng thc hnh hc t c bn
n nng cao v m rng. Cc bi ton v bt ng thc hnh hc c
trnh by trong lun vn ny c th tm phn thnh cc nhm sau:
I. Nhm cc bi ton m trong li gii i hi nht thit phi c
hnh v. Phng php gii cc bi ton nhm ny ch yu l "phng
php hnh hc", nh v thm ng ph, s dng tnh cht gia ng
vung gc v ng xin, gia ng thng v ng gp khc, quan
h gia cc cnh, gia cnh v gc trong mt tam gic, hay t gic v.v..
Bt ng thc v cc tr trong hnh hc phng thuc nhm ny l ni
dung thng gp trong cc k thi chn hc sinh gii ton hay thi vo cc
trng chuyn.
II. Nhm th hai gm cc bi ton m khi gii chng cn phi s
dng cc h thc lng bit, nh cc h thc lng gic, h thc
ng trung tuyn, ng phn gic, cng thc cc bn knh, cng thc
www.MATHVN.com - HOANG NGOC QUANG, Yen Bai
4
din tch ca tam gic v.v.. Cc bi ton ny c quan tm nhiu
v chng c trnh by kh phong ph trong cc ti liu [4,7], v th
lun vn ny s khng cp nhiu n cc bt ng thc trong tam
gic c trong cc ti liu trn mc d chng rt hay m ch nu ra mt
s bt ng thc c bn nht tin s dng sau ny.
III. Nhm th ba gm cc bi ton lin quan n cc bt ng thc
hnh hc ni ting, c bit l bt ng thc Ptolemy v bt ng thc
Erdos-Mordell v cc bt ng thc c trng nh bt ng thc Hayshi,
bt ng thc Weizenbock, bt ng thc Klamkin v.v.. Cc bt ng
thc ny cn t c gii thiu bng Ting Vit v thng gp trong cc
thi Olympic Quc t.
Bn lun vn "Mt s bt ng thc hnh hc" gm c m u,
bn chng ni dung, kt lun v ti liu tham kho.
Chng 1. Cc bt ng thc trong tam gic v t gic.
Chng ny trnh by mt s bt ng thc thuc nhm I v nhm II.
Chng 2. Bt ng thc Ptolemy v cc m rng.
Chng ny trnh by ng thc Ptolemy, bt ng thc Ptolemy v
cc bi ton p dng. Cc bi ton ny ch yu c trch ra t cc
thi v ch cc nc, thi v ch khu vc v thi IMO, mt s l do
tc gi sng tc. Ngoi ra, cn trnh by mt s m rng bt ng thc
Ptolemy trong t gic v trong t din.
Chng 3. Bt ng thc Erdos - Mordell v cc m rng.
Chng ny trnh by bt ng thc Edos-Mordell v cc bi ton lin
quan. Ngoi ra, cn trnh by mt s m rng bt ng thc ny trong
tam gic, trong t gic v trong a gic [11-13].
Chng 4. Cc bt ng thc c trng.
Chng ny trnh by mt s bt ng thc lin quan n tng khong
cch t mt hay nhiu im ca mt phng n cc nh hoc cc cnh
ca tam gic vi cc tham s dng ty c gi l trng s hay gi
tt l trng. l cc bt ng thc Hyashi, Weizenbock, Klamkin, Jian
www.MATHVN.com - HOANG NGOC QUANG, Yen Bai
5
Liu, v.v.. Cc bt ng thc ny cn t c gii thiu bng Ting Vit,
mt s l kt qu nghin cu ca cc chuyn gia Quc t trong lnh vc
bt ng thc hnh hc [9,13-14].
Lun vn ny c hon thnh ti trng i hc Khoa hc - i
hc Thi Nguyn vi s hng dn ca TS. Nguyn Vn Ngc. Tc gi
xin c by t lng bit n su sc i vi s quan tm hng dn ca
Thy, ti cc thy c trong Ban Gim hiu, Phng o to v Khoa
Ton-Tin Trng i hc Khoa hc. ng thi tc gi xin cm n ti
S GD - T tnh Yn Bi, Ban Gim c, cc ng nghip Trung tm
GDTX - HNDN H Tng Mu huyn Lc Yn to iu kin cho tc
gi hc tp v hon thnh k hoch hc tp.
Thi Nguyn, ngy 20 thng 6 nm 2011.
Tc gi
Hong Ngc Quang
www.MATHVN.com - HOANG NGOC QUANG, Yen Bai
6
Chng 1
Cc bt ng thc trong tam gic
v t gic
Chng ny trnh by cc bt ng thc trong tam gic v t gic
t c bn n nng cao. Ni dung ch yu c hnh thnh t cc ti
liu [1-7], [10], [12] v [15].
K hiu ABC l tam gic ABC vi cc nh l A, B, C. thun
tin, ln ca cc gc ng vi cc nh A, B, C cng c k hiu tng
ng l A, B, C.
di cc cnh ca tam gic: BC = a, CA = b, AB = c.
Na chu vi ca tam gic: p =
a +b +c
2
.
ng cao vi cc cnh: h
a
, h
b
, h
c
.
ng trung tuyn vi cc cnh: m
a
, m
b
, m
c
.
ng phn gic vi cc cnh: l
a
, l
b
, l
c
.
Bn knh ng trn ngoi tip v ng trn ni tip: R v r.
Bn knh ng trn bng tip cc cnh: r
a
, r
b
, r
c
.
Din tch tam gic ABC: S, S
ABC
hay [ABC].
gii c cc bi ton bt ng thc hnh hc, trc ht ta cn
trang b nhng kin thc c s l cc bt ng thc i s c bn v
cc ng thc, bt ng thc n gin trong tam gic.
1.1. Cc bt ng thc i s c bn
nh l 1.1. (Bt ng thc AM-GM) Gi s a
1
, a
2
, , a
n
l cc s
thc khng m. Khi
a
1
+a
2
+ +a
n
n

n

a
1
a
2
...a
n
. (1.1)
ng thc xy ra khi v ch khi a
1
= a
2
= = a
n
.
www.MATHVN.com - HOANG NGOC QUANG, Yen Bai
7
H qu 1.1. Vi mi b s dng a
1
, a
2
, , a
n
ta c
n

a
1
a
2
...a
n

n
1
a
1
+
1
a
2
+ +
1
a
n
. (1.2)
ng thc xy ra khi v ch khi a
1
= a
2
= = a
n
.
H qu 1.2. Vi mi b s dng a
1
, a
2
, , a
n
ta c
1
a
1
+
1
a
2
+ +
1
a
n

n
2
a
1
+a
2
+ +a
n
. (1.3)
ng thc xy ra khi v ch khi a
1
= a
2
= ... = a
n
.
H qu 1.3. Vi mi b s khng m a
1
, a
2
, , a
n
v m = 1, 2, ta
c
a
m
1
+a
m
2
+ +a
m
n
n

_
a
1
+a
2
+ +a
n
n
_
m
. (1.4)
ng thc xy ra khi v ch khi a
1
= a
2
= = a
n
.
nh l 1.2. (Bt ng thc Cauchy - Schwarz) Cho hai dy s thc
a
1
, a
2
, , a
n
v b
1
, b
2
, , b
n
. Khi
(a
1
b
1
+a
2
b
2
+ +a
n
b
n
)
2

_
a
2
1
+a
2
2
+ +a
2
n
_ _
b
2
1
+b
2
2
+ +b
2
n
_
. (1.5)
ng thc xy ra khi v ch khi
a
1
b
1
=
a
2
b
2
= =
a
n
b
n
.
nh l 1.3. (Bt ng thc Jensen) Cho f(x) l hm s lin tc v c
o hm cp hai trn I (a, b) v n im x
1
, x
2
, , x
n
ty trn on
I (a, b). Khi
i, Nu f

(x) > 0 vi mi x I (a, b) th


f(x
1
) +f(x
2
) + +f(x
n
) nf
_
x
1
+x
2
+ +x
n
n
_
.
ii, Nu f

(x) < 0 vi mi x I (a, b) th


f(x
1
) +f(x
2
) + +f(x
n
) nf
_
x
1
+x
2
+ +x
n
n
_
.
y I (a, b) nhm ngm nh l mt trong bn tp hp (a, b) , [a, b) ,
(a, b] , [a, b].
www.MATHVN.com - HOANG NGOC QUANG, Yen Bai
8
nh l 1.4. (Bt ng thc Chebyshev) Cho hai dy s thc n iu
cng chiu a
1
, a
2
, , a
n
v b
1
, b
2
, , b
n
. Khi ta c
a
1
b
1
+a
2
b
2
+a
n
b
n

1
n
(a
1
+a
2
+ +a
n
) (b
1
+b
2
+ +b
n
) . (1.6)
Nu hai dy s thc a
1
, a
2
, , a
n
v b
1
, b
2
, , b
n
n iu ngc chiu
th bt ng thc trn i chiu.
nh l 1.5. (Bt ng thc Nesbitt) Cho a, b, c l cc s thc dng.
Bt ng thc sau lun ng
a
b +c
+
b
c +a
+
c
a +b

3
2
. (1.7)
ng thc xy ra khi v ch khi a = b = c.
1.2. Cc ng thc v bt ng thc c bn trong tam gic
1.2.1. Cc ng thc c bn trong tam gic
nh l 1.6. (nh l hm s sin) Trong tam gic ABC ta c
a
sin A
=
b
sin B
=
c
sin C
= 2R.
nh l 1.7. (nh l hm s cosin) Trong tam gic ABC ta c
a
2
= b
2
+c
2
2bc cos A, b
2
= c
2
+a
2
2ca cos B, c
2
= a
2
+b
2
2ab cos C.
nh l 1.8. (Cc cng thc v din tch) Din tch tam gic ABC
c tnh theo mt trong cc cng thc sau
S =
1
2
ah
a
=
1
2
bh
b
=
1
2
ch
c
(1.8)
=
1
2
bc sin A =
1
2
ca sin B =
1
2
ab sin C (1.9)
= pr (1.10)
=
abc
4R
(1.11)
= (p a)r
a
= (p b)r
b
= (p c)r
c
(1.12)
=
_
p (p a) (p b) (p c). (1.13)
Cng thc (1.13) c gi l cng thc H-rng.
www.MATHVN.com - HOANG NGOC QUANG, Yen Bai
9
nh l 1.9. (nh l ng phn gic) Trong mt tam gic, ng
phn gic ca mt gc chia cnh i din thnh hai on thng t l vi
hai cnh k hai on y .
nh l 1.10. (Cng thc ng phn gic) Trong tam gic ABC ta
c
l
a
=
2bc
b +c
cos
A
2
, l
b
=
2ca
c +a
cos
B
2
, l
c
=
2ab
a +b
cos
C
2
.
nh l 1.11. (nh l ng trung tuyn) Trong mt tam gic, ba
ng trung tuyn gp nhau ti mt im c gi l trng tm ca
tam gic. Trn mi ng trung tuyn, khong cch t trng tm n
nh bng hai ln khong cch trng tm n chn ng trung tuyn.
nh l 1.12. (Cng thc ng trung tuyn) Trong tam gic ABC ta
c
m
2
a
=
b
2
+c
2
2

a
2
4
, m
2
b
=
c
2
+a
2
2

b
2
4
, m
2
c
=
a
2
+b
2
2

c
2
4
.
nh l 1.13. (Cng thc bn knh ng trn ni tip) Trong tam
gic ABC ta c
r = (p a) tan
A
2
= (p b) tan
B
2
= (p c) tan
C
2
.
nh l 1.14. (Cng thc bn knh ng trn bng tip) Trong tam
gic ABC ta c
r
a
= p tan
A
2
, r
b
= p tan
B
2
, r
c
= p tan
C
2
.
nh l 1.15. (Cc h thc lng gic c bn) Vi mi tam gic ABC
ta lun c cc h thc sau
sin A + sin B + sin C = 4 cos
A
2
cos
B
2
cos
C
2
, (1.14)
sin 2A + sin 2B + sin 2C = 4 sin Asin Bsin C, (1.15)
cos A + cos B + cos C = 1 + 4 sin
A
2
sin
B
2
sin
C
2
, (1.16)
cos 2A + cos 2B + cos 2C = 1 4 cos Acos Bcos C, (1.17)
www.MATHVN.com - HOANG NGOC QUANG, Yen Bai
10
sin
2
A + sin
2
B + sin
2
C = 2 (1 + sin Asin Bsin C) , (1.18)
cos
2
A + cos
2
B + cos
2
C = 1 2 cos Acos Bcos C, (1.19)
tan A + tan B + tan C = tan Atan Btan C, (1.20)
cot
A
2
+ cot
B
2
+ cot
C
2
= cot
A
2
cot
B
2
cot
C
2
, (1.21)
tan
A
2
tan
B
2
+ tan
B
2
tan
C
2
+ tan
C
2
tan
A
2
= 1, (1.22)
cot Acot B + cot Bcot C + cot C cot A = 1. (1.23)
Ring vi h thc (1.20) th tam gic ABC cn gi thit khng vung.
1.2.2. Cc bt ng thc c bn trong tam gic
nh l 1.16. (Bt ng thc tam gic) Trong tam gic ABC ta c
|b c| < a < b +c, |c a| < b < c +a, |a b| < c < a +b.
nh l 1.17. (Cc bt ng thc lng gic c bn) Vi mi tam gic
ABC ta lun c cc bt ng thc sau
sin A + sin B + sin C
3

3
2
, (1.24)
cos A + cos B + cos C
3
2
, (1.25)
cos
A
2
+ cos
B
2
+ cos
C
2

3

3
2
, (1.26)
sin
A
2
+ sin
B
2
+ sin
C
2

3
2
, (1.27)
sin
A
2
sin
B
2
sin
C
2

1
8
, (1.28)
cos Acos Bcos C
1
8
, (1.29)
sin
2
A + sin
2
B + sin
2
C
9
4
, (1.30)
tan
A
2
+ tan
B
2
+ tan
C
2

3, (1.31)
tan A + tan B + tan C 3

3, (1.32)
cot A + cot B + cot C

3. (1.33)
www.MATHVN.com - HOANG NGOC QUANG, Yen Bai
11
Ring vi bt ng thc (1.32) th tam gic ABC cn gi thit khng
vung. ng thc xy ra trong cc bt ng thc trn khi v ch khi
ABC l tam gic u.
1.3. Bt ng thc trong tam gic
Tam gic l hnh n gin nht trong cc a gic, mi a gic bt k
u c th chia thnh cc tam gic v s dng tnh cht ca n. V vy,
nghin cu cc bt ng thc trong tam gic s hu ch trong vic gii
quyt cc bt ng thc trong a gic. Trc ht, chng ta nghin cu
cc bt ng thc c bn sau y:
1.3.1. Bt ng thc v di cc cnh
nh l 1.18. Cho hai ng trn c bn knh ln lt l R v R

(R R

), khong cch gia tm ca chng bng d. iu kin cn v


hai ng trn ct nhau l R R

d R +R

.
Chng minh.
Hnh 1.1 Hai ng trn khng ct nhau.
R rng nu hai ng trn ngoi nhau (hnh 1.1 A) th ta c
R + R

< d. Nu hai ng trn cha nhau (hnh 1.1 B) th ta cng c


ngay d < RR

. Nu hai ng trn ct nhau ti mt im M th theo


bt ng thc v ba cnh ca tam gic OO

M, vi O v O

ln lt l
tm ca ng trn bn knh R v R

, ta c R R

d R +R

.
o li, nu R R

d R +R

th hai ng trn cho khng


th ngoi nhau hoc cha nhau c (nu khng phi c R + R

< d
hoc d < R R

). Do chng ch c th ct nhau.
www.MATHVN.com - HOANG NGOC QUANG, Yen Bai
12
nh l 1.19. Cc s dng a, b, c l di 3 cnh ca mt tam gic
khi v ch khi a +b > c, b +c > a, c +a > b.
Chng minh. Nu a, b, c l di 3 cnh ca tam gic th theo bt ng
thc v 3 cnh ca tam gic ta c a +b > c, b +c > a, c +a > b.
Ngc li, nu c a, b v c l 3 s thc dng tha mn a+b > c, b+c >
a, c + a > b, th ta c th chn hai im A v B trn mt phng cch
nhau mt khong c. Ly A v B lm tm dng hai ng trn bn knh
tng ng l a v b. T cc bt ng thc a +b > c, b +c > a, c +a > b
ta c |a b| < c < a + b. Theo nh l 1.18 th hai ng trn tm A
v B phi ct nhau ti mt im C. Vy a, b, c l di cc cnh ca
tam gic ABC theo cch dng trn.
nh l 1.20. Cho trc tam gic ABC v mt im M trong tam
gic. Khi ta c MB +MC < AB +AC.
Chng minh.
Hnh 1.2
Ko di BM v pha M ct
cnh AC ti im N. Theo nh
l 1.19 ta c
MB +MC < MB +MN +NC
=BN +NC < AB +AN +NC
=AB +AC.
Bi ton 1.1. Cho M l mt im nm trong tam gic ABC. Chng
minh rng
p < MA +MB +MC < 2p.
Trong p l na chu vi ca tam gic ABC.
Gii. p dng nh l 1.19 cho cc tam gic MAB, MBC v MCA ta c
AB < MA+MB, BC < MB+MC, CA < MC+MA. Cng theo v ba
bt ng thc trn ri chia c hai v cho 2 ta c p < MA+MB+MC.
Mt khc, theo nh l 1.20 ta c MA+MB < CA+CB, MB+MC <
AB+AC, MC +MA < BC +BA. Cng theo v ba bt ng thc trn
v em chia c hai v cho 2 ta c AM +BM +CM < 2p.
www.MATHVN.com - HOANG NGOC QUANG, Yen Bai
13
nh l 1.21. Trong mt tam gic ng vi gc ln hn l cnh di hn
v ngc li.
Chng minh. Xt tam gic ABC. Ta chng minh nu

ABC >

ACB th
AC > AB v ngc li.
Hnh 1.3
Tht vy, trong gc

ABC ta k tia Bx
to vi cnh BC gc bng gc

ACB. Do

ABC >

ACB, nn Bx ct cnh AC ti
im D v to thnh tam gic cn DBC,
do DB = DC. Mt khc, trong tam
gic ABD ta c AD + DB > AB. Do
AC = AD +DC = AD +DB > AB.
Phn ngc li ca nh l l hin nhin. V nu

ABC <

ACB th
ta phi c AC < AB l iu v l.
Bi ton 1.2. Chng minh rng ng vung gc AH h t im A
xung ng thng d cho trc lun nh hn ng xin AB.
Gii. Tam gic AHB l tam gic vung ti H, do

AHB = 90
0
>

ABH. Theo nh l trn, ta c AB > AH.


S tng ng gia ln cnh v gc cn ng cho cnh ca hai tam
gic khc nhau. Dng nh l 1.21 ta d dng chng minh kt qu sau.
nh l 1.22. Cho trc hai tam gic ABC v A

c hai cp cnh
bng nhau AB = A

v AC = A

. Ta c bt ng thc

BAC >

khi v ch khi BC > B

.
Chng minh.
Hnh 1.4
Trc ht, gi s rng

BAC >

, ta s chng minh BC > B

.
Khng mt tnh tng qut gi s
AB AC. Ta em hnh tam gic
ABC t chng ln hnh tam gic
A

sao cho A A

, C C

v nh B, B

nm cng pha so vi
ng thng i qua AC.
www.MATHVN.com - HOANG NGOC QUANG, Yen Bai
14
Do AB = A

, nn ta c

ABB

=

AB

B. V

CBB

<

ABB

CB

B >

AB

B, nn ta c

CBB

<

CB

B. Theo nh l 1.21, ta c
CB > CB

, hay l CB > C

.
Nu nh

BAC =

B

th ta cng d dng thy rng BC = B

,
do ABC v A

(c.g.c). Vy ta c

BAC >

B

khi v ch khi
BC > B

.
Bi ton 1.3. Cho tam gic ABC v AM l trung tuyn. Chng minh
rng

BAC 90
0
khi v ch khi AM
1
2
BC.
Gii. Gi A

l im i xng vi A qua trung im M ca cnh BC.


T gic ABA

C l t gic c hai ng cho ct nhau ti trung im


ca mi ng nn ABA

C l hnh bnh hnh.


Hnh 1.5
Xt hai tam gic ABA

v ABC c
cnh AB l cnh chung v c cp cnh
A

B v AC bng nhau. Theo nh l


1.22, ta c BC AA

khi v ch khi

BAC

ABA

. Do

BAC +

ABA

=
180
0
, cho nn

BAC

ABA

khi v
ch khi

BAC 90
0
. Tm li, AM =
1
2
AA


1
2
BC khi v ch khi

BAC
90
0
.
nh l 1.23. Trong nhng ng xin ni mt im M cho trc vi
im N trn mt ng thng d cho trc, ng xin no c hnh chiu
di hn th di hn.
1.3.2. Bt ng thc v cc i lng c bit
Trong mt tam gic, mi quan h gia cc cnh dn n mi quan h
vi cc i lng c bit. Vi ng cao ta d thy l ng cao tng
ng vi cnh ln hn th ngn hn. i vi ng trung tuyn v ng
phn gic ta cng s chng minh rng ng vi cnh di hn l ng
trung tuyn v ng phn gic ngn hn.
nh l 1.24. Trong tam gic ABC ng vi cnh di hn l ng cao,
ng trung tuyn v ng phn gic ngn hn.
www.MATHVN.com - HOANG NGOC QUANG, Yen Bai
15
Chng minh. Gi s c < b, ta s chng minh rng h
b
< h
c
, m
b
< m
c
v
l
b
< l
c
.
Hnh 1.6
V c < b nn suy ra h
b
=
S
2b
<
S
2c
= h
c
.
chng minh m
b
< m
c
, ta gi M, N v
P l trung im ca cc cnh AB, AC v
BC, tng ng (hnh 1.6). p dng nh
l 1.22 cho PAB v PAC l hai tam
gic c hai cp cnh bng nhau (AP chung
v BP = CP), ta c

APB <

APC. Gi G
l trng tm ca tam gic ABC. Xt hai
tam gic GPB v GPC l hai tam gic c hai cp cnh bng nhau
(GP chung v PB = PC). Do c

APB <

APC, nn BG < CG. Vy
m
b
=
3
2
BG <
3
2
CG = m
c
.
Hnh 1.7
Gi phn gic ca gc

B l BL v phn
gic xut pht t C l CK. Theo nh l
ng phn gic ta c
LC
LA
=
a
c
CL =
ab
a+c
. Tng t
KB
KA
=
a
b
BK =
ac
a+b
.
Do c < b, nn BK < CL. Dng hnh
bnh hnh BKCT (hnh 1.7), ta c

BTC =

BKC =

A +

C
2
v ta c

BTC <

BLC.
Mt khc, v TC = BK, v BK < CL
nn TC < CL. Trong tam gic TLC, ng
vi cnh ln hn l gc ln hn theo nh
l 1.21, cho nn

CLT <

CTL. T cc bt ng thc

BLC >

BTC v

CLT <

CTL, ta c

BLT <

BTL. Theo nh l 1.21 ta c BT > BL
m CK = BT suy ra CK > BL.
nh l 1.25. Trong tam gic ABC ta lun c
m
a
l
a
h
a
.
Chng minh. Gi H l chn ng cao, L l chn ng phn gic v M
l chn ng trung tuyn xut pht t nh A. Ta chng minh rng L
nm trn on thng ni HM, v p dng nh l 1.23 c bt ng
thc cn chng minh.
www.MATHVN.com - HOANG NGOC QUANG, Yen Bai
16
Hnh 1.8
nh l hin nhin ng cho trng hp
tam gic ABC cn ti nh A. tin
chng minh trong trng hp tam gic
khng cn ti A, khng mt tnh tng qut
ta gi s AB < AC. Gi A

i xng vi A
qua M, ta c BACA

l hnh bnh hnh.


Trong tam gic AA

C ta c AC >
A

C = AB v do theo nh l 1.21 ta
c

BAM =

MA

C >

CAM v im L
nm trong gc

BAM. Mt khc, do

BAH ph vi gc

B v

CAH ph
vi gc

C, cho nn

BAH <

CAH. Do L phi nm trong gc

CAH.
Tm li, im L nm gia im H v im M v ta c HM > HL.
Theo nh l 1.23, ta c AH < AL < AM.
nh l 1.26. ng trung tuyn AM ca tam gic ABC nh hn na
tng cc cnh AB v AC cng xut pht t mt nh A.
Chng minh. Gi A

l im i xng vi im A qua im M, ta c
ABA

C l hnh bnh hnh. Do AM =


1
2
AA

<
1
2
(A

C +AC) =
1
2
(AB +AC).
Bi ton 1.4. Chng minh rng nu M l im nm trn ng phn
gic ngoi ca gc C ca tam gic ABC (M khc C) th MA+MB >
CA +CB.
Gii.
Hnh 1.9
Gi s A

l im i xng
vi im A qua ng phn gic
ngoi ca gc C. Khi cc im
A

, C, B thng hng v MA

=
MA. Do MA+MB = MA

+
MB > A

B = CA

+ CB =
CA +CB.
www.MATHVN.com - HOANG NGOC QUANG, Yen Bai
17
1.4. Cc bt ng thc sinh ra t cc cng thc hnh hc
nh l 1.27. (Cng thc Euler) Gi R v r ln lt l bn knh ca
ng trn ngoi tip v ng trn ni tip tam gic ABC, d l khong
cch gia tm hai ng trn . Ta c
d
2
= R
2
2Rr. (1.34)
Chng minh.
Hnh 1.10
Gi O, I ln lt l tm ng
trn ngoi tip, ni tip ABC.
Bit rng ng trn ngoi tip
tam gic BCI c tm D l trung
im ca cung

BC. Gi M l
trung im ca BC v Q l hnh
chiu ca I trn OD. Khi
OB
2
OI
2
=OB
2
DB
2
+DI
2
OI
2
=OM
2
MD
2
+DQ
2
QO
2
=(MO +DM) (MO DM) + (DQ+QO) (DQQO)
=DO(MO DM +DQ+OQ) = R(2MQ) = 2Rr.
Vy OI
2
= R
2
2Rr, ngha l d
2
= R
2
2Rr.
H qu 1.4. (Bt ng thc Euler) K hiu R, r ln lt l bn knh
ng trn ngoi tip v bn knh ng trn ni tip tam gic ABC.
Khi
R 2r. (1.35)
ng thc xy ra khi v khi tam gic ABC u.
Bi ton 1.5. Cho tam gic ABC vi R l bn knh ng trn ngoi
tip, r l bn knh ng trn ni tip v p l na chu vi tam gic ABC.
Chng minh rng r
p
3

R
2
.
www.MATHVN.com - HOANG NGOC QUANG, Yen Bai
18
Gii. Ta c [ABC] =
abc
4R
= pr, suy ra 2p = a+b+c 3
3

abc = 3
3

4Rrp.
Do 8p
3
27(4Rrp) 27(8r
2
p), v R 2r. Vy p 3

3r.
Bt ng thc th hai,
p
3

3

R
2
tng ng vi a + b + c 3

3R.
S dng nh l hm s sin, bt ng thc ny tng ng vi sin A+
sin B + sin C
3

3
2
. Bt ng thc ny ng v hm s f(x) = sin x l
hm li trn (0, ), do
sin A+sin B+sin C
3
sin
_
A+B+C
3
_
= sin 60
0
=

3
2
.
nh l 1.28. (Cng thc Leibniz) Cho tam gic ABC vi di cc
cnh l a, b, c. Gi G l trng tm v (O, R) l ng trn ngoi tip
tam gic. Khi
OG
2
= R
2

1
9
_
a
2
+b
2
+c
2
_
. (1.36)
Chng minh.
Hnh 1.11
chng minh bi ton ny ta s dng
nh l Stewart "Nu L l im nm trn
cnh BC ca ABC v nu AL = l, BL =
m, LC = n, th a(l
2
+mn) = b
2
m+c
2
n".
p dng nh l Stewart cho OAA

,
trong A

l trung im ca BC, ta
c AA

(OG
2
+ AG.GA

) = A

O
2
.AG +
AO
2
.GA

. V AO = R, AG =
2
3
AA

, GA

=
1
3
AA

nn OG
2
+
2
9
A

A
2
=
2
3
A

O
2
+
1
3
R
2
.
Mt khc, v A

A
2
=
2(b
2
+c
2
)a
2
4
v A

O
2
= R
2

a
2
4
, ta c
OG
2
=
_
R
2

a
2
4
_
2
3
+
1
3
R
2

2
9
_
2(b
2
+c
2
) a
2
4
_
=R
2

a
2
6

2(b
2
+c
2
) a
2
18
= R
2

a
2
+b
2
+c
2
9
.

H qu 1.5. (Bt ng thc Leibniz) Cho tam gic ABC vi di


cc cnh l a, b, c. (O, R) l ng trn ngoi tip tam gic. Ta c bt
ng thc sau
9R
2
a
2
+b
2
+c
2
. (1.37)
ng thc xy ra khi v ch khi O l trng tm ca tam gic ABC.
www.MATHVN.com - HOANG NGOC QUANG, Yen Bai
19
Bi ton 1.6. Cho tam gic ABC vi di cc cnh l a, b, c. Chng
minh rng 4

3 [ABC]
9abc
a+b+c
.
Gii. S dng bt ng thc Leibniz vi lu 4R. [ABC] = abc ta c
9R
2
a
2
+ b
2
+ c
2

a
2
b
2
c
2
16[ABC]
2

a
2
+b
2
+c
2
9
4 [ABC]
3abc

a
2
+b
2
+c
2
. Mt
khc, Bt ng thc Cauchy-Schwarz cho a +b +c

a
2
+b
2
+c
2
.
Do 4

3 [ABC]
9abc
a+b+c
.
Bi ton 1.7. Gi s ng trn ni tip tam gic ABC tip xc cc
cnh AB, BC, CA ti D, E, F, tng ng.K hiu p l na chu vi ca
tam gic ABC. Chng minh rng EF
2
+FD
2
+DE
2

p
2
3
.
Gii. Thy rng ng trn ni tip tam gic ABC l ng trn ngoi
tip tam gic DEF. p dng bt ng thc Leibniz cho tam gic DEF,
ta c EF
2
+ FD
2
+ DE
2
9r
2
. Mt khc, theo bi ton 1.5 ta c
p
2
27r
2
. Do EF
2
+FD
2
+DE
2

p
2
3
.
nh l 1.29. (nh l Euler) Cho tam gic ABC ni tip trong ng
trn tm O, bn knh R. M l mt im bt k nm trong mt phng
tam gic. Gi X, Y, Z ln lt l hnh chiu vung gc ca M ln cc
cnh BC, CA, AB. Khi din tch ca tam gic XY Z c tnh theo
din tch tam gic ABC v khong cch MO bi cng thc sau
[XY Z] =
1
4

1
MO
2
R
2

[ABC] . (1.38)
Chng minh.
Hnh 1.12
Ko di AM, BM, CM ct ng trn
ngoi tip ti cc im X

, Y

, Z

tng
ng. Ta c

ZXM =

MBZ (t gic
BZMX ni tip),

MBZ =

ABY

(B, Z, A
thng hng v B, M, Y

thng hng),

ABY

=

AX

(cng chn cung

AY

).
T suy ra

ZXM =

AX

. Tng t

Y XM =

AX

. T suy ra

ZXY =

. Ta s k hiu hai gc ny l X v
www.MATHVN.com - HOANG NGOC QUANG, Yen Bai
20
X

. Ta c
[XY Z] =
1
2
XY.XZ. sin X
=
1
2
MC. sin C.MBsin B. sin X (nh l hm s sin)
=
1
2
MB.MY

.
MC
MY

sin B. sin C. sin X


=
1
2

MO
2
R
2

BC
Z

. sin Bsin C. sin X(phng tch, MBC MZ

)
=
1
2

MO
2
R
2

.BC. sin C. sin B.


sin X

=
1
8

1
MO
2
R
2

AC.BC. sin C (v sin B =


AC
2R
,
sin X

=
1
2R
)
=
1
4

1
MO
2
R
2

[ABC] .

Ch 1.1.
1) Tam gic XY Z nu trong nh l c gi l tam gic Pedal.
2) Nu M nm trn ng trn ngoi tip tam gic ABC th [XY Z] =
0. iu c ngha l tam gic XY Z suy bin thnh ng thng,
chnh l ng thng Euler.
3) Nu M I (I l tm ng trn ni tip tam gic ABC) th XY Z
l tam gic ni tip ng trn tm I, bn knh r c cc gc X, Y, Z
tng ng bng

2

A
2
,

2

B
2
,

2

C
2
. Bng cc php bin i s cp t
cng thc (1.38) s suy ra cng thc Euler OI
2
= R
2
2Rr.
H qu 1.6. Cho tam gic ABC v mt im M bt k nm trong mt
phng tam gic. Gi X, Y, Z ln lt l hnh chiu vung gc ca M ln
cc cnh BC, CA, AB. Khi
[XY Z]
1
4
[ABC] . (1.39)
ng thc xy ra khi v ch khi M l tm ca ng trn ngoi tip
tam gic ABC.
www.MATHVN.com - HOANG NGOC QUANG, Yen Bai
21
Bi ton 1.8. Cho t gic li ABCD ni tip trong ng trn tm O
(vi O nm bn trong t gic). Gi MNPQ l t gic m cc nh ln
lt l hnh chiu ca giao im 2 ng cho ca t gic ABCD n
cc cnh AB, BC, CD, DA. Chng minh rng
[MNPQ]
1
2
[ABCD] .
Gii. Gi K l giao im 2 ng cho AC v BD ca t gic ABCD.
Hnh 1.13
D thy KMN l tam gic Pedal dng
t im K ca tam gic ABC. Do
p dng h qu 1.6 ta c [KMN]
1
4
[ABC]. Lm tng t cho cc tam gic
KNP, KPQ, KQM v cng cc kt qu li
[KMN] + [KNP] + [KPQ] + [KQM]

1
4
([ABC] + [BCD] + [CDA] + [DAB]) .
Suy ra [MNPQ]
1
2
[ABCD].
ng thc xy ra khi v ch khi K l tm ca ng trn ngoi tip t
gic ABCD.
Bi ton 1.9. (Balkan, 1999) Cho ABC l mt tam gic nhn v
L, M, N l cc chn ng cao h t trng tm G ca ABC ti cc
cnh BC, CA, AB, tng ng. Chng minh rng
4
27
<
[LMN]
[ABC]

1
4
.
Gii. Ta c tam gic LMN l tam gic Pedal dng t trng tm
G ca tam gic ABC. p dng nh l 1.12, ta c [LMN] =
1
4

1
OG
2
R
2

[ABC] =
R
2
OG
2
4R
2
[ABC] (v G nm trong tam gic ABC).
+ D thy [LMN]
1
4
[ABC]. ng ng thc xy ra khi v ch khi
G O hay khi v ch khi tam gic ABC u.
+ Ta chng minh bt ng thc cn li. Tht vy, rng OG =
1
3
OH.
V tam gic nhn, H nm trong tam gic v OH R nn
[LMN]
[ABC]
=
R
2

1
9
OH
2
4R
2

R
2

1
9
R
2
4R
2
=
2
9
>
4
27
.
www.MATHVN.com - HOANG NGOC QUANG, Yen Bai
22
nh l 1.30. (Cng thc hnh bnh hnh) Cho t gic ABCD, gi x
l khong cch gia trung im ca hai ng cho AC v BD. Ta c
AB
2
+BC
2
+CD
2
+DA
2
= AC
2
+BD
2
+ 4x
2
. (1.40)
Chng minh. Gi M, N ln lt l trung im ca AC v BD. p dng
cng thc ng trung tuyn ta c
x
2
=
NA
2
+NC
2
2

AC
2
4
=
AB
2
+DA
2
2

BD
2
4
+
BC
2
+CD
2
2

BD
2
4
2

AC
2
4
hay AB
2
+BC
2
+CD
2
+DA
2
= AC
2
+BD
2
+ 4x
2
.
H qu 1.7. (Bt ng thc hnh bnh hnh) Cho t gic ABCD. Ta
c
AB
2
+BC
2
+CD
2
+DA
2
AC
2
+BD
2
. (1.41)
ng thc xy ra khi v ch khi t gic ABCD l hnh bnh hnh.
Bi ton 1.10. (a trung Hi, 2000) Cho P, Q, R, S l trung im ca
cc cnh BC, CD, DA, AB, tng ng, ca t gic li ABCD. Chng
minh rng
4
_
AP
2
+BQ
2
+CR
2
+DS
2
_
5
_
AB
2
+BC
2
+CD
2
+DA
2
_
.
Gii. Ta bit cng thc ng trung tuyn XM ca tam gic XY Z
l XM
2
=
1
2
XY
2
+
1
2
XZ
2

1
4
Y Z
2
. Ta thay b (X, Y, Z, M) bng
(A, B, C, P), (B, C, D, Q), (C, D, A, R), (D, A, B, S) vo trong cng thc
ny v cng 4 cng thc li vi nhau thu c cng thc th 5. Nhn
c hai v ca cng thc th 5 vi 4, ta c
4
_
AP
2
+BQ
2
+CR
2
+DS
2
_
= AB
2
+BC
2
+CD
2
+DA
2
+ 4
_
AC
2
+BC
2
_
.
Do ta ch cn chng minh bt ng thc
AC
2
+BC
2
AB
2
+BC
2
+CD
2
+DA
2
.
y l bt ng thc hnh bnh hnh. Du "=" xy ra khi v ch khi t
gic ABCD l hnh bnh hnh. iu phi chng minh.
www.MATHVN.com - HOANG NGOC QUANG, Yen Bai
23
1.5. Bt ng thc trong cc tam gic c bit
1.5.1. Cc bt ng thc trong tam gic u
Tam gic u c mt s tnh cht c bit, ni chung khng cn ng
trong mt tam gic ty . Trong mc ny, ta ch nghin cu mt s bt
ng thc trong tam gic u ABC lin quan mi quan h gia p
a
, p
b
, p
c
vi PA, PB, PC (trong p
a
, p
b
, p
c
ln lt l khong cch t P n
cc cnh BC, CA, AB).
Bi ton 1.11. Cho ABC l tam gic u cnh a, gi P l mt im
nm trong tam gic. Chng minh rng
Hnh 1.14
i)
1
p
a
+
1
p
b
+
1
p
c

3
a
,
ii)
1
p
a
+p
b
+
1
p
b
+p
c
+
1
p
c
+p
a

3
a
.
Gii. Gi D, E, F l chn ng vung
gc ca P ln cc cnh BC, CA, AB tng
ng. Ta c [ABC] = [BCP] + [CAP] +
[ABP], do ah = ap
a
+ ap
b
+ ap
c
. V
h =

3
2
a nn p
a
+p
b
+p
c
=

3
2
a. p dng
bt ng thc (1.2), ta c
1
p
a
+
1
p
b
+
1
p
c

9
p
a
+p
b
+p
c
=
6

3
a
.
Li p dng bt ng thc (1.2) c
1
p
a
+p
b
+
1
p
b
+p
c
+
1
p
c
+p
a

9
p
a
+p
b
+p
b
+p
c
+p
c
+p
a
=
3

3
a
.
Bi ton 1.12. Cho tam gic u ABC cnh a v P l mt im ty
nm trong tam gic. Chng minh rng
PA
2
.PB
2
.PC
2

8a
3
3

3
p
a
p
b
p
c
. (1.42)
Trc ht ta chng minh b sau
www.MATHVN.com - HOANG NGOC QUANG, Yen Bai
24
B 1.1. Cho P l mt im ty nm trong tam gic u ABC
cnh a. K hiu =

BPC, =

CPA, =

APB, ta c ng thc
PA
2
.PB
2
.PC
2
=
a
3
p
a
p
b
p
c
sin sin sin
. (1.43)
Chng minh. Vit li din tch tam gic BPC theo hai cch ta c
BP.CP. sin = a.p
a
, tng t CP.AP. sin = a.p
b
, AP.BP. sin = a.p
c
.
Nhn theo v 3 ng thc ny, ta thu c ng thc (1.43).
Gii. Gi f(x) = ln(sin x), x (0, ) .V f

(x) =
1
sin
2
x
< 0, nn f l
hm li. p dng bt ng thc Jensen, ta c
ln(sin ) +ln(sin ) +ln(sin )
3
ln(sin
+ +
3
).
Suy ra sin . sin . sin
3

3
8
. Thay bt ng thc ny vo (1.43) ta
c bt ng thc (1.42).
Bi ton 1.13. Cho tam gic u ABC cnh a v P l mt im ty
nm trong tam gic. Chng minh rng
PA.PB.PC 8p
a
p
b
p
c
. (1.44)
Gii. V [BPC] + [CPA] + [APB] = [ABC] nn
1
2
ap
a
+
1
2
ap
b
+
1
2
ap
c
=
a
2

3
4
. Suy ra p
a
+ p
b
+ p
c
=
a

3
2
. Do p
a
p
b
p
c

_
p
a
+p
b
+p
c
3
_
3
=
a
3

3
72
.
Thay bt ng thc ny vo (1.42), ta c bt ng thc (1.44).
Bi ton 1.14. Cho tam gic u ABC cnh a v P l mt im ty
nm trong tam gic. Chng minh rng
PA.PB +PB.PC +PC.PA a
2
. (1.45)
Gii. V

2
+

2
+

2
= 180
0
nn
cos + cos + cos
3
2
, (1.46)
du ng thc xy ra khi v ch khi = = = 120
0
.
By gi, p dng nh l hm s cosin cho tam gic PAB, ta c a
2
=
www.MATHVN.com - HOANG NGOC QUANG, Yen Bai
25
PA
2
+PB
2
2PA.PB. cos hay cos =
PA
2
+PB
2
a
2
2PA.PB
. Tng t cos =
PB
2
+PC
2
a
2
2PB.PC
, cos =
PC
2
+PA
2
a
2
2PC.PA
.
Cng theo v 3 bt ng thc trn v s dng (1.46), ta c
PA
2
+PB
2
a
2
2PA.PB
+
PB
2
+PC
2
a
2
2PB.PC
+
PC
2
+PA
2
a
2
2PC.PA
+
3
2
0,
Do
_
PA
2
.PB +PB
2
.PA+PA.PB.PC
_
+
_
PC
2
.PB +PB
2
.PC +PA.PB.PC
_
+
+
_
PA
2
.PC +PC
2
.PA+PA.PB.PC
_
a
2
(PA+PB +PC) 0,
tng ng vi
(PA +PB +PC)
_
PA.PB +PB.PC +PC.PA a
2
_
0.
T c bt ng thc PA.PB +PB.PC +PC.PA a
2
.
Du ng thc xy ra khi v ch khi P O.
Tam gic Pompeiu
Nm 1936, nh ton hc Rumani, Dimitrie Pompeiu pht hin kt
qu n gin nhng p sau y trong hnh hc phng Euclide
nh l 1.31. (nh l Pompeiu). Cho P l mt im ty nm
trong mt phng cha tam gic u ABC. Khi cc khong cch
PA, PB, PC l di cc cnh ca mt tam gic. Tam gic ny suy
bin nu im P nm trn ng trn ngoi tip tam gic ABC.
Chng minh.
Hnh 1.15
Xt trng hp im P khng nm trn
ng trn ngoi tip tam gic ABC. p
dng bt ng thc Ptolemy (Xem nh l
2.2) cho 4 im A, B, P, C ta c
PA.BC < PC.AB +PB.AC,
PB.AC < PA.BC +PC.AB,
PC.AB < PA.BC +PB.AC.
V tam gic ABC u nn AB = BC =
CA = l, do PA < PC +PB, PB < PA+PC, PC < PA+PB. Vy
PA, PB, PC l di 3 cnh ca mt tam gic.
www.MATHVN.com - HOANG NGOC QUANG, Yen Bai
26
Ch 1.2.
+ Tam gic vi di cc cnh bng PA, PB, PC c gi l tam
gic Pompeiu.
+ Khi P nm trong tam gic ABC, tam gic Pompeiu c th c
xy dng mt cch d dng nh sau: Quay tam gic ABP quanh tm A,
mt gc 60
0
, c tam gic AB

C. Khi AP = AB

= PB

, BP = CB

,
tam gic Pompeiu s l PCB

.
Bi ton 1.15. Cho tam gic u ABC cnh a v P l mt im ty
nm trong tam gic. Chng minh rng
3R
p

3 PA +PB +PC a

3. (1.47)
Trong R
p
l bn knh ng trn ngoi tip tam gic Pompeiu c
di cc cnh bng PA, PB, PC.
Gii. Gi M l trung im ca BC, theo bt ng thc tam gic AP +
PM AM, mt khc li c PM
PB+PC
2
. Suy ra 2PA+PB +PC
a

3, tng t 2PB+PC +PA a

3, 2PC +PA+PB a

3. Cng
theo v 3 bt ng thc ny ta c PA +PB +PC a

3.
p dng PA
2
+ PB
2
+ PC
2
9R
2
p
v bt ng thc Cauchy-Schwarz,
ta c 9R
2
p
PA
2
+PB
2
+PC
2

1
3
(PA +PB +PC)
2
.
Suy ra 3R
p

3 PA +PB +PC.
Bi ton 1.16. Cho P l mt im nm trong tam gic u ABC cnh
a. Gi T l din tch ca tam gic Pompeiu vi di cc cnh bng
PA, PB, PC. Khi
T =

3
12
_
a
2
3d
2
_
. (1.48)
Trong d = OP (O l tm ca tam gic u ABC).
Gii.
Ta thc hin 3 php quay tng t nh hnh 1.15, l quay tam
gic APB quanh tm A mt gc 60
0
, quay tam gic BPC quanh tm
B mt gc 60
0
v quay tam gic CPA quanh tm C mt gc 60
0
. Ta
s c mt hnh lc gic AB

CA

BC

, trong cc tam gic Pompeiu


PBA

, PAC

, PCB

c cng din tch T.


www.MATHVN.com - HOANG NGOC QUANG, Yen Bai
27
Hnh 1.16
V APC = BA

C, APB =
AB

C, AC

B = BPC nn din tch


ca lc gic = 2. [ABC] =
2a
2

3
4
. Mt
khc APB

, BPC

, CPA

l cc tam gic
u. Do
2a
2

3
4
= 3T+
PA
2

3
4
+
PB
2

3
4
+
PC
2

3
4
Lun c
PA
2
+PB
2
+PC
2
= 3OP
2
+OA
2
+OB
2
+OC
2
V OP = d, OA = OB = OC =
a
2

3
3
nn
PA
2
+PB
2
+PC
2
= 3d
2
+a
2
. T ta c cng thc (1.48).
Nhn xt 1.1. T cng thc (1.47) v (1.48) ta c cc h qu sau:
1) T

3
12
a
2
. ng thc xy ra khi v ch khi P O.
2) r
p

a
6

R
p
2
, trong R
p
, r
p
ln lt l bn knh vng trn ngoi
tip, ni tip tam gic Pompeiu vi di cc cnh bng PA, PB, PC.
1.5.2. Cc bt ng thc trong tam gic vung v tam gic
cn
Bi ton 1.17. Cho gc vung xAy. B l im trn tia Ax, C l im
trn tia Ay (B = A; C = A). Chng minh rng AB +

3AC 2BC.
Gii.
Hnh 1.17
Trong gc xAy v tia Az sao cho

xAz = 30
0
, do

yAz = 60
0
. V
BHAz, CKAz(H, K Az), Az ct
BC ti I. Xt ABH c

AHB =
90
0
,

BAH = 30
0
nn l na tam gic u,
cnh AB. Suy ra BH =
1
2
AB m BH
BI. Do
AB 2BI. (1.49)
www.MATHVN.com - HOANG NGOC QUANG, Yen Bai
28
Xt ACK c

AKC = 90
0
,

CAK = 60
0
nn l na tam gic u, cnh
AC. Suy ra CK =
AC

3
2
m CK IC. Do

3AC 2IC. (1.50)


Cng theo v (1.49) v (1.50) c AB +

3AC (BI +CI) = 2BC.


Bi ton 1.18. Cho tam gic ABC cn ti A. D l im trn cnh BC,
E l im trn trn tia i ca tia CB sao cho CE = BD. Chng minh
rng AD +AE > 2AB.
Gii.
Hnh 1.18
Trn tia i ca tia BC ly im
F sao cho BF = BD. p dng
nh l 1.26 ta c AD + AF > 2AB.
Mt khc, xt ABF v ACE c
AB = AC,

ABF =

ACE (v

ABC =

ACB,

ABC+

ABF =

ACE+

ACB =
180
0
), BF = CE. Do ABF =
ACE (c.g.c). Suy ra AF = AE. Vy
AD +AE > 2AB.
Bi ton 1.19. (Iran, 2005) Cho tam gic ABC vung ti A. Gi D l
giao im ca phn gic trong ca gc

A vi cnh BC v I
a
l tm ca
ng trn bng tip cnh BC ca tam gic ABC. Chng minh rng
AD
DI
a

2 1.
Gii.
Hnh 1.19
Gi E, F ln lt l im tip xc ca
ng trn bng tip vi cc cnh AB, AC
tng ng. Ta c r
a
= I
a
E = AF = FI
a
=
p, trong p l na chu vi ca tam gic
ABC. Hn na,
AD
DI
a
=
h
a
r
a
. V ah
a
= bc,
ta c
AD
DI
a
=
h
a
r
a
=
bc
ap
=
_
bac
4R
_ _
4Rr
a
2
_
_
1
rp
_
=
4Rr
a
2
. V 2R = a v 2r = b + c a nn
www.MATHVN.com - HOANG NGOC QUANG, Yen Bai
29
AD
DI
a
=
b+ca
a
=
b+c
a
1. Mt khc a =

b
2
+c
2

_
(b+c)
2
2
=
b+c

2
hay
b +c a

2. Do
AD
DI
a

2 1.
Bi ton 1.20. (Rumani, 2007) Cho ABC l mt tam gic vung cn
ti A. Vi im P ty nm trong tam gic, xt ng trn tm A v
bn knh AP ct cc cnh AB v AC ti M v N, tng ng. Hy xc
nh v tr ca P MN +BP +CP t gi tr nh nht.
Gii.
Hnh 1.20
Xt im Q trn ng trung trc
ca BC tha mn AQ = AP. Gi S
l giao im ca BP v tip tuyn vi
ng trn ti Q. Khi SP +PC
SC. Do BP + PC = BS + SP +
PC BS +SC.
Mt khc, BS +SC BQ+QC, nn
BP + PC t gi tr nh nht nu
P Q.
Gi T l trung im ca MN. V
AMQ cn v MT l mt trong
nhng chiu cao ca n, khi MT =
ZQ trong Z l chn ng vung
gc h t Q xung AB. Khi MN + BQ + QC = 2(MT + QC) =
2(ZQ+QC) t gi tr nh nht khi Z, Q, C thng hng v iu ny c
ngha CZ l chiu cao. Bng php i xng, BQ cng l chiu cao v
do P l trc tm.
1.6. Cc bt ng thc khc trong tam gic
Bi ton 1.21. (Bt ng thc Weitzenbock, IMO 1961) Gi a, b, c l
di cc cnh ca mt tam gic. Chng minh rng
a
2
+b
2
+c
2
4

3S. (1.51)
Gii. Theo bt ng thc AM - GM ta c
S =
_
p(p a)(p b)(p c)
_
p
_
(pa)+(pb)+(pc)
3
_
3
=

3
9
p
2
.
www.MATHVN.com - HOANG NGOC QUANG, Yen Bai
30
Do , theo bt ng thc Cauchy - Schwarz ta c
4

3S 4

3
_

3
9
_
a +b +c
2
_
2
_
=
1
3
(a +b +c)
2

1
3
_
a
2
+b
2
+c
2
_ _
1
2
+ 1
2
+ 1
2
_
= a
2
+b
2
+c
2
.
Bi ton sau cho ta mt kt qu mnh hn bt ng thc Weitzenbock.
Bi ton 1.22. (Bt ng thc Hadwiger - Finsler) Gi a, b, c l di
cc cnh ca mt tam gic. Chng minh rng
a
2
+b
2
+c
2
4

3S + (a b)
2
+ (b c)
2
+ (c a)
2
(1.52)
ng thc xy ra khi v ch khi a = b = c.
Gii. V a
2
= b
2
+c
2
2bccosA v S =
1
2
bcsinA nn ta c
a
2
+b
2
+c
2
4

3S = 2(b
2
+c
2
) 2bccosA 2

3bcsinA
=2(b
2
+c
2
) 4bccos
_

3
A
_
2(b
2
+c
2
2bc) = 2 (b c)
2
. (1.53)
Mt khc, khng mt tnh tng qut ta gi s b a c. Khi ta c
bt ng thc
(b c)
2
(a b)
2
+ (c a)
2
. (1.54)
V bt ng thc trn tng ng vi bt ng thc (ab)(ac) 0.
T bt ng thc (1.53) v (1.54) ta c bt ng thc (1.52).
ng thc xy ra khi v ch khi
_
_
_
cos
_

3
A
_
= 1
_
a = b
a = c
a = b = c.
Bi ton 1.23. (im Torricelli) Tm im O trong tam gic ABC cho
trc sao cho tng khong cch t im O ti ba nh ca tam gic l
nh nht c th.
Gii. Xt lm hai trng hp:
a) Tam gic ABC c ba gc nh hn 120
0
.
www.MATHVN.com - HOANG NGOC QUANG, Yen Bai
31
Dng tam gic u BCD pha ngoi ca tam gic ABC. Gi T l
giao im ca ng trn ngoi tip tam gic BCD vi AD. D chng
minh rng T nhn ba cnh ca tam gic ABC di ba gc bng nhau.
Ta chng minh rng vi mt im O ty trong tam gic ABC khc
im T th ta c OA +OB +OC TA +TB +TC.
Hnh 1.21
im T c gi l im
Torricelli ca tam gic ABC
v c tng cc khong cch
ti cc nh ca tam gic
ABC nh nht.
Tht vy, theo nh l
Pompeiu, ta c OB +OC
OD, do
OA +OB +OC OA +OD AD. (1.55)
Mt khc, v T nm trn ng trn ngoi tip tam gic u BCD nn
TA +TB +TC = TA +TD = AD. (1.56)
T (1.55) v (1.56) suy ra OA + OB + OC TA + TB + TC. ng
thc xy ra khi v ch khi O T.
b) Tam gic ABC c mt gc, chng hn

B > 120
0
Hnh 1.22
Dng tam gic u BCD pha ngoi
tam gic ABC. Do

B > 120
0
, cho nn vi
im O ty trong tam gic ABC, im
B nm trong tam gic ODA. Theo nh l
Pompeiu, ta c OB +OC OD.
Mt khc, theo nh l 1.20 i vi tam
gic ODA, ta c OA + OD BA + BD.
T ta c OA + OB + OC OA + OD BA + BD = BA + BC.
Nh vy, khi O B, tng khong cch t O ti cc nh ca tam gic
ABC l nh nht c th.
Tm li, trong trng hp tam gic ABC c mt nh khng nh hn
120
0
, th chnh nh ny l nh cn tm.
www.MATHVN.com - HOANG NGOC QUANG, Yen Bai
32
Bi ton 1.24. Cho tam gic ABC ni tip ng trn (O) vi
H l trc tm v AD, BE, CE l cc ng cao. K hiu D

=
AD (O), E

= BE (O), F

= CF (O). Chng minh rng


(i)
AD
HD
+
BE
HE
+
CF
HF
9, (ii)
HD
HA
+
HE
HB
+
HF
HC

3
2
,
(iii)
AD
DD

+
BE
EE

+
CF
FF

9, (iv)
AD
AD

+
BE
BE

+
CF
CF


9
4
.
Gii.
Hnh 1.23
+ Chng minh i).
Ta c
[HBC]
[ABC]
=
HD
AD
,
[HCA]
[ABC]
=
HE
BE
,
[HAB]
[ABC]
=
HF
CF
nn
HD
AD
+
HE
BE
+
HF
CF
= 1. By gi, p
dng bt ng thc (1.2) ta c
AD
HD
+
BE
HE
+
CF
HF

9
HD
AD
+
HE
BE
+
HF
CF
= 9.
+ Chng minh ii).
Ta c
HD
HA
=
HD
ADHD
=
[HBC]
[ABC][HBC]
=
[HBC]
[HCA]+[HAB]
. tng t,
HE
HB
=
[HCA]
[HAB]+[HBC]
,
HF
HC
=
[HAB]
[HBC]+[HCA]
. Cng 3 bt ng thc
ny, sau p dng bt ng thc Nesbitt
ta c bt ng thc
HD
HA
+
HE
HB
+
HF
HC

3
2
.
+ Chng minh iii).
D chng minh c HD = DD

, HE = EE

v HF = FF

. Thay vo
(i) ta c bt ng thc
AD
DD

+
BE
EE

+
CF
FF

9.
+ Chng minh iv).
Ta c
AD

AD
=
AD+DD

AD
= 1 +
HD
AD
. Tng t,
BE

BE
= 1 +
HE
BE
,
CF

CF
= 1 +
HF
CF
.
Cng 3 ng thc ny ta c
AD

AD
+
BE

BE
+
CF

CF
= 3 +
HD
AD
+
HE
BE
+
HF
CF
=
4. By gi p dng bt ng thc (1.2) ta c
AD
AD

+
BE
BE

+
CF
CF


9
AD

AD
+
BE

BE
+
CF

CF
=
9
4
.
Bi ton 1.25. (Tp ch THTT s 266) Gi s M l im nm trong
tam gic ABC. Gi A
1
, B
1
, C
1
ln lt l hnh chiu ca M trn cc
ng thng BC, CA, AB. Chng minh rng
MA
2
(MB
1
+MC
1
)
2
+
MB
2
(MC
1
+MA
1
)
2
+
MC
2
(MA
1
+MB
1
)
2
3.
www.MATHVN.com - HOANG NGOC QUANG, Yen Bai
33
ng thc xy ra khi no ?
Gii.
Hnh 1.24
Ta c
MB
1
+MC
1
MA
=
MB
1
MA
+
MC
1
MA
=sin

MAB
1
+ sin

MAC
1
2 sin

MAB
1
+

MAC
1
2
= 2 sin
A
2
.
Suy ra
MA
MB
1
+MC
1

1
2 sin
A
2
. Tng t ta c
MB
MC
1
+MA
1

1
2 sin
B
2
,
MC
MA
1
+MB
1

1
2 sin
C
2
. Vy
MA
2
(MB
1
+MC
1
)
2
+
MB
2
(MC
1
+MA
1
)
2
+
MC
2
(MA
1
+MB
1
)
2

1
4
_
1
sin
2 A
2
+
1
sin
2 B
2
+
1
sin
2 C
2
_

3
4
3

1
sin
2 A
2
sin
2 B
2
sin
2 C
2

3
4
3

1
_
1
8
_
2
= 3.
ng thc xy ra khi v ch khi ABC u v M l tm ca tam gic.
Bi ton 1.26. (Trch thi hc sinh gii Quc gia, 1991) Cho tam
gic ABC vi trng tm l G v ni tip trong ng trn bn knh R,
cc ng trung tuyn ca tam gic ny xut pht t cc nh A, B, C
ko di ct ng trn ln lt ti D, E, F. Chng minh rng
3
R

1
GD
+
1
GE
+
1
GF

3
_
1
AB
+
1
BC
+
1
CA
_
.
Gii. Gi M, N, P ln lt l trung im ca cnh BC, CA, AB. T
AM.MD = BM.MC c MD =
a
2
4m
a
. Suy ra
GD = GM +MD =
m
a
3
+
a
2
4m
a
(1.57)
www.MATHVN.com - HOANG NGOC QUANG, Yen Bai
34
Hnh 1.25
T (1.57), p dng bt ng thc AM
GM ta c GD 2
_
a
2
12
=
a

3

1
GD

3
BC
. Tng t, c
1
GE

3
CA
,
1
GF

3
AB
.
Cng theo v 3 bt ng thc ny ta c
1
GD
+
1
GE
+
1
GF

3
_
1
AB
+
1
BC
+
1
CA
_
.
Li t (1.57) c
GA
GD
=
2m
a
3
_
m
a
3
+
a
2
4m
a
_
=
8m
2
a
4m
2
a
+ 3a
2
. p dng cng thc ng
trung tuyn c
GA
GD
=
2b
2
+ 2c
2
a
2
a
2
+b
2
+c
2
.
Tng t tnh
GB
GE
v
GC
GF
ri cng li c
GA
GD
+
GB
GE
+
GC
GF
=
3a
2
+3b
2
+3c
2
a
2
+b
2
+c
2
= 3.
T
AD
GD
+
BE
GE
+
CF
GF
= 6. rng AD, BE, CF u khng ln hn
2R, thay vo ta c
1
GD
+
1
GE
+
1
GF

3
R
.
Bi ton 1.27. (Ty ban nha, 1998) Mt ng thng cha trng tm
G ca tam gic ABC ct cnh AB ti P v cnh CA ti Q. Chng minh
rng
PB
PA
.
QC
QA

1
4
.
Gii. V
PB
PA
.
QC
QA

1
4
_
PB
PA
+
QC
QA
_
2
, ta s chng minh
PB
PA
+
QC
QA
= 1.
Hnh 1.26
V BB

, CC

song song vi trung


tuyn AA

m B

, C

nm trn PQ.
Cc tam gic APG v BPB

ng
dng ; tam gic AQG v CQC

cng
ng dng, do
PB
PA
=
BB

AG
v
QC
QA
=
CC

AG
. Cng 2 ng thc ny li vi lu
rng AG = 2GA

= BB

+ CC

ta
c
PB
PA
+
QC
QA
= 1. T c bt
ng thc cn chng minh.
Bi ton 1.28. (Ba Lan, 1999) Cho D l mt im trn cnh BC
ca tam gic ABC sao cho AD > BC. im E trn CA tha mn
AE
EC
=
BD
AD BC
. Chng minh rng AD > BE.
Gii.
www.MATHVN.com - HOANG NGOC QUANG, Yen Bai
35
Hnh 1.27
Ly F trn AD sao cho AF = BC
v Gi E

l giao im ca BF v
AC. p dng nh l hm s sin cho
tam gic AE

F, BCE

v BDF, ta
c
AE

C
=
AF. sin

AFE

sin

AE

F
.
sin

BE

C
BC. sin

CBE

=
sin

BFD
sin

DBF
=
BD
FD
=
AE
EC
. Do E

E.
Ly G trn BD sao cho BG = AD
v H l giao im ca GE vi ng
thng qua A v song song vi BC. D thy cc tam gic ECG v EAH
ng dng nn
AH
CG
=
AE
EC
=
BD
ADBC
=
BD
BGBC
=
BD
CG
, suy ra AH = DB.
Do BDAH l mt hnh bnh hnh, suy ra BH = AD v BHG cn.
Vy BH = BG = AD > BE.
Bi ton 1.29. (Tp ch THTT s 265) Gi AD, BE, CF l cc ng
phn gic trong ca tam gic ABC. Chng minh rng
p(DEF)
1
2
p(ABC),
trong k hiu p(XY Z) l chu vi ca tam gic XY Z. ng thc xy
ra khi no ?
Gii.
Hnh 1.28
T tnh cht ca ng
phn gic BE ta c
AE
CE
=
c
a
,
suy ra
AE
b
=
AE
AE+CE
=
c
a+c
.
Do AE =
bc
a+c
. Tng t
AF =
bc
a+b
.
Theo nh l cosin
trong AEF v ABC
c
EF
2
=AE
2
+AF
2
2AF.AF. cos A
=
_
bc
a +c
_
2
+
_
bc
a +b
_
2
2
b
2
c
2
(a +c) (a +b)
.
b
2
+c
2
a
2
2bc
www.MATHVN.com - HOANG NGOC QUANG, Yen Bai
36
=
a
2
bc
(a +c) (a +b)

abc (a +b +c) (b c)
2
(a +c)
2
(a +b)
2
Suy ra EF
2

a
2
bc
(a+c)(a+b)
. T
EF
2

a
2
bc
4

ac.

ab
=
1
4
.

ac.

ab
1
4
_

ac +

ab
2
_
2

1
16
_
a +c
2
+
a +b
2
_
2
=
1
16
_
2a +b +c
2
_
2
.
Do EF
2a+b+c
8
. Tng t FD
2b+c+a
8
, DE
2c+a+b
8
.
Cng theo v 3 bt ng thc trn c DE+EF +FD
1
2
(a +b +c).
hay p(DEF)
1
2
p(ABC).
Bi ton 1.30. (IMO, 1991 ) Cho tam gic ABC. Gi I l tm ng
trn ni tip tam gic. ng phn gic trong ca cc gc A, B, C ln
lt ct cc cnh i din tng ng ti L, M, N. Chng minh rng
1
4

AI.BI.CI
AL.BM.CN

8
27
.
Gii.
Hnh 1.29
S dng tnh cht ng phn
gic c
BL
LC
=
c
b
, rng BL +
LC = a, ta c BL =
ac
b+c
v
LC =
ab
b+c
. Tip tc, p dng tch
cht ng phn gic cho phn
gic BI ca gc

ABL ta thu c
IL
AI
=
BL
AB
=
ac
(b+c)c
=
a
b+c
. Do
AL
AI
=
AI+IL
AI
= 1 +
IL
AI
= 1 +
a
b+c
=
a+b+c
b+c
. Khi ,
AI
AL
=
b+c
a+b+c
. Tng
t,
BI
BM
=
c+a
a+b+c
,
CI
CN
=
a+b
a+b+c
. Do bt ng thc cn chng minh a
v dng cha cc bin a, b v c
1
4
<
(b +c)(c +a)(a +b)
(a +b +c)
3

8
27
.
www.MATHVN.com - HOANG NGOC QUANG, Yen Bai
37
p dng bt ng thc AM GM c
(b +c)(c +a)(a +b)
_
(b +c) + (c +a) + (a +b)
3
_
3

8
27
(a +b +c)
3
.
Bt ng thc phi c chng minh.
chng minh bt ng thc tri, trc ht rng
(b +c)(c +a)(a +b)
(a +b +c)
3
=
(a +b +c)(ab +bc +ca) abc
(a +b +c)
3
. (1.58)
Bit rng a +b +c = 2p, ab +bc +ca = p
2
+r
2
+ 4rR, abc = 4Rrp thay
vo (1.58) c
(b +c)(c +a)(a +b)
(a +b +c)
3
=
2p(p
2
+r
2
+ 4rR) 4Rrp
8p
3
=
=
2p
2
+ 2pr
2
+ 4Rrp
8p
3
=
1
4
+
2r
2
+ 4Rr
8p
2
>
1
4
.
Bi ton 1.31. (IMO Shorlist, 1996) Cho ABC l tam gic u v
P l mt im trong n. Cc ng thng AP, BP, CP ct cc cnh
BC, CA, AB ti cc im A
1
, B
1
, C
1
, tng ng. Chng minh rng
A
1
B
1
.B
1
C
1
.C
1
A
1
A
1
B.B
1
C.C
1
A.
Gii. p dng nh l hm s cosin cho CA
1
B
1
, ta c
A
1
B
2
1
= A
1
C
2
+B
1
C
2
A
1
C.B
1
C A
1
C.B
1
C.
Tng t, B
1
C
2
1
B
1
A.C
1
A, C
1
A
1
C
1
B.A
1
B. Nhn 3 bt ng thc
ny, ta c
A
1
B
2
1
.B
1
C
2
1
.C
1
A
2
1
A
1
C.B
1
C.B
1
A.C
1
A.C
1
B.A
1
B. (1.59)
By gi, cc ng thng AA
1
, BB
1
, CC
1
ng quy, v vy p dng nh
l Ceva ta c A
1
B.B
1
C.C
1
A = AB
1
.BC
1
.CA
1
, thay vo (1.59) ta thu
c bt ng thc cn chng minh. ng thc xy ra khi v ch khi
CA
1
= CB
1
, BA
1
= BC
1
v AB
1
= AC
1
. iu ny xy ra khi v ch khi
P l tm ca ng trn ngoi tip tam gic ABC.
www.MATHVN.com - HOANG NGOC QUANG, Yen Bai
38
Bi ton 1.32. (IMO Shorlist, 1999) Cho tam gic ABC v M l mt
im nm trong n. Chng minh rng
min {MA, MB, MC} +MA +MB +MC < AB +BC +CA.
Trc ht, ta chng minh b sau:
B 1.2. Nu M l mt im nm trong t gic li ABCD th
MA +MB < AD +DC +CB.
Chng minh.
Hnh 1.30 Hnh 1.31
Gi N l giao im ca AM v CD (Hnh 1.30). Khi MA+MB <
MA + MN + NB AN + NC + CB AD + DN + NC + CB =
AD +DC +CB.
Gii. Ly D, E, F theo th t l trung im ca BC, CA, AB (Hnh
1.31). Xt im M nm trong tam gic, M s thuc t nht hai trong ba
hnh thang ABDE, BCEF, CAFD. Khng mt tnh tng qut, gi s
M ABDE, BCEF. p dng b trn ta c
MA +MB < AE +ED +DB,
MB +MC < BF +FE +EC.
Cng theo v hai bt ng thc trn, ta c
MB + (MA +MB +MC) < AB +BC +CA.
Vy min {MA, MB, MC} +MA +MB +MC < AB +BC +CA.
www.MATHVN.com - HOANG NGOC QUANG, Yen Bai
39
Bi ton 1.33. (IMO Shorlist, 2002) Cho tam gic ABC v F l mt
im trong n tha mn

AFB =

BFC =

CFA . Cc ng thng BF
v CF ct cc cnh AC v AB ti D v E, tng ng. Chng minh
rng
AB +AC 4DE.
Trc ht, ta chng minh b sau:
B 1.3. Cho tam gic ABC, cc im P v Q nm trn cc tia FD,
FE tng ng, sao cho PF DF, QF EF, trong > 0. Nu

PFQ 90
0
th PQ DE.
Chng minh. t

PFQ = . V 90
0
, ta c cos 0. By gi, p
dng nh l hm s cosin, ta c PQ
2
= PF
2
+QF
2
2PF.QF. cos


(DF)
2
+ (EF)
2
2 (DF) . (EF) . cos

= (DE)
2
. Do PQ
DE.
Gii. Lu rng

AFE =

BFE =

CFD =

AFD = 60
0
. Gi P, Q l giao
im ca cc ng thng BF, EF vi ng trn ngoi tip tam gic
CFA, AFB tng ng. Khi , d dng thy c hai tam gic CPA, AQB
l u. Gi P
1
l chn ng vung gc h t F xung cnh AC v gi
s ng trung trc ca AC ct ng trn ngoi tip tam gic CFA
ti P v P
2
.
Hnh 1.32
Gi M l trung im
ca AC. Khi
PD
DF
=
PM
FP
1

PM
MP
2
= 3 v
vy PF 4DF. Tng
t, ta c QF 4EF.
p dng h qu trn
vi = 4 v =

DEF = 120
0
ta c
PQ 4DE. Cui cng,
p dng bt ng thc
tam gic, AB +AC = AQ+AP PQ 4DE.
Bi ton 1.34. (IMO, 2006) Cho tam gic ABC vi I l tm ng
trn ni tip. Mt im P nm trong tam gic tha mn

PBA+

PCA =
www.MATHVN.com - HOANG NGOC QUANG, Yen Bai
40

PBC +

PCB. Chng minh rng AP > AI v ng thc xy ra khi v


ch khi P I.
Gii. t

A = ,

B = ,

C = . V

PBA +

PCA +

PBC +

PCB =
+ nn t iu kin bi ton ta c

PBC +

PCB =
+
2
. Suy ra

BPC = 90
0
+

2
.
Hnh 1.33
Mt khc,

BIC = 180
0

+
2
=
90
0
+

2
. Do

BPC =

BIC, v v
P v I nm cng pha vi BC nn
cc imB, I, P, C cng nm trn mt
ng trn. Ni cch khc, P nm
trn ng trn ngoi tip tam gic
BCI. Gi l ng trn ngoi tip
tam gic ABC. R rng tm ca
l trung im M ca cung BC ca
. y cng l giao im th hai ca
phn gic AI v . T tam gic APM
ta c AP +PM AM = AI +IM = AI +PM. Do AP AI. ng
thc xy ra khi v ch khi P nm trn on AM, iu ny xy ra khi v
ch khi P I.
1.7. Cc bt ng thc trong t gic
K hiu ABCD l t gic li vi cc nh l A, B, C, D c v theo
mt chiu nht nh no (cng chiu kim ng h hay ngc chiu
kim ng h). n gin, ln ca gc ng vi cc nh A, B, C, D
cng c k hiu l A, B, C, D.
di cc cnh ca tam gic: AB = a, BC = b, CD = c, DA = d.
Na chu vi ca t gic: p =
a +b +c +d
2
.
di cc ng cho: AC = m, BD = n.
Din tch ca t gic: S = S
ABCD
hay [ABCD]
www.MATHVN.com - HOANG NGOC QUANG, Yen Bai
41
1.7.1. Cc bt ng thc c bn trong t gic
Bi ton 1.35. Cho t gic ABCD, c AB+BD AC +DC. Chng
minh AB < AC.
Gii. Gi O l giao im ca AC v BD . Xt cc tam gic OAB v
ODC ta c
Hnh 1.34
AB < OA + OB, DC < OC + OD.
Do
AB +CD <(OA +OC) + (OB +OD)
=AC +BD. (1.60)
Mt khc, theo gi thit ta c
AB +BD AC +DC. (1.61)
Cng theo v (1.60) v (1.61) ta c 2AB+DC +BD < 2AC +BD+
DC. Suy ra AB < AC.
Bi ton 1.36. Tng hai ng cho ca mt t gic li ABCD nh
hn chu vi ca t gic v ln hn na chu vi ca n.
Gii. Gi O l giao im ca AC v BD, ta c
AC +BD =(OA +OB) + (OC +OD) > AB +CD. (1.62)
AC +BD =(OA +OD) + (OB +OC) > AD +BC. (1.63)
Cng theo v cc bt ng thc (1.62) v (1.63) ta thu c
AC +BD >
AB +BC +CD +DA
2
.
Mt khc, AC < AB + BC v AC < DA + CD. Cng theo v hai bt
ng thc ny, ta c
AC <
AB +BC +CD +DA
2
. (1.64)
Tng t,
BD <
AB +BC +CD +DA
2
. (1.65)
Cng theo v hai bt ng thc (1.64) v (1.65) , ta thu c
AC +BD < AB +BC +CD +DA.
www.MATHVN.com - HOANG NGOC QUANG, Yen Bai
42
Bi ton 1.37. Cho t gic ABCD, gi M, N, P, Q ln lt l trung
im ca cc cnh AB, BC, CD, DA. Chng minh rng
a)MP
AD +BC
2
. (1.66)
b)MP +NQ
AB +BC +CD +DA
2
. (1.67)
Gii.
Hnh 1.35
a) Gi I l trung im ca BD, MI
l ng trung bnh ca ABD. Suy ra
MI =
AD
2
. IP l ng trung bnh ca
DBC, suy ra IP =
BC
2
. Xt 3 im
I, M, P ta c MP MI + IP, suy ra
MP
AD+BC
2
.
b) p dng a) ta c NQ
AB+DC
2
.
Do MP + NQ
AD+BC
2
+
AB+DC
2
=
AB+BC+CD+DA
2
.
Bi ton 1.38. Cho hnh vung ABCD cnh a. M, N l hai im
trong hnh vung cho. Chng minh rng MN a

2.
Gii.
Hnh 1.36
V ABCD l hnh vung cnh
a nn AC = AB

2 = a

2.
V ng trn (O) ngoi tip
hnh vung ABCD. Ta c ng
knh ca (O) l a

2, M v N l
hai im nm trong O.
Gi M

l dy cung i qua M
v N. Ta c MN M

m
M

2 (ng kinh l dy
cung ln nht trong ng trn).
Do MN a

2.
Bi ton 1.39. Cho hnh thang ABCD c y nh l AB v

C +

D
90
0
. Gi M v N ln lt l trung im ca cc cnh AB v CD. Chng
minh rng MN
CDAB
2
.
www.MATHVN.com - HOANG NGOC QUANG, Yen Bai
43
Gii.
Hnh 1.37
Qua M v ng thng song song
vi AD ctDC ti E v qua M v
ng thng song song vi BC ct
DC ti F. Suy ra

D =

E
1
,

C =

F
1
.
Suy ra

E
1
+

F
1
=

D +

C 90
0

EMF 90
0
. Theo bi ton 1.3 tam
gic MEF c

EMF 90
0
v MN l
trung tuyn nn MN
EF
2
.
Mt khc, c AB//DC v AD//ME nn ADEM l hnh bnh hnh.
Suy ra DE = AM =
1
2
AB, tng t FC = MB =
1
2
AB. Do
EF = CD AB.
Vy MN
CDAB
2
.
Bi ton 1.40. Cho t gic ABCD, M l mt im thuc cnh CD
(M khc C, D). Chng minh rng
MA +MB < max {CA +CB; DA +DB} . (1.68)
Gii.
Hnh 1.38
Gi A

l im i xng ca A qua CD.


A

B ct CD P.
V M thuc on CD nn M thuc
A

BC hoc A

BD. Theo nh l 1.20


ta c
_
MA

+MB < CA

+CB
MA

+MB < DA

+DB

_
MA +MB < CA +CB
MA +MB < DA +DB
Do MA +MB < max {CA +CB; DA +DB}.
Ch 1.3. T bi ton 1.40 ta c cc kt qu sau:
1) Cho t gic ABCD, M l mt im thuc cnh CD (M c
th trng vi C hoc D). Ta c bt ng thc MA + MB
max {CA +CB; DA +DB}.
www.MATHVN.com - HOANG NGOC QUANG, Yen Bai
44
2) Cho ABCD l hnh ch nht v im M nm trn cnh CD. Ta c
bt ng thc MA +MB CA +CB.
3) Cho ABCD l hnh vung cnh a v im M nm trn cnh CD. Ta
c bt ng thc MA +MB (1 +

2)a.
ng thc trong cc bt ng thc trn xy ra khi v ch khi M C
hoc M D.
Bi ton 1.41. ( thi vo lp 10 chuyn ton-tin, HSP, HQG H
Ni 1998-1999) Cho hnh ch nht ABCD v im M nm trong hnh
ch nht v c th nm trn cc cnh ca ABCD. Chng minh rng
MA +MB +MC +MD AB +AC +AD.
Gii.
Hnh 1.39
Qua M v ng thng song song vi
AD ct AB, DE ln lt ti E, F. p
dng ch 1.3 ca bi ton 1.40 vo cc
hnh ch nht AEFD, EBCF v ABCD
ta c MA+MD EA+ED, MB+MC
EB + EC, ED + EC AD + AC. Do
MA+MB +MC +MD (EA+EB) +
(ED +EC) AB +AC +AD.
Ta c bi ton tng qut hn sau y
Bi ton 1.42. (Tuyn tp 5 nm tp ch THTT) Cho t gic ABCD,
M l mt im trong t gic. t d
A
= AB + AC + AD, d
B
= BC +
BD + BA, d
C
= CD + CA + CB, d
D
= DA + DB + DC. Chng minh
rng
MA +MB +MC +MD < max {d
A
; d
B
; d
C
; d
D
} .
Gii. Ko di AM mt on MB

bng MB. Qua M k ng trung


trc ca BB

. ng ny theo th t ct hai cnh t gic ti I, J. C


th xy ra mt trong ba trng hp hnh (A), (B), (C). V trong cc hnh
(B), (C) bi ton c chng minh tng t nhng n gin hn trong
trng hp (A) nn y ta ch chng minh trong trng hp (A). Khng
mt tnh tng qut gi s rng IC +ID = max {IC +ID, JC +JD} .
www.MATHVN.com - HOANG NGOC QUANG, Yen Bai
45
Hnh 1.40
p dng bi ton 1.40 cho t gic CIJD ta c MC+MD < IC+ID.
Li c MA+MB = MA+MB

= AB

< IA+IB

= IA+IB. Do
MA+MB +MC +MD < IA+IB +IC +ID = IA+ID +BC. p
dng bi ton 1.40 cho t gic ABCD ta c
IA +ID < max {CA +CD; BA +BD} .
Vy MA +MB +MC +MD < max {CA +CD; BA +BD} +BC =
=max {BC +BD +BA; CD +CA +CB} = max {d
A
; d
C
}
max {d
A
; d
B
; d
C
; d
D
} .
Ch 1.4. T bi ton 1.42 ta c cc kt qu sau:
1) Cho hnh ch nht ABCD c di cc cnh l a, b v di ng
cho l c. M l mt im nm bn trong hnh ch nht . Ta c bt
ng thc MA +MB +MC +MD < a +b +c.
2) Cho hnh vung ABCD cnh v M l mt im nm bn trong hnh
vung . Ta c bt ng thc MA +MB +MC +MD < (2 +

2)a.
1.7.2. Cc bt ng thc khc trong t gic
Bi ton 1.43. (IMO shorlist) Din tch ca mt t gic vi cc cnh
a, b, c v d l S. Chng minh rng
S
a +c
2
.
b +d
2
.
Gii. Trc tin, gi s t gic ABCD khng li. Khi mt trong cc
ng cho ca n, chng hn BD s khng c im chung vi phn
trong ca t gic.
www.MATHVN.com - HOANG NGOC QUANG, Yen Bai
46
Hnh 1.41
Ly i xng im C qua BD cho ta
mt t gic li ABC

D c cng cnh nhng


din tch ln hn din tch t gic ABCD.
Do khng mt tnh tng qut, ta c th
gi thit rng t gic ABCD li.
By gi ta chia t gic bi ng cho AC
thnh hai tam gic ABC v ADC. Ta c
[ABC]
ab
2
, [ADC]
dc
2
. Do
S = [ABC] + [ADC]
ab +cd
2
. (1.69)
Lm tng t vi ng cho BD, ta c
S = [BAD] + [BCD]
bc +da
2
. (1.70)
Cng theo v cc bt ng thc (1.69) v (1.70), ta c
2S
ab +bc +cd +da
2
=
(a +c) (b +d)
2
hay S
a +c
2
.
b +d
2
.
Bi ton 1.44. (Olympic Ty Ban Nha, 2000) Chng minh rng trong
tt c cc t gic li c din tch bng 1, th tng di cc cnh v
cc ng cho ln hn hoc bng 2
_
2 +

2
_
.
Gii. Gi a, b, c, d ln lt l di cc cnh v e, f ln lt l di
cc ng cho ca t gic. Ta s chng minh a + b + c + d 4 v
e +f 2

2.
Gi S l din tch t gic. Ta bit rng S =
1
2
ef sin , trong l
gc gia hai ng cho. V S = 1 nn ef 2. p dng bt ng thc
AM GM suy ra e +f 2

2. ng thc xy ra khi v ch khi e = f.


Mt khc, theo bi ton trn c S
a+c
2
.
b+d
2
. p dng AMGM v
s dng S = 1 ta suy ra a +b +c +d 4. ng thc xy ra khi v ch
khi a = b = c = d.
T suy ra bt ng thc cn chng minh. C hai ng thc xy
ra khi v ch khi t gic li l hnh vung.
www.MATHVN.com - HOANG NGOC QUANG, Yen Bai
47
Bi ton 1.45. (Olympic a trung hi, 1998) Cho ABCD l mt hnh
vung ni tip ng trn. M l mt im trn cung

AB. Chng minh


rng MC.MD 3

3MA.MB.
Gii.
Hnh 1.42
t =

ACM, =

BDM. Khi ta
c + =

4
v
MA.MB
MC.MD
= tan . tan .
By gi, rng
tan . tan . tan tan
3
_
+ +
3
_
,
y =

4
.
Suy ra tan . tan
1
3

3
.
Do MC.MD 3

3MA.MB.
Bi ton 1.46. (IMO, shorlist 1996) Cho ABCD l t gic li. K
hiu R
A
, R
B
, R
C
v R
D
l bn knh ng trn ngoi tip tam gic
DAB, ABC, BCD v CDA tng ng. Chng minh rng R
A
+ R
C
>
R
B
+R
D
nu v ch nu

A +

C >

B +

D.
Gii. Gi X = AC BD. Trong hai gc

AXB v

AXD c t nht mt
gc ln hn hay bng 90
0
. Ta gi s

AXB 90
0
. t =

CAB, =

ABD,

=

BDC,

DCA. Cc gc ny u nhn v + =

.
Hn na R
A
=
AD
2 sin
, R
B
=
BC
2 sin
, R
C
=
BC
2 sin

, R
D
=
AD
2 sin

.
By gi, ta xt 3 trng hp sau:
1. Nu

B +

D = 180
0
th ABCD l t gic ni tip ng trn v d
c R
A
+R
C
= R
B
+R
D
.
2. Nu

B +

D > 180
0
th D nm trong ng trn ngoi tip tam
gic ABC. iu ny dn n >

, <

, suy ra R
A
< R
D
v
R
C
< R
B
. Do R
A
+R
C
< R
B
+R
D
.
3. Nu

B +

D < 180
0
th D nm ngoi ng trn ngoi tip tam
gic ABC. iu ny dn n <

, >

, suy ra R
A
> R
D
v
R
C
> R
B
. Do R
A
+R
C
> R
B
+R
D
.
www.MATHVN.com - HOANG NGOC QUANG, Yen Bai
48
Chng 2
Bt ng thc Ptolemy v cc m
rng
Chng ny trnh by nh l Ptolemy, bt ng thc Ptolemy v cc
bi ton p dng. Ngoi ra cn trnh by mt s m rng ca bt ng
thc ny trong t gic v m rng trong t din. Ni dung ch yu c
hnh thnh t cc ti liu [1], [5], [8], [10] v [12].
2.1. nh l Ptolemy
nh l Ptolemy hay ng thc Ptolemy miu t quan h gia
di bn cnh v hai ng cho ca mt t gic ni tip ng trn.
nh l ny mang tn nh ton hc v thin vn hc ngi Hy Lp c
i Ptolemy.
nh l 2.1. Cho ABCD l t gic ni tip ng trn. Khi
AC.BD = AB.CD +AD.BC. (2.1)
Chng minh.
Hnh 2.1
Ly im M thuc ng cho BD sao
cho

MCD =

BCA. Khi , d thy cc
tam gic ABC v DMC ng dng nhau,
suy ra
CD
MD
=
CA
AB
CD.AB = CA.MD.
Cng d chng minh rng hai tam gic
BCM v ACD ng dng, do ta c
BC
BM
=
AC
AD
BC.AD = AC.BM. Cng
theo v hai bt ng thc trn, ta thu c
CD.AB +BC.AD = AC.BM +CA.MD
= AC.BD.
www.MATHVN.com - HOANG NGOC QUANG, Yen Bai
49
Bi ton 2.1. Cho tam gic ABC ni tip ng trn (O). ng phn
gic ca

BAC ct (O) ti D khc A. Gi K, L ln lt l hnh chiu ca
B, C trn AD. Chng minh rng AD BK +CL.
Gii.
Hnh 2.2
Do BKA, CLA l cc tam
gic vung nn ta c BK + CL =
(AB +AC) sin
A
2
. Mt khc, t gic
ABDC ni tip nn theo ng thc
ptolemy ta c AB.DC + AC.BD =
AD.BC m DC = BD =
BC
2 cos
D
2
=
BC
2 cos
A
2
nn AB +AC = 2AD. cos
A
2
.
Suy ra BK +CL = AD. sin A AD.
Bi ton 2.2. (Tp ch THTT s 261) Cho tam gic ABC. Cc ng
phn gic trong xut pht t A, B, C ct ng trn ngoi tip tam gic
ABC ti A
1
, B
1
, C
1
tng ng. Chng minh rng
AA
1
.BB
1
.CC
1
16R
2
r.
Gii.
Hnh 2.3
p dng nh l Ptolemy cho t gic
ABA
1
C ta c AA
1
.BC = AB.CA
1
+
AC.BA
1
hay AA
1
.a = c.CA
1
+b.BA
1
.
Do AA
1
l phn gic gc

BAC cho nn
A
1
l im chnh gia cung

BC v do
A
1
B = A
1
C. Suy ra a.AA
1
= (b + c)A
1
B.
Mt khc theo nh l hm s sin, ta c
A
1
B
sin

A
1
AB
= 2R, suy ra A
1
B = 2Rsin
A
2
.
Vy AA
1
=
b+c
a
.2Rsin
A
2
. Tng t BB
1
=
c+a
b
.2Rsin
B
2
, CC
1
=
a+b
c
.2Rsin
C
2
.
rng r = 4R. sin
A
2
. sin
B
2
. sin
C
2
v (a + b)(b + c)(c + a) 8abc. Suy
ra AA
1
.BB
1
.CC
1
=
8R
3
(a +b)(b +c)(c +a)
abc
sin
A
2
sin
B
2
sin
C
2
16R
2
r.
www.MATHVN.com - HOANG NGOC QUANG, Yen Bai
50
Bi ton 2.3. Cho tam gic ABC v im M bt k nm trn ng
trn (O) ngoi tip ABC. Chng minh rng
MA
a
+
MB
b
+
MC
c
min
_
a
b
+
b
a
;
b
c
+
c
b
;
c
a
+
a
c
_
2.
Gii.
Hnh 2.4
Hin nhin ta c bt ng thc
phi. Ta chng minh bt ng thc
tri bng phng php phn chng.
Gi s
MA
a
+
MB
b
+
MC
c
<
min
_
a
b
+
b
a
;
b
c
+
c
b
;
c
a
+
a
c
_
. Khng
gim tnh tng qut, gi s M nm
trn cung

BC khng cha A.
Do t gic ABMC ni tip, nn
p dng ng thc Ptolemy ta c
MA.BC = MC.AB + MB.AC. Suy
ra MA =
MB.b+MC.c
a
. Khi
MA
a
+
MB
b
+
MC
c
=
MB.
b
a
+MC.
c
a
a
+
MB.
a
b
+MC.
a
c
a
=
MB.
_
a
b
+
b
a
_
+MC
_
a
c
+
c
a
_
a

MB.
_
a
b
+
b
a
_
+MC
_
a
c
+
c
a
_
MB +MC
. (2.2)
Mt khc,
MA
a
+
MB
b
+
MC
c
< min
_
a
b
+
b
a
;
b
c
+
c
b
;
c
a
+
a
c
_
suy ra
MA
a
+
MB
b
+
MC
c
<
a
b
+
b
a
v
MA
a
+
MB
b
+
MC
c
<
a
c
+
c
a
.
T
MA
a
+
MB
b
+
MC
c
=
MB.
_
MA
a
+
MB
b
+
MC
c
_
+MC.
_
MA
a
+
MB
b
+
MC
c
_
MB +MC
<
MB.
_
a
b
+
b
a
_
+MC
_
a
c
+
c
a
_
MB +MC
. (2.3)
(2.2) v (2.3) mu thun, do gi s l sai. Vy c bt ng thc
cn chng minh.
Bi ton 2.4. (Ailen, 1995) Cho ba im A, X, D thng hng vi X
nm gia A v D. Gi B l im tha mn

ABX = 120
0
v C l im
www.MATHVN.com - HOANG NGOC QUANG, Yen Bai
51
nm gia B v X. Chng minh rng
2AD

3 (AB +BC +CD) .


Gii.
Hnh 2.5
Dng tam gic u AXO sao cho
B v O khc pha vi AX. D thy
ABXO l t gic ni tip ng trn
ng knh
2

3
AX. p dng nh l
Ptolemy cho t gic ny, ta c
AB.OX +BX.AO = AX.BO.
V OX = AO = AX nn
AX (AB +BX) = AX.BO AX.
2

3
AX. Suy ra 2AX

3 (AB +BX). Do 2AD = 2 (AX +XD)

3 (AB +BX)
+2XD

3 (AB +BC +CX) +

3XD

3 (AB +BC +CD) .


Bi ton 2.5. (V ch Ton ln th 2, Hng Kng, 1999) Gi I v O
ln lt l tm cc ng trn ni v ngoi tip tam gic ABC. Gi
s tam gic ABC khng u. Chng minh

AIO 90
0
nu v ch nu
2BC AB +CA.
Gii.
Hnh 2.6
Ko di AI ct ng trn ngoi tip
ABC ti D. V AD l phn gic ca

BAC nn DB = DC. Mt khc, ta c

DIB =

DAB +

ABI =

DCB +

IBC =

DBC +

IBC =

DBI Suy ra DB = DI =
DC. p dng ng thc Ptolemy cho t
gic ABCD, ta c AD.BC = AB.CD +
AC.BD = DI (AB +AC).
Do

AIO 90
0
AI ID 2
AD
DI
=
AB +AC
BC
2BC AB +AC.
www.MATHVN.com - HOANG NGOC QUANG, Yen Bai
52
Bi ton 2.6. (Chn i tuyn Singapore, 2002) Cho ABCD l t gic
ni tip ng trn. Chng minh rng
|AC BD| |AB CD| .
Gii.
Hnh 2.7
Gi E, F ln lt l trung im ca
AC, BD. p dng nh l cng thc ng
trung tuyn. Ta c
AB
2
+BC
2
+CD
2
+DA
2
=2BE
2
+
1
2
AC
2
+ 2DE
2
+
1
2
AC
2
=2
_
2EF
2
+
1
2
DB
2
_
+AC
2
.
Vy AC
2
+BD
2
+ 4EF
2
= AB
2
+BC
2
+
CD
2
+DA
2
. p dng nh l Ptolemy cho
t gic ABCD, ta c AC.BD = AB.CD +AD.BC. T suy ra
(AC BD)
2
+ 4EF
2
= (AB CD)
2
+ (AD BC)
2
. (2.4)
Mt khc, gi M l trung im ca AB, xt tam gic MEF, ta c
EF |MF ME| EF
1
2
|AD BC|. Do
4EF
2
(AD BC)
2
. (2.5)
T (2.4) v (2.5) thu c bt ng thc cn chng minh.
Bi ton 2.7. (IMO shorlist, 1996) Cho tam gic ABC v O l mt im
bt k trong tam gic. Gi (H), (I), (K) theo th t l cc ng trn
ngoi tip cc tam gic BOC, COA, AOB. Dng A

= AO (H), B

=
BO (I), C

= CO (K). Chng minh cc bt ng thc sau


a) OA

.OB

.OC

8.OA.OB.OC.
b)
OA

OA
+
OB

OB
+
OC

OC
6.
Gii.
www.MATHVN.com - HOANG NGOC QUANG, Yen Bai
53
Hnh 2.8
t
x = sin

BOC

= sin

COB

,
y = sin

COA

= sin

AOC

,
z = sin

AOB

= sin

BOA

.
Khi ta d dng chng minh c
CA

BC
=
y
x
,
BA

BC
=
z
x
. (2.6)
Mt khc, v t gic A

OBC ni tip
nn theo ng thc Ptolemy ta c
OA

=
OB.CA

+OC.BA

BC
. (2.7)
Cng theo v (2.6) v (2.7), sau p dng bt ng thc AG GM,
ta c
OA

=
y
x
.OB +
z
x
.OC

yz.OB.OC
x
. (2.8)
Tng t, ta c
OB

=
z
y
.OC +
x
y
.OA

zx.OC.OA
y
, (2.9)
OC

=
x
z
.OA +
y
z
.OB

xy.OA.OB
z
. (2.10)
Nhn theo v ba bt ng thc (2.8), (2.9) v (2.10) ta chng minh c
phn a.
Mt khc, ta c
OA

OA
+
OB

OB
+
OC

OC
=
_
y
x
.
OB
OA
+
z
x
.
OC
OA
_
+
_
z
y
.
OC
OB
+
x
y
OA
OB
_
+
_
x
z
.
OA
OC
+
y
z
OB
OC
_
=
_
y
x
.
OB
OA
+
x
y
.
OA
OB
_
+
_
z
y
.
OC
OB
+
y
z
OB
OC
_
+
_
x
z
.
OA
OC
+
z
x
OC
OA
_
6.
Phn b c chng minh.
2.2. Bt ng thc Ptolemy
nh l 2.2. (Bt ng thc Ptolemy) Vi 4 im A, B, C, D bt k
trn mt phng, ta c
AB.CD +AD.BC AC.BD. (2.11)
Chng minh.
www.MATHVN.com - HOANG NGOC QUANG, Yen Bai
54
Hnh 2.9
Dng im E sao cho tam gic
BCD ng dng vi tam gic BEA.
Khi theo tnh cht ca tam gic
ng dng, ta c
BA
EA
=
BD
CD
. Suy ra
BA.CD = EA.BD. (2.12)
Mt khc, hai tam gic EBC v ABD
cng ng dng do c
BA
BD
=
BE
BC
v

EBC =

ABD. T
EC
BC
=
AD
BD
.
Suy ra
AD.BC = EC.BD. (2.13)
Cng theo v (2.12) v (2.13) ta suy ra
AD.CD +AD.BC = BD. (EA +EC) BD.AC.
Du bng xy ra khi v ch khi A, E, C thng hng, tc l khi A v D
cng nhn BC di mt gc bng nhau, v khi t gic ABCD ni
tip trong mt ng trn.
Bt ng thc Ptolemy c nhiu ng dng trong cc bi ton bt
ng thc hnh hc, c bit l trong cc bi ton so snh di cc
on thng. Trc ht ta xt ng dng bt ng thc Ptolemy trong
vic chng minh mt s kt qu kinh in ca hnh hc phng.
Bi ton 2.8. (Bt ng thc Erdos - Modell) Cho tam gic ABC. M
l mt im bt k nm trong tam gic. t R
1
= MA, R
2
= MB, R
3
=
MC; r
1
, r
2
, r
3
ln lt l khong cch t M n BC, CA, AB. Khi ta
c bt ng thc
R
1
+R
2
+R
3
2 (r
1
+r
2
+r
3
) .
Gii. Ni di AM ct ng trn ni tip tam gic ABC ti A

. p
dng nh l Ptolemy cho t gic ni tip ABA

C, ta c
AB.CA

+AC.BA

= BC.AA

.
H A

D vung gc vi AC v A

E vung gc vi AB th r rng A

B
A

E, A

C A

D. Do a.AA

c.A

D+b.A

E hay 1
A

D
AA

.
c
a
+
A

E
AA

.
b
a
.
www.MATHVN.com - HOANG NGOC QUANG, Yen Bai
55
Hnh 2.10
Nhng
A

D
AA

=
r
2
R
1
v
A

E
AA

=
r
3
R
1
nn
t R
1
r
2
.
c
a
+ r
3
.
b
a
. Tng t ta
c cc nh gi cho R
2
v R
3
. Do
R
1
+R
2
+R
3
r
1
_
c
b
+
b
c
_
+r
2
_
c
a
+
a
c
_
+
r
3
_
b
a
+
a
b
_
2 (r
1
+r
2
+r
3
). Du bng
xy ra khi v ch khi tam gic ABC u v
M trng vi tm O ca tam gic.
Bi ton 2.9. (im Torricelli) Cho tam gic ABC. Hy tm im M
trong mt phng tam gic ABC sao cho MA + MB + MC t gi tr
nh nht. im M tm c c gi l im Torricelli ca tam gic
ABC.
Gii. Trn cnh BC, dng ra pha ngoi tam gic u BCA

. p dng
bt ng thc Ptolemy cho t gic MBA

C ta c
BM.CA

+CM.BA

BC.MA

.
Hnh 2.11
Do CA

= BA

= BC nn ta
c BM + CM MA

. Suy ra
AM+BM+CM MA+MA

AA

tc l MA + MB + MC AA

(l
hng s)
Du bng xy ra khi v ch khi
i) T gic BMCA

ni tip.
ii) M nm gia A v A

.
D thy ta c th tm c im M
tha mn c hai iu kin ny khi v
ch khi tt c cc gc ca tam gic ABC u khng ln hn 120
0
.
Trng hp tam gic ABC c mt gc ln hn 120
0
, chng hn

A > 120
0
th im M cn tm chnh l im A (xem li gii bi ton 1.23)
Nhn xt 2.1.
www.MATHVN.com - HOANG NGOC QUANG, Yen Bai
56
1) Phng php trn c th p dng cho bi ton tng qut hn: "Cho
tam gic ABC v cc s thc dng m, n, p. Hy tm im M trong mt
phng tam gic sao cho m.MA + n.MB + p.MC t gi tr nh nht"
t gi tr nh nht.
2) Vic dng tam gic u BCA

ra pha ngoi trong li gii bi ton


Torricenlli chnh l mt cch lm mu mc p dng c bt ng
thc Ptolemy. tng chung l: dnh gi tng p.MA+q.MB, ta c
th dng im N sao cho p.NA = q.NB. Sau p dng bt ng thc
ptolemy cho t gic ANMB ta c NA.MB + NB.MA AB.MN.
T
p.NA.MB +p.NB.MA p.AB.MN
q.NB.MB +p.NB.MA p.AB.MN
p.MA +q.MB p.AB.
MN
NB
.
Ch rng im N l c nh, nh th p.MA+q.MB c nh
gi thng qua MN. tng ny l cha kha gii hng lot cc bi
ton cc tr hnh hc.
Bi ton 2.10. Cho im M nm trong gc nhn xOy. Hai im A, B
ln lt thay i trn Ox, Oy sao cho 2OA = 3OB. Tm v tr ca A, B
sao cho 2MA + 3MB t gi tr nh nht.
Gii. p dng bt ng thc Ptolemy cho t gic OAMB, ta c
OA.MB +OB.MA OM.AB.
T 2OA.OB + 2OB.MA 2OM.AB 3OB.MB + 2OB.MA
2OM.AB 2MA+ 3MB 2OM.
_
AB
OB
_
. V tam gic OAB lun ng
dng vi chnh n nn
AB
OB
l mt i lng khng i. T suy ra
2MA + 3MB t gi tr nh nht bng 2OM.
_
AB
OB
_
. Du bng xy ra
khi v ch khi t gic OAMB ni tip.
Bi ton 2.11. (IMO, 2001) Cho tam gic ABC vi trng tm G v
di cc cnh a = BC, b = CA, c = AB. Tm im P trn mt phng
tam gic sao cho i lng AP.AG + BP.BG + CP.CG t gi tr nh
nht v tm gi tr nh nht theo a, b, c.
www.MATHVN.com - HOANG NGOC QUANG, Yen Bai
57
Gii. V ng trn ngoi tip tam gic BGC. Ko di trung tuyn AL
ct ng trn ny ti K. Gi M, N l trung im cc cnh AC, AB
tng ng. p dng nh l hm s sin cho cc tam gic BGL, CGL, ta
c
BG
sin

BLG
=
BL
sin

BGK
,
CG
sin

CLG
=
CL
sin

CGK
. (2.14)
Hnh 2.12
Nhng L l trung im ca BC
v sin

BLG = sin

CLG, nn t (2.14)
ta c
BG
CG
=
sin

CGK
sin

BGK
. Ta c BK =
2R. sin

BGK, CK = 2R. sin

CGK,
trong R l bn knh ng trn
ngoi tip tam gic BCG. Do
CK
BK
=
BG
CG
hay
BG
CK
=
CG
BK
. Tng t
AG
BG
=
sin

BGN
sin

AGN
=
sin

BGN
sin

CGK
. Hn na BC =
2R. sin

BGC = 2R. sin

BGN. T
BG
CK
=
AG
BC
. Nh vy
BG
CK
=
CG
BK
=
AG
BC
.
By gi, p dng bt ng bt ng
thc Ptolemy cho t gic PBKC ta c PK.BC BP.CK + CP.BK.
T PK.AG BP.BG+CP.CG. Suy ra (AP +PK) AG AP.AG+
BP.BG+CP.CG v cui cng AK.AG AP.AG+BP.BG+CP.CG.
Du bng xy ra khi v ch khi (1) P nm trn cung

BGC ( c ng
thc bt ng thc Ptolemy) v (2) P nm trn AK ( c ng thc
trong bt ng thc tam gic). Do gi tr ny t c khi P G.
D dng tnh c rng AG
2
+BG
2
+CG
2
=
_
a
2
+b
2
+c
2
_
3
.
Nhn xt 2.2. C th thy y l trng hp c bit ca bi ton
Torricelli tng qut. Ch rng t ba on AG, BG, CG c th dng
c mt tam gic , ta ch cn dng tam gic BCK ng dng vi
tam gic l c. Cch gii nu trn ch ra cch dng tng minh cho
im K.
Bi ton 2.12. Cho ABCD l hnh vung v P l mt im ty
trong mt phng. Trong s cc khong cch PA, PB, PCv PD, gi M
www.MATHVN.com - HOANG NGOC QUANG, Yen Bai
58
l gi tr ln nht v m l gi tr nh nht. Chng minh rng
PA +PB +PC +PD
_
1 +

2
_
M +m.
ng thc xy ra khi P nm trn ng trn ngoi tip hnh vung.
Gii. Khng gim tnh tng qut, gi s M = PD, th d dng thy
m = PB. p dng bt ng thc Ptolemy cho t gic PADC, ta c
PA.CD +PC.AD PD.AC
vi ng thc xy ra khi v ch khi A, P, C, D nm trn mt ng trn.
V AD = CD = AC/

2, nn PA +PC

2PD. Do
PA +PB +PC +PD
_
1 +

2
_
PD +PB =
_
1 +

2
_
M +m.
ng thc xy ra khi P nm trn ng trn ngoi tip hnh vung.
Bi ton 2.13. (Tp ch THTT s 259) Cho tht gic u
A
1
A
2
A
3
A
4
A
5
A
6
A
7
v im M bt k. Chng minh rng
MA
1
+MA
3
+MA
5
+MA
7
MA
2
+MA
4
+MA
6
.
Gii. t A
1
A
2
= a, A
1
A
3
= b, A
1
A
4
= c. p dng bt ng thc
Ptolemy cho t gic A
1
A
2
A
3
M v A
5
A
6
A
7
M, ta c
(MA
1
+MA
3
) a MA
2
.b, (2.15)
(MA
5
+MA
7
) a MA
6
.b. (2.16)
Cng theo v 2 bt ng thc trn c
(MA
1
+MA
3
+MA
5
+MA
7
) a (MA
2
+MA
6
) .b. (2.17)
Tip tc p dng bt ng thc Ptolemy cho t gic A
2
A
4
A
6
M, c
(MA
2
+MA
6
) b MA
4
.c. (2.18)
T (2.17) v (2.18) c
(MA
1
+MA
3
+MA
5
+MA
7
) a MA
4
.c. (2.19)
www.MATHVN.com - HOANG NGOC QUANG, Yen Bai
59
T (2.17) v (2.19) c
(MA
1
+MA
3
+MA
5
+MA
7
)
_
a
b
+
a
c
_
MA
2
+MA
4
+MA
6
. (2.20)
p dng nh l Ptolemy cho t gic ni tip A
1
A
3
A
4
A
5
c ab+ac = bc,
suy ra
_
a
b
+
a
c
_
= 1. Thay vo (2.20) c
MA
1
+MA
3
+MA
5
+MA
7
MA
2
+MA
4
+MA
6
.
ng thc xy ra khi cc ng thc (2.15), (2.16) v (2.18) xy ra
ngha l M thuc cung nh

A
1
A
7
ca ng trn ngoi tip a gic
A
1
A
2
A
3
A
4
A
5
A
6
A
7
.
Bi ton 2.14. (Olympic Ton hc 30/4, bi ngh, 2000) Chng t
rng trong tam gic ABC ta c
ab
m
a
m
b
+
bc
m
b
m
c
+
ca
m
c
m
a
4,
vi a, b, c l di ba cnh v m
a
, m
b
, m
c
l di cc ng trung
tuyn tng ng ca tam gic .
Gii.
Hnh 2.13
iu phi chng minh tng
ng vi
abm
c
+bcm
a
+cam
b
4m
a
m
b
m
c
.
p dng bt ng thc Ptolemy cho
t gic CMGN, ta c
GC.MN GN.MC +GM.NC,
t
2
3
m
c
c
2

1
3
m
b
a
2
+
1
3
m
a
b
2
hay 2m
c
c m
b
a +m
a
b. Suy ra
2m
c
c
2
acm
b
+bcm
a
. (2.21)
Mt khc, p dng bt ng thc Ptolemy cho t gic ABMN ta c
AM.BN AN.BM +AB.MN, suy ra m
a
m
b

ab
4
+
c
2
2
, tc l
4m
a
m
b
m
c
abm
c
+ 2c
2
m
c
. (2.22)
T (2.21) v (2.22) suy ra iu phi chng minh.
www.MATHVN.com - HOANG NGOC QUANG, Yen Bai
60
Bi ton 2.15. (Trung Quc, 1988) Cho t gic ABCD ni tip ng
trn (O, R). Cc tia AB, BC, CD, DA ct ng trn (O, 2R) ln lt
ti cc im A

, B

, C

, D

tng ng. Chng minh rng


A

+B

+C

+D

2 (AB +BC +CD +DA) .


Khi no du ng thc xy ra ?
Gii. p dng bt ng thc Ptolemy cho t gic OAD

, ta c
OD

.AA

OA.A

+ OA

.AD

. Suy ra 2AA

+ 2AD

hay
2AB + 2BA

+ 2AD

. Tng t vi ba bt ng thc khc, ta


i n
2 (AB +BC +CA +AD) + 2 (BA

+CB

+DC

+AD

)
(A

+B

+C

+D

) + 2 (BA

+CB

+DC

+AD

) .
T suy ra bt ng thc cn chng minh.
Hnh 2.14
ng thc xy ra khi tt c cc t gic
OAD

, OBA

, OCB

, ODC

u
ni tip. Nu OAD

ni tip th

OAA

OD

=

OA

= 180
0


OAD

=

OAD.
Do OA l phn gic ca

DAB. Tng
t, im O nm trn cc phn gic khc
ca ABCD. Suy ra

OAB =

OAD =

ODA =

ODC =

OBA. T ,

AOB =

BOC =

COD =

DOA. Vy A, B, C, D
cch u nhau trn ng trn, ngha l
ABCD l hnh vung. Ngc li, nu ABCD l mt hnh vung th
hin nhin ng thc xy ra.
Tm li, ng thc xy ra khi v ch khi ABCD l mt hnh vung.
Bi ton 2.16. (IMO shorlist, 1997) Cho lc gic li ABCDEF c
AB = BC, CD = DE, EF = FA. Chng minh rng
BC
BE
+
DE
DA
+
FA
FC

3
2
.
Khi no du ng thc xy ra ?
www.MATHVN.com - HOANG NGOC QUANG, Yen Bai
61
Gii. p dng bt ng thc Ptolemy cho t gic ABCE, ta
c EA.BC + EC.AB BE.AC. S dng AB = BC, ta c
BC (EA +EC) BE.AC, hay
BC
BE

AC
EA+EC
.
Tng t, p dng bt ng thc Ptolemy cho cc t gic
ACDE, EFAC v s dng CD = DE, EF = FA, ta c
DE
DA

CE
AC+AE
,
FA
FC

EA
CE+CA
. Cng theo v ba bt ng thc trn v p dng bt ng
thc Nesbitt ta thu c bt ng thc cn chng minh.
c du bng ta phi c du bng xy ra ba bt ng thc
Ptolemy v bt ng thc Nesbitt. Du bng xy ra bt ng thc
Nesbitt khi tam gic EAC u, nh th

EAC = 60
0
. V ABCE l t
gic ni tip nn gc

B phi bng 120
0
. Tng t, gc

D v

F cng
bng 120
0
. Mt khc, thy rng cc tam gic ABC, CDE, EFA bng
nhau (g.c.g). Nh vy, lc gic c tt c cc cnh u bng nhau v tt
c cc gc bng 120
0
, vy n l lc gic u. Ngc li, hin nhin l vi
lc gic u, ta c du bng xy ra.
Bi ton 2.17. (IMO, 1995) Cho ABCDEF l lc gic sao cho AB =
BC = CD, DE = EF = FA v

BCD =

EFA = 60
0
. G v H l hai
im ty . Chng minh rng
AG+BG+GH +DH +EH CF.
Gii.
Hnh 2.15
T gi thit ta c BCD v
AEF l cc tam gic u. Ly C

v F

ln lt i xng vi C v
F qua BE. Khi CF = C

v ta c DEF

v ABC

l cc
tam gic u. p dng bt ng
thc Ptolemy cho t gic F

DHE
v C

AGB, ta c
F

H.DE F

E.DH +F

D.EH
hay F

H DH +EH
v C

G.AB C

A.BG+C

B.AG
www.MATHVN.com - HOANG NGOC QUANG, Yen Bai
62
hay C

G BG+AG. Do
AG+BG+GH +DH +EH C

G+GH +HF

= CF.
Bi ton 2.18. (New Zealand, 1998) Cho M, A
1
, A
2
, , A
n
(n 3) l
cc im phn bit trong mt phng tha mn A
1
A
2
= A
2
A
3
= =
A
n1
A
n
= A
n
A
1
. Chng minh rng
1
MA
1
.MA
2
+
1
MA
2
.MA
3
+
1
MA
n1
.MA
n

1
MA
1
.MA
n
.
Xc nh du ng thc xy ra khi no?
Gii. Vi mi s nguyn k gia 1 v n 1 , p dng bt ng thc
Ptolemy cho t gic MA
1
A
k
A
k+1
ta c
MA
1
.A
k
A
k+1
+A
1
A
k
.MA
k+1
A
1
A
k+1
.MA
k
, (2.23)
(vi k = 1 suy bin thnh ng thc). Chia c hai v bt ng thc trn
cho MA
1
.MA
k
.MA
k+1
ta c
A
k
A
k+1
MA
k
.MA
k+1

A
1
A
k+1
MA
1
.MA
k+1

A
1
A
k
MA
1
.MA
k
.
Ly tng n 1 bt ng thc ny, ta c
A
1
A
2
MA
1
.MA
2
+
A
2
A
3
MA
2
.MA
3
+ +
A
n1
A
n
MA
n1
.MA
n

A
1
A
n
MA
1
.MA
n

A
1
A
1
MA
1
.MA
1
=
A
1
A
n
MA
1
.MA
n
.
V A
1
A
2
= A
2
A
3
= = A
n1
A
n
= A
n
A
1
nn ta c bt ng thc
cn chng minh.
ng thc xy ra khi v ch khi trong (2.23) du ng thc xy ra vi
tt c cc gi tr k = 1, 2, , n1. iu ny xy ra khi v ch khi theo
th t A
1
, A
k
, A
k+1
, M nm trn mt ng trn. Trong trng hp ny
A
1
A
2
, A
n
l mt a gic u v M thuc cung

A
k
A
n
ngn nht.
www.MATHVN.com - HOANG NGOC QUANG, Yen Bai
63
2.3. nh l Bretschneider
nh l 2.3. Cho t gic ABCD, gi a, b, c, d ln lt l di cc
cnh AB, BC, CD.DA v m, n l di cc ng cho AC, BD. Khi
ta c
m
2
n
2
= a
2
c
2
+b
2
d
2
2abcd. cos(A +C). (2.24)
Chng minh.
Hnh 2.16
Trn cnh AB ra pha ngoi dng tam
gic BAK ng dng vi tam gic ACD,
trong

BAK =

DCA,

ABK =

CAD,
cn trn cnh AD ra pha ngoi dng tam
gic DMA ng dng vi tam gic ABC,
trong

DAM =

BCA,

ADM =

CAB.
T cc tam gic ng dng ny suy ra
AK =
ac
m
, AM =
bd
m
, KB = DM =
ad
m
.
Ngoi ra,

KBD+

MDB =

CAD+

ABD+

BDA+

CAB = 180
0
, ngha
l t gic KBDM l hnh bnh hnh. Do KM = BD = n.
Mt khc

KAM =

A +

C. p dng nh l hm s cosin cho tam gic
KAM, ta c m
2
n
2
= a
2
c
2
+b
2
d
2
2abcd. cos(A +C).
Ch 2.1. V 0 < < 2 nn 1 cos < 1, do t kt qu
trn suy ra |ac bd| < mn ac + bd. Du "=" xy ra khi v ch khi
cos = 1 = ABCD l t gic ni tip.
Nh vy nh l Ptolemy v c bt ng thc Ptolemy u l h qu
ca nh l Bretschneider.
2.4. nh l Casey
nh l 2.4. Cho t gic ABCD ni tip ng trn (O). Bn ng
trn , , , tip xc vi (O) ln lt ti A, B, C, D. Gi a l di
on tip tuyn chung ca hai ng trn , , vi , cng tip xc
ngoi hoc cng tip xc trong vi (O). Nu hai ng trn , c mt
www.MATHVN.com - HOANG NGOC QUANG, Yen Bai
64
ng trn tip xc ngoi (O) v mt ng trn tip xc trong vi (O)
th a l di tip tuyn chung trong ca , . Cc i lng b, c, d, x, y
l di cc tip tuyn chung ca cc cp ng trn xc nh tng
t (hnh 2.17). Khi ta c
xy = ac +bd. (2.25)
Chng minh.
Hnh 2.17
Ta chng minh cho trng hp
, , , u tip xc ngoi vi (C).
Cc trng hp cn li c chng
minh tng t.
Gi R
a
, R
b
, R
c
, R
d
, R ln lt l bn
knh ca cc ng trn , , , , (O).
t

A

OB

=

AOB = , ta c a
2
=
A

B
2
(R
a
R
b
)
2
; A

B
2
= (R+R
a
)
2
+
(R +R
b
)
2
2(R +R
a
)(R +R
b
) cos .
Suy ra
a
2
= 2(R +R
a
)(R +R
b
)(1 cos ), (2.26)
AB
2
= 2R
2
2R
2
cos = 2R
2
(1 cos ). (2.27)
T (2.26) v (2.27) suy ra a =
_
(R +R
a
)(R +R
b
)
R
AB. Chng minh
tng t ta c
b =
_
(R +R
b
)(R +R
c
)
R
BC; c =
_
(R +R
c
)(R +R
d
)
R
CD;
d =
_
(R +R
d
)(R +R
a
)
R
DA; x =
_
(R +R
a
)(R +R
c
)
R
AC;
y =
_
(R +R
b
)(R +R
d
)
R
BD.
p dng nh l Ptolemy cho t gic ABCD ta thu c (2.25)
Nhn xt 2.3. ng thc (2.25) vn ng khi cc bn knh R
a
=
0, R
b
= 0, R
c
= 0, R
d
= 0; Khi R
a
= R
b
= R
c
= R
d
= 0 th nh l
www.MATHVN.com - HOANG NGOC QUANG, Yen Bai
65
Casey tr thnh nh l Ptolemy. Phi chng nh l Casey c suy ra
bng cch "n" cc im A, B, C, D.
nh l Casey l mt m rng ca nh l Ptolemy, n s gip ch cho
vic gii mt s bi ton lin quan n ng trn tip xc nhau,
di tip tuyn v dy cung. Sau y l mt s bi ton p dng nh l
Casey.
Bi ton 2.19. Cho hai ng trn (O
1
) v (O
2
) tip xc ngoi nhau
ti I v cng tip xc trong vi ng trn O. Mt tip tuyn chung
ngoi ca O
1
v (O
2
) ct O ti B v C, trong khi tip tuyn chung
trong ca chng ct O ti im A cng pha vi I. Chng minh rng I
l tm ng trn ni tip tam gic ABC.
Gii.
Hnh 2.18
Gi s BC tip xc (O
1
) ti X v
(O
2
) ti Y v ct AI ct BC ti D.
t BC = a, CA = b, AB = c, BX =
x, CY = y, AI = z, DX = DI =
DY = u. p dng nh l Casey cho
b 4 ng trn (A), (O
1
), (B), (C) v
(A), (O
2
), (C), (B) ta c
az +bx = c (2u +y) , (2.28)
az +cy = b (2u +x) . (2.29)
Ly (2.28)tr (2.29) theo v, ta c
bx cy = u(c b). T
x +u
y +u
=
c
b
,
tc l
BD
CD
=
AB
AC
. Suy ra AD l phn gic ca gc A v BD =
ac
b +c
.
Mt khc, cng hai v (2.28) v (2.29) theo v, ta c az = u(b +c).
Suy ra
z
u
=
b +c
a
, tc l
AI
ID
=
AB +AC
BD +DC
=
AB
BD
, do BI l phn
gic ca gc B. Bi ton c chng minh.
Bi ton 2.20. Cho tam gic ABC ni tip ng trn (O, R) v

BAC = 60
0
. ng trn (O

, R

) tip xc trong vi (O, R) v tip xc


vi hai cnh AB, AC. Chng minh rng R


2
3
R.
www.MATHVN.com - HOANG NGOC QUANG, Yen Bai
66
Gii. Gi X, Y ln lt l tip im ca AB, AC vi ng trn (O

, R

).
t l = AX = AY = XY (ABC cn c

BAC = 60
0
). ng trn
(O, R) tip xc trong vi cc ng trn (A), (C), (O

), (B) nn p
dng nh l Casey ta c b(c l) +c(b l) = al.
Hnh 2.19
Suy ra
l =
2bc
a +b +c
=
bc
p
=
bc
S
r =
2bc
bc sin A
r
=
4

3
r
2

3
R (vR 2r). (2.30)
Mt khc, li c
R

= O

X =
XY
2 sin 60
0
=
l

3
(2.31)
T (2.30) v (2.31) suy ra R


2
3
R.
Bi ton 2.21. Cho ba ng trn (C), (C
1
), (C
2
) trong hai ng
trn (C
1
), (C
2
) tip xc trong vi (C) ln lt ti B, C v (C
1
), (C
2
) tip
xc ngoi vi nhau ti D. Tip tuyn chung trong ca (C
1
), (C
2
) ct (C)
ti A, A
1
, AB ct (C
1
) ti im th hai l K, AC ct (C
2
) ti im th
hai l L. Chng minh rng
1
DA
+
1
DA
1
=
2
KL
.
Gii. t M = (C
1
) BC, N = (C
2
) BC. V tia tip tuyn BT vi
hai ng trn (C
1
), (C). Ta chng minh KL l tip tuyn chung ca
hai ng trn (C
1
), (C
2
).
Hnh 2.20
Tht vy, ta c

MKB =

MBT =

CBT =

CAB. Suy ra MK//AC. Do

MKL =

KLA. Ta c KAL CAB
(v AK.AB = AL.AC = AD
2

AK
AC
=
AL
AB
) suy ra

KLA =

CBA, do KL l
tip tuyn ca (C
1
). Tng t KL l tip
tuyn ca (C
2
)
p dng nh l Casey cho b 4 ng
trn (A), (C
1
), (D), (C
2
) tip xc trong vi
www.MATHVN.com - HOANG NGOC QUANG, Yen Bai
67
ng trn (C), ta c AD.A
1
D + AD.A
1
D = AA
1
.KL
2
KL
=
AA
1
AD.A
1
D
=
1
AD
+
1
A
1
D
.
Bi ton 2.22. Cho hai ng trn (C
1
), (C
2
) tip xc trong vi ng
trn (C) ln lt ti M, N. Hai ng trn (C
1
), (C
2
) ct nhau hoc
tip xc ngoi vi nhau. Trc ng phng ca (C
1
), (C
2
) ct (C) ti
hai im A, B. ng thng AM, AN ct (C
1
), (C
2
) ti im th hai l
K, L tng ng. Chng minh rng AB 2KL. ng thc xy ra khi
no ?
Gii.
Hnh 2.21 Hnh 2.22
t C
1
(O
1
, R
1
), C
2
(O
2
, R
2
), C(O, R), {P, Q} = (C
1
)(C
2
) (hnh 2.21),
nu (C
1
) tip xc (C
2
) th P Q (hnh 2.22).
AB l trc ng phng ca (C
1
), (C
2
) suy ra AK.AM = AL.AN
AKL ANM

AKL =

ANM =
1
2
s

AM.
Ta c

O
1
KM =

O
1
MK = 90
0

1
2

MOA = 90
0

1
2
s

AM

O
1
KL = 90
0
, suy ra KL l tip tuyn ca (C
1
), ta c O
1
K//OA//O
2
L
KLO
2
L, suy ra KL l tip tuyn ca (C
2
).
p dng nh l Casey cho b 4 ng trn ng trn
(A), (C
1
), (B), (C
2
) tip xc vi ng trn (C), ta c

AP.AQ.

BP.BQ +

AP.AQ.

BP.BQ = AB.KL suy ra


www.MATHVN.com - HOANG NGOC QUANG, Yen Bai
68
AB.KL = 2

AP.AQ.

BP.BQ 2
AP +BP
2
AQ+BQ
2
=
AB
2
2
.
Do AB 2KL.
Du ng thc xy ra khi v ch khi (C
1
) tip xc (C
2
) v AP = BP.
2.5. M rng bt ng thc Ptolemy trong khng gian
nh l 2.5. Cho ABCD l t din bt k, ta c
AC.BD +AD.BC > AB.CD. (2.32)
Chng minh.
Hnh 2.23
Trong mt phng (BCD) ta
ly im E sao cho B v E
khc pha vi ng thng CD
v AC = CE, AD = DE. T
ta c ACD = ECD. Suy ra
AP = PE, trong P l giao
im ca BE v CD. p dng
bt ng thc Ptolemy cho t
gic BCED, ta c BE.CD
CE.BD +BC.DE.
Mt khc, AB.CD
(AP +PB) CD = PE.CD +
PB.CD = BE.CD.
Vy AB.CD BE.CD CE.BD + BC.DE = AC.BD + AD.BC.
Ngoi ra du ng thc khng xy ra.
Bi ton 2.23. (Olympic 30/4, Vit Nam 2000) Cho hnh chp tam
gic S.ABC. Gi s cc trung tuyn ca cc tam gic SAB, SBC, SCA
k t S to vi nhng cnh y AB, BC, CA cc gc khng t bng
nhau. Chng minh din tch mt mt bn ca hnh chp nh hn tng
din tch cc mt bn cn li.
www.MATHVN.com - HOANG NGOC QUANG, Yen Bai
69
Gii. Gi l gc to bi cc trung tuyn SM, SK, SL vi cc cnh
y AB, BC, CA. V vai tr ca cc mt bn l nh nhau nn ta ch
cn chng minh [SAB] < [SBC] + [SCA]. Tht vy, p dng nh l
2.5 cho t din SKLM ta c SM.KL < SK.LM + SL.KM. Bt ng
thc trn tng ng vi cc bt ng thc sau SM.AB < SK.BC +
SL.CA SM.AB. sin < SK.BC. sin + SL.CA. sin [SAB] <
[SBC] + [SCA] .
Bi ton 2.24. (Tp ch THTT, s 264) Trn cnh CD ca hnh t din
ABCD ly im N (N khc C, D). K hiu p(XY Z) l chu vi tam gic
XY Z. Chng minh rng NC.p(DAB) +ND.p(CAB) > CD.p(NAB).
Gii. Xt bt ng thc Ptolemy cho cc b 4 im (N, A, C, D) v
(N, C, B, D) ta c
NC.DA +ND.CA > CD.NA, (2.33)
NC.DB +ND.CB > CD.NB. (2.34)
Mt khc, v N thuc on CD nn NC +ND = CD. Do
NC.AB +ND.AB = CD.AB. (2.35)
Cng theo v 3 bt ng thc (2.33),(2.34) v (2.35) ta c bt ng
thc cn chng minh.
www.MATHVN.com - HOANG NGOC QUANG, Yen Bai
70
Chng 3
Bt ng thc Erdos-Mordell v
cc m rng
Bt ng thc Erdos-Mordell l mt bt ng thc ni ting trong
tam gic, c nh ton hc Paul Erdos xut nm 1935 v li gii
u tin a ra l ca Louis Mordell s dng nh l hm s cosin.
Chng ny trnh by bt ng thc Erdos-Mordel v cc bi ton
p dng. Ngoi ra cn trnh by mt s m rng ca bt ng thc ny
trong tam gic v m rng trong a gic. Ni dung ch yu c hnh
thnh t cc ti liu [11 -13].
Ngoi cc k hiu s dng chng 1, ta s dng thm cc k hiu
sau: Cho tam gic ABC v P l mt im nm trong tam gic. K hiu
R
1
, R
2
, R
3
ln lt l khong cch t P n cc nh A, B, C v r
1
, r
2
, r
3
ln lt l khong cch t P n cc cnh BC, CA, AB.
3.1. Bt ng thc Erdos-Mordell trong tam gic
nh l 3.1. (Bt ng thc Erdos-Mordell) Cho tam gic ABC v P
l im nm trong tam gic. Khi lun c bt ng thc
R
1
+R
2
+R
3
2 (r
1
+r
2
+r
3
) . (3.1)
ng thc xy ra khi v ch khi tam gic ABC u v P l trc tm
ca n.
Bt ng thc Erdos-Mordell c rt nhiu cch chng minh khc
nhau. Ngoi chng minh bng cch s dng nh l Ptolemy, ta trnh
by hai cch chng minh n gin sau y:
Chng minh 1. K tia Ax i xng vi tia AP qua phn gic ca gc
A. T B v C k cc ng vung gc BB

v CC

ti tia Ax. Ta c

PAC =

BAB

,

PAB =

CAC

. Do a BB

+CC

= AB. sin

BAB

+
AC. sin

CAC

= c. sin

PAC +b. sin

PAB = c.
r
2
R
1
+b.
r
3
R
1
.
www.MATHVN.com - HOANG NGOC QUANG, Yen Bai
71
Hnh 3.1
Suy ra
R
1
r
2
c
a
+r
3
b
a
, (3.2)
tng t, ta c
R
2
r
3
a
b
+r
1
c
b
, (3.3)
R
3
r
1
b
c
+r
2
a
c
. (3.4)
Cng theo v cc bt ng thc
(3.2), (3.3), (3.4) v p dng bt
ng thc AM GM ta c
R
1
+R
2
+R
3
r
1
_
c
b
+
b
c
_
+r
2
_
a
c
+
c
a
_
+r
3
_
a
b
+
b
a
_
2 (r
1
+r
2
+r
3
) .
R rng du ng thc xy ra khi v ch khi a = b = c v trong (3.2),
(3.3), (3.4) du ng thc xy ra. iu ny xy ra khi v ch khi tam
gic ABC u v P l trc tm ca n.
Chng minh 2. y l chng minh u tin c a ra bi Mordell.
Hnh 3.2
p dng nh l hm s sin v hm
s cosin, ta c
R
1
. sin A = EF =
_
r
2
2
+r
2
3
2r
2
r
3
cos ( A),
R
2
. sin B = FD =
_
r
2
3
+r
2
1
2r
3
r
1
cos ( B),
R
3
. sin C = DE =
_
r
2
1
+r
2
2
2r
1
r
2
cos ( C).
Mt khc, ta c
EF
2
=r
2
2
+r
2
3
2r
2
r
3
cos ( A)
=r
2
2
+r
2
3
2r
2
r
3
(cos Bcos C sin Bsin C) .
=(r
2
sin C +r
3
sin B)
2
+ (r
2
cos C r
3
cos B)
2
(r
2
sin C +r
3
sin B)
2
.
Suy ra EF r
2
sin C +r
3
sin B. Do R
1
r
2
sin C
sin A
+r
3
sin B
sin A
.
Tng t R
2
r
3
sin A
sin B
+r
1
sin C
sin B
, R
3
r
1
sin B
sin C
+r
2
sin A
sin C
.
www.MATHVN.com - HOANG NGOC QUANG, Yen Bai
72
Cng theo v 3 bt ng thc trn, ri p dng bt ng thc AMGM
ta c
R
1
+R
2
+R
3
=r
1
_
sin B
sin C
+
sin C
sin B
_
+r
2
_
sin A
sin C
+
sin C
sin A
_
+
+r
3
_
sin A
sin B
+
sin B
sin A
_
2 (r
1
+r
2
+r
3
) .
Du ng thc xy ra khi v ch khi sin A = sin B = sin C v r
1
= r
2
=
r
3
, iu ny c ngha tam gic ABC u v P l trc tm ca n.
Bi ton 3.1. Cho tam gic ABC v P l mt im ty trong tam
gic. Chng minh rng
2
_
1
R
1
+
1
R
2
+
1
R
3
_

1
r
1
+
1
r
2
+
1
r
3
. (3.5)
Gii.
Hnh 3.3
Xt php nghch o N cc P v
phng tch d = r
2
N :A A

; B B

; C C

,
A
1
A

1
; B
1
B

1
; C
1
C

1
.
Khi , ta c
PA.PA

= PB.PB

= PC.PC

= d
2
,
PA
1
.PA

1
= PB
1
PB

1
= PC
1
.PC

1
= d
2
.
Hn na A

, B

, C

nm trn
B

1
C

1
, C

1
A

1
, A

1
B

1
tng ng v
PA

, PB

, PC

vung gc vi B

1
C

1
, C

1
A

1
, A

1
B

1
tng ng.
p dng bt ng thc Erdos-Mordell cho tam gic A

1
B

1
C

1
ta c
PA

1
+PB

1
+PC

1
2 (PA

+PB

+PC

).
V PA

1
=
d
2
PA
1
, PB

1
=
d
2
PB
1
, PC

1
=
d
2
PC
1
, PC

=
d
2
PC
, PB

=
d
2
PB
,
PA

=
d
2
PA
nn d
2
_
1
PA
1
+
1
PB
1
+
1
PC
1
_
2d
2
_
1
PA
+
1
PB
+
1
PC
_
.
Do 2
_
1
R
1
+
1
R
2
+
1
R
3
_

1
r
1
+
1
r
2
+
1
r
3
.
www.MATHVN.com - HOANG NGOC QUANG, Yen Bai
73
Bi ton 3.2. Cho tam gic ABC v P l mt im ty trong tam
gic. Chng minh rng
R
1
r
1
+R
2
r
2
+R
3
r
3
2 (r
1
r
2
+r
2
r
3
+r
3
r
1
) . (3.6)
Gii.
Hnh 3.4
Gi h
a
l di ng cao xut pht t
A, ta c ah
a
= 2[ABC] = ar
1
+ br
2
+ cr
3
.
V h
a
R
1
+r
1
nn a(R
1
+r
1
) ah
a
, suy
ra aR
1
br
2
+ cr
3
hay R
1
r
2
b
a
+ r
3
c
a
.
Do R
1
r
1
r
1
r
2
b
a
+ r
3
r
1
c
a
. Tng t
R
2
r
2
r
2
r
3
c
b
+r
1
r
2
a
b
, R
3
r
3
r
3
r
1
a
c
+r
2
r
3
b
c
.
Cng theo v 3 bt ng thc trn v p
dng bt ng thc AG-GM, ta c
R
1
r
1
+R
2
r
2
+R
3
r
3

_
a
b
+
b
a
_
r
1
r
2
+
_
b
c
+
c
b
_
r
2
r
3
+
_
c
a
+
a
c
_
r
3
r
1
2 (r
1
r
2
+r
2
r
3
+r
3
r
1
) .
Bi ton 3.3. Cho tam gic ABC v P l mt im ty trong tam
gic. Chng minh rng
R
1
R
2
+R
2
R
3
+R
3
R
1
4 (r
1
r
2
+r
2
r
3
+r
3
r
1
) . (3.7)
Gii. Xt php nghch o N cc P v phng tch d = r
2
N : A A

; B B

; C C

.
Gi r

1
, r

2
, r

3
l khong cch t P ti cc cnh B

, C

, A

, tng
ng. Ta c r

1
.B

= 2 [PB

] =
PB

.PC

.B

PA

1
=
r
1
.PB

.PC

.B

d
2
,
hay r

1
=
r
1
.PB

.PC

d
2
, trong A

1
l nh ca A
1
(A
1
l hnh chiu ca
P trn BC). Tng t r

2
=
r
2
.PC

.PA

d
2
, r

3
=
r
3
.PA

.PB

d
2
.
p dng bt ng thc Erdos-Mordell cho tam gic A

, ta c
PA

+PB

+PC

2 (r

1
+r

2
+r

3
). V PA.PA

= PB.PB

= PC.PC

=
www.MATHVN.com - HOANG NGOC QUANG, Yen Bai
74
d
2
nn
1
PA
+
1
PB
+
1
PC
2
_
r
1
PB.PC
+
r
2
PC.PA
+
r
3
PA.PB
_
Bt ng
thc trn tng ng vi
R
2
R
3
+R
3
R
1
+R
1
R
2
2 (R
1
r
1
+R
2
r
2
+R
3
r
3
) . (3.8)
T bt ng thc (3.6) v (3.8) ta thu c
R
1
R
2
+R
2
R
3
+R
3
R
1
4 (r
1
r
2
+r
2
r
3
+r
3
r
1
) .
Bi ton 3.4. Cho tam gic ABC v P l mt im ty trong tam
gic. Chng minh rng
R
1
R
2
R
3
8r
1
r
2
r
3
. (3.9)
Gii. Gi C
1
l im trn BC sao cho BC
1
= AB. Khi AC
1
= 2c sin
B
2
.
Mt khc, gi K, H ln lt l chn ng vung gc h t A, C
1
ln
Hnh 3.5
ng thng BP, v A v
C
1
khng nm cng pha i
vi ng thng BP nn
AC
1
C
1
H + AK, suy ra
R
2
.AC
1
R
2
.C
1
H + R
2
.AK hay
R
2
.AC
1
2[BPC
1
] + 2[BAP].
Do R
2
.2c sin
B
2
c.r
1
+ c.r
3
,
hay R
2

r
1
+r
3
2 sin
B
2
, tng t
R
1

r
2
+r
3
2 sin
A
2
, R
3

r
1
+r
2
2 sin
C
2
. Nhn 3 bt ng thc trn, ta c
R
1
R
2
R
3

1
8 sin
A
2
. sin
B
2
. sin
C
2
(r
1
+r
2
) (r
2
+r
3
) (r
3
+r
1
) . (3.10)
V sin
A
2
. sin
B
2
. sin
C
2
=
r
4R
v R 2r nn t (3.10), ta c
R
1
R
2
R
3
(r
1
+r
2
) (r
2
+r
3
) (r
3
+r
1
) . (3.11)
p dng bt ng thc AM GM cho v phi ca (3.11), ta c
R
1
R
2
R
3
8r
1
r
2
r
3
.
www.MATHVN.com - HOANG NGOC QUANG, Yen Bai
75
Nhn xt 3.1. T bi ton trn, ta c cc kt qu sau
1.
R
2
1
r
2
r
3
+
R
2
2
r
3
r
1
+
R
2
3
r
1
r
2
12,
2.
R
1
r
2
+r
3
+
R
2
r
3
+r
1
+
R
3
r
1
+r
2
3,
3.
R
1

r
2
r
3
+
R
2

r
3
r
1
+
R
3

r
1
r
2
6.
Chng minh. p dng bt ng thc AM GM, ta c
1.
R
2
1
r
2
r
3
+
R
2
2
r
3
r
1
+
R
2
3
r
1
r
2
3
3

R
2
1
r
2
r
3
R
2
2
r
3
r
1
R
2
3
r
1
r
2
3
3

8
2
= 12.
2.
R
1
r
2
+r
3
+
R
2
r
3
+r
1
+
R
3
r
1
+r
2
3
3
_
R
1
r
2
+r
3
R
2
r
3
+r
1
R
3
r
1
+r
2
3.
3.
R
1

r
2
r
3
+
R
2

r
3
r
1
+
R
3

r
1
r
2
3
3

R
1

r
2
r
3
R
2

r
3
r
1
R
3

r
1
r
2
3
3

8 = 6.
Bi ton 3.5. Cho tam gic ABC v P l mt im ty trong tam
gic. Chng minh rng
R
1
+R
2
+R
3
6r. (3.12)
Gii. D thy R
1
+r
1
h
a
, R
2
+r
2
h
b
, R
3
+r
3
h
c
. Cng 3 bt ng
thc ny theo v v p dng bt ng thc Erdos-Modell ta c
h
a
+h
b
+h
c
(R
1
+R
2
+R
3
) + (r
1
+r
2
+r
3
)
3
2
(R
1
+R
2
+R
3
) .
(3.13)
By gi p dng bt ng thc AM - GM ta c
9 (h
a
+h
b
+h
c
)
_
1
h
a
+
1
h
b
+
1
h
c
_
= (h
a
+h
b
+h
c
)
1
r
. (3.14)
T (3.13) v (3.14) ta c 9r h
a
+h
b
+h
c

3
2
(R
1
+R
2
+R
3
).
Do R
1
+R
2
+R
3
6r.
www.MATHVN.com - HOANG NGOC QUANG, Yen Bai
76
Bi ton 3.6. (IMO, 1991). Cho tam gic ABC v P l im trong tam
gic. Chng minh rng c t nht mt trong cc gc

PAB,

PBC,

PCA
nh hn hoc bng 30
0
.
Gii. Gi s D, E, F l chn cc ng vung gc h t P tng ng
xung cc cnh BC, CA v AB. Gi s cc gc

PAB,

PBC,

PCA u
ln hn 30
0
. Khi
PA +PB +PC =
PF
sin

PAB
+
PD
sin

PBC
+
PE
sin

PCA
<
PF
sin 30
0
+
PD
sin 30
0
+
PE
sin 30
0
= 2 (PD +PE +PF) ,
mu thun vi bt ng thc Erdos - Mordell. Suy ra cc gc

PAB,

PBC,

PCA u ln hn 30
0
l sai. T c iu phi chng
minh.
Bi ton 3.7. (Olympic 30/4, 1999, Vit Nam, ngh) Cho M l
mt im bt k trong tam gic ABC. K hiu cc khong cch t M
n 3 nh A, B, C l R
a
, R
b
, R
c
v cc khong cch t M n 3 cnh
BC, CA, AB l d
a
, d
b
, d
c
. Chng minh rng
d
a
+d
b
+d
c
2
_
d
b
d
c
R
a
+
d
c
d
a
R
b
+
d
a
d
b
R
c
_
.
Gii.
Hnh 3.6
Gi A
1
, B
1
, C
1
ln lt l chn
ng vung gc ca M xung
BC, CA, AB. Ta c
B
1
C
1
= MA. sin A = R
a
. sin A,
C
1
A
1
= MB. sin B = R
b
. sin B,
A
1
B
1
= MC. sin C = R
c
. sin C.
K MA
2
B
1
C
1
, MB
2
C
1
A
1
,
MC
2
A
1
B
1
, ta c
MA
2
= MB
1
. sin

MB
1
A
2
= MB
1
. sin

MAC
1
=
MB
1
.MC
1
MA
=
d
b
d
c
R
a
,
www.MATHVN.com - HOANG NGOC QUANG, Yen Bai
77
MB
2
= MC
1
. sin

MC
1
B
2
= MC
1
. sin

MBA
1
=
MC
1
.MA
1
MB
=
d
c
d
a
R
b
,
MC
2
= MA
1
. sin

MA
1
C
2
= MA
1
. sin

MCB
1
=
MA
1
.MB
1
MC
=
d
a
d
b
R
c
.
p dng bt ng thc Erdos-Mordell cho tam gic A
1
B
1
C
1
, ta c
MA
1
+MB
1
+MC
1
2 (MA
2
+MB
2
+MC
2
)
hay d
a
+d
b
+d
c
2
_
d
b
d
c
R
a
+
d
c
d
a
R
b
+
d
a
d
b
R
c
_
.
Bi ton 3.8. (IMO, 1996) Cho ABCDEF l mt lc gic li tha mn
AB song song vi DE, BC song song vi EF v CD song song vi FA.
K hiu R
A
, R
C
, R
E
tng ng l cc bn knh ng trn ngoi tip cc
tam gic FAB, BCD v DEF. Gi p l chu vi ca hnh lc gic. Chng
minh rng
R
A
+R
C
+R
E

p
2
.
Gii.
Hnh 3.7
Dng cc im M, N, P sao cho
MDEF, NFAB v PBCD l cc
hnh bnh hnh. Khi M, N, F
thng hng; N, P, B thng hng v
P, M, D cng thng hng. Gi s
XY Z l tam gic to bi cc ng
thng qua B, D, F v vung gc vi
FA, BC, DE, tng ng, trong B
nm trn Y Z, D nm trn ZX v
F nm trn XY . rng MNP
ng dng vi XY Z.
V DEF = DMF nn chng c bn knh ng trn ngoi tip
bng nhau. Hn na, v XM l ng knh ca ng trn ngoi tip
tam gic DMF, do XM = 2R
E
. Tng t Y N = 2R
A
v ZP = 2R
C
.
Do , bt ng thc cn chng minh c th c vit li l
XM +Y N +ZP BN +BP +DP +DM +FM +FN.
www.MATHVN.com - HOANG NGOC QUANG, Yen Bai
78
Trng hp M N P bt ng thc trn l bt ng thc Erdos
- Mordell. Ta chng minh cc trng hp cn li.
Hnh 3.8
Gi Y

, Z

i xng vi Y v
Z qua ng phn gic trong ca
gc

X. K hiu G, H ln lt l
chn ng vung gc ca X v
M trn Y

. t x = Y

, y =
XZ

, z = XY

, ta c XM
z
x
DM +
y
x
FM, Y N
x
y
FN +
z
y
BN, ZP
y
z
BP +
x
z
DP.
Cng theo v 3 bt ng thc
trn, ta c
XM +Y N +ZP
z
x
DM +
y
x
FM +
x
y
FN +
z
y
BN +
y
z
BP +
x
z
DP.
(3.15)
rng
y
z
BP +
z
y
BN =
_
y
z
+
z
y
__
BP +BN
2
_
+
_
y
z

z
y
__
BP BN
2
_
.
V hai tam gic XY Z v MNP ng dng, ta c th t r nh sau
r =
FM FN
XY
=
BN BP
Y Z
=
DP DM
ZX
.
p dng bt ng thc AM - GM, ta c
y
z
BP +
z
y
BN =
_
y
z
+
z
y
__
BP +BN
2
_

r
2
_
yx
z

zx
y
_
BP +BN
r
2
_
yx
z

zx
y
_
.
Tng t, ta c cc bt ng thc
x
y
FN +
y
x
FM FN +FM
r
2
_
xz
y

yz
x
_
,
z
x
DM +
x
z
DP DM +DP
r
2
_
zy
x

xy
z
_
.
www.MATHVN.com - HOANG NGOC QUANG, Yen Bai
79
Nu cng cc bt ng thc trn v th vo (3.15), ta c
XM +Y N +ZP BN +BP +DP +DM +FM +FN.
iu phi chng minh.
3.2. Bt ng thc Erdos-Mordell trong tam gic m rng
B 3.1. Gi s x, y, z,
1
,
2
,
3
l cc s thc vi
1
+
2
+
3
= .
Khi ta c
x
2
+y
2
+z
2
2 (yz cos
1
+zxcos
2
+xy cos
3
) (3.16)
Chng minh. V
3
= (
1
+
2
) nn cos
3
= cos (
1
+
2
) =
sin
1
sin
2
cot
1
cot
2
. Ta c
x
2
+y
2
+z
2
2 (yz cos
1
+zxcos
2
+xy cos
3
)
=(z (x cos
2
+y cos
1
))
2
+ (x sin
2
y sin
1
)
2
0.
H qu 3.1. Gi s p, q, r l cc s dng, cn
1
,
2
,
3
l cc s thc
tha mn
1
+
2
+
3
= . Khi ta c bt ng thc
p cos
1
+q cos
2
+r cos
3

1
2
_
qr
p
+
rp
q
+
pq
r
_
(3.17)
Chng minh. Thay (x, y, z) =
_
_
qr
p
,
_
rp
q
,
_
pq
r
_
vo (3.16).
nh l 3.2. (Bt ng thc Barrow) Cho tam gic ABC, M l mt
im nm trong n. Gi R
1
, R
2
, R
3
ln lt l cc khong cch t M ti
cc nh A, B, C v l
1
, l
2
, l
3
l di cc ng phn gic trong ca cc
gc

BMC,

CMA,

AMB. Ta c bt ng thc
R
1
+R
2
+R
3
2 (l
1
+l
2
+l
3
) . (3.18)
Chng minh. t

BMC = 2,

CMA = 2,

AMB = 2. Ta c + +
= . S dng cng thc ng phn gic v nh l AM GM c
l
1
=
2R
2
R
3
R
2
+R
3
cos
_
R
2
R
3
cos ,
www.MATHVN.com - HOANG NGOC QUANG, Yen Bai
80
l
2
=
2R
3
R
1
R
3
+R
1
cos
_
R
3
R
1
cos ,
l
3
=
2R
1
R
2
R
1
+R
2
cos
_
R
1
R
2
cos .
T 3 bt ng thc trn v p dng b 3.1 ta c
2 (l
1
+l
2
+l
3
) 2
_
R
2
R
3
cos + 2
_
R
3
R
1
cos + 2
_
R
1
R
2
cos
R
1
+R
2
+R
2
.
Ch 3.1. Bt ng thc Erdos-Mordell ch l h qu ca bt ng thc
Barrow. Tht vy, ta c l
1
r
1
, l
2
r
2
, l
3
r
3
. Suy ra R
1
+R
2
+R
3

2 (r
1
+r
2
+r
3
).
nh l 3.3. Cho P l mt im ty nm trong tam gic ABC. Khi
vi mi s thc x, y, z 0 bt ng thc sau lun ng
R
1
x
2
+R
1
y
2
+R
3
z
2
2 (r
1
yz +r
2
zx +r
3
xy) .
ng thc xy ra khi v ch khi tam gic ABC u, P l trc tm v
x = y = z.
Chng minh. Vi P l im nm trong tam gic ABC, theo (3.2), (3.3)
v (3.4) ta c cc bt ng thc
R
1
x
2
r
2
c
a
x
2
+r
3
b
a
x
2
, R
2
y
2
r
3
a
b
y
2
+r
1
c
b
y
2
, R
3
z
2
r
1
b
c
z
2
+r
2
a
c
z
2
.
Cng theo v 3 bt ng thc trn, sau p dng bt ng thc
AM GM ta c
R
1
x
2
+R
2
y
2
+R
3
z
2
r
1
_
c
b
y
2
+
b
c
z
2
_
+r
2
_
a
c
z
2
+
c
a
x
2
_
+r
3
_
a
b
y
2
+
b
a
x
2
_
2 (r
1
yz +r
2
zx +r
3
xy) .
Sau y ta s m rng bt ng thc Erdos-Mordell i vi hai im
ty nm trong tam gic, c kt qu trc ht ta xt cc b
sau.
www.MATHVN.com - HOANG NGOC QUANG, Yen Bai
81
B 3.2. Vi mi s thc x
1
, x
2
, x
3
, y
1
, y
2
, y
3
tha mn x
1
x
2
+x
2
x
3
+
x
3
x
1
0 v y
1
y
2
+y
2
y
3
+y
3
y
1
0 ta lun c bt ng thc sau
(x
2
+x
3
) y
1
+ (x
3
+x
1
) y
2
+ (x
1
+x
2
) y
3
2
_
(x
1
x
2
+x
2
x
3
+x
3
x
1
) (y
1
y
2
+y
2
y
3
+y
3
y
1
)
Du ng thc xy ra khi v ch khi
x
1
y
1
=
x
2
y
2
=
x
3
y
3
.
Chng minh. p dng bt ng thc Cauchy-Schwarz ta c
(x
2
+x
3
) y
1
+ (x
3
+x
1
) y
2
+ (x
1
+x
2
) y
3
=(x
1
+x
2
+x
3
) (y
1
+y
2
+y
3
) (x
1
y
1
+x
2
y
2
+x
3
y
3
)
=
_
_
x
2
1
+x
2
2
+x
2
3
+ 2 (x
1
x
2
+x
2
x
3
+x
3
x
1
)
_
y
2
1
+y
2
2
+y
2
3
+ 2 (y
1
y
2
+y
2
y
3
+y
3
y
1
)

(x
1
y
1
+x
2
y
2
+x
3
y
3
)
2
_
(y
1
y
2
+y
2
y
3
+y
3
y
1
) (x
1
x
2
+x
2
x
3
+x
3
x
1
) +
_
_
x
2
1
+x
2
2
+x
2
3
_ _
y
2
1
+y
2
2
+y
2
3
_
(x
1
y
1
+x
2
y
2
+x
3
y
3
)
2
_
(y
1
y
2
+y
2
y
3
+y
3
y
1
) (x
1
x
2
+x
2
x
3
+x
3
x
1
).
B 3.3. Cho tam gic ABC v x, y, z l cc s thc dng. Bt ng
thc sau lun ng
x sin A +y sin B +z sin C
1
2
(xy +yz +zx)
_
x +y +z
xyz
. (3.19)
ng thc xy ra khi v ch khi x = y = z v ABC l tam gic u.
Chng minh. Trc ht ta c
x
2
+y
2
+z
2
+ 2yz cos 2A + 2zxcos 2B + 2xy cos 2C 0. (3.20)
V x
2
+ y
2
+ z
2
+ 2yz cos 2A + 2zxcos 2B + 2xy cos 2C =
(x +z cos 2B +y cos 2C)
2
+ (z sin 2B y sin 2C)
2
0.
Trong (3.20) thay (x, y, z) bng
_
1
x
,
1
y
,
1
z
_
, ta c bt ng thc
x cos 2A +y cos 2B +z cos 2C
1
2
_
yz
x
+
zx
y
+
xy
z
_
. (3.21)
www.MATHVN.com - HOANG NGOC QUANG, Yen Bai
82
Thay cos 2A = 1 2 sin
2
A, cos 2B = 1 2 sin
2
B, cos 2C = 1 2 sin
2
C
vo (3.21), ta c
2
_
xsin
2
A + ysin
2
B +zsin
2
C
_
x +y +z +
1
2
_
yz
x
+
zx
y
+
xy
z
_
hay
xsin
2
A + ysin
2
B +zsin
2
C
(yz +zx +xy)
2
4xyz
. (3.22)
By gi, p dng bt ng thc Cauchy-Schwarz v s dng (3.22), ta
c
(x sin A + y sin B +z sin C)
2

_
xsin
2
A + ysin
2
B +zsin
2
C
_
(x +y +z)

(yz +zx +xy)


2
4xyz
(x +y +z) .
Suy ra x sin A + y sin B +z sin C
1
2
(yz + zx + xy)
_
x +y +z
xyz
.
ng thc xy ra khi v ch khi
_
_
_
x +z cos 2B +y cos 2C = 0
z sin 2B y sin 2C = 0
sin A = sin B = sin C

_
x = y = z
A = B = C

_
x = y = z
ABC u.
B 3.4. Cho tam gic ABC v im P ty trong n, ta lun c
bt ng thc
_
abr
1
r
2
+bcr
2
r
3
+car
3
r
1
2 (r
1
r
2
+r
2
r
3
+r
3
r
1
) . (3.23)
Chng minh. t x = [PBC] , y = [PCA] , z = [PAB], ta c
x
x +y +z
=
[PBC]
[ABC]
=
r
1
a
bc sin A
=
2r
1
bc
.
a
2 sin A
=
2Rr
1
bc
.
Suy ra r
1
=
bc
2R
.
x
x +y +z
.
Tng t r
2
=
ca
2R
.
y
x +y +z
, r
3
=
ab
2R
.
z
x +y +z
.
Do bt ng thc (3.23) tng ng vi
abc
2R(x +y +z)

xy +yz +zx
abc
R(x +y +z)
2
_
a
2R
yz +
b
2R
zx +
c
2R
xy
_
www.MATHVN.com - HOANG NGOC QUANG, Yen Bai
83
hay
1
2
(x +y +z)

xy +yz +zx yz sin A +zxsin B +xy sin C. (3.24)


Bt ng thc (3.24) suy ra t b 3.3 bng cch thay (x, y, z) bng
_
1
x
,
1
y
,
1
z
_
. Hon thnh chng minh b 3.4.
nh l 3.4. Cho P, Q l hai im ty nm trong tam gic ABC.
Gi R
1
, R
2
, R
3
v r
1
, r
2
, r
3
ln lt l cc khong cch t P ti cc nh
A, B, C v cc cnh BC, CA, AB. i vi im Q nh ngha D
1
, D
2
, D
3
v d
1
, d
2
, d
3
mt cch tng t. Khi ta c cc bt ng thc sau.
R
1
D
1
+R
2
D
2
+R
3
D
3
4 (r
1
r
2
+r
2
r
3
+r
3
r
1
) , (3.25)
R
1
D
1
+R
2
D
2
+R
3
D
3
4 (d
1
d
2
+d
2
d
3
+d
3
d
1
) . (3.26)
Chng minh.
Hnh 3.9
Theo cc bt ng thc (3.2), (3.3), (3.4), b 3.2, b 3.4 v nh
l 4.1 ta c
R
1
D
1
+R
2
D
2
+R
3
D
3
= aR
1
.
D
1
a
+bR
2
.
D
2
b
+cR
3
.
D
3
c
(cr
2
+br
3
)
D
1
a
+ (ar
3
+cr
1
)
D
2
b
+ (br
1
+ar
2
)
D
3
c
2

(abr
1
r
2
+bcr
2
r
3
+ca
3
r
1
)
_
D
1
D
2
ab
+
D
2
D
3
bc
+
D
3
D
1
ca
_
2
_
abr
1
r
2
+bcr
2
r
3
+car
3
r
1
4 (r
1
r
2
+r
2
r
3
+r
3
r
1
) .
www.MATHVN.com - HOANG NGOC QUANG, Yen Bai
84
Bt ng thc (3.25) c chng minh. Cn bt ng thc (3.26) c
chng minh tng t.
H qu 3.2. Vi mi tam gic ABC v hai im trong n P, Q, ta c
R
1
D
1
+R
2
D
2
+R
3
D
3
4
_
(r
1
r
2
+r
2
r
3
+r
3
r
1
) (d
1
d
2
+d
2
d
3
+d
3
d
1
).
(3.27)
Chng minh. Cng theo v hai bt ng thc (3.25) v (3.26), sau p
dng bt ng thc AM GM, ta thu c bt ng thc (3.27).
nh l 3.5. Cho tam gic ABC bt k vi di cc cnh l a, b, c
v P l im nm trong tam gic. K hiu R
1
, R
2
, R
3
l khong cch t
P tng ng n cc nh A, B, C v r
1
, r
2
, r
3
tng ng l khong cch
t P n cc cnh BC, CA, AB. Vi mi s thc [0, 1] ta lun c
R

1
+R

2
+R

3
2

(r

1
+r

2
+r

3
) .
Trc ht ta xt b sau:
B 3.5. Vi mi s dng x, y v vi mi s thc tha mn 0 <
1 ta c
(x +y)

2
1
(x

+y

) . (3.28)
Chng minh. Ta c bt ng thc (3.28) tng ng vi
_
x
y
+ 1
_

2
1
__
x
y
_

+ 1
_
. (3.29)
t t =
x
y
> 0, bt ng thc (3.29) tr thnh (t + 1)

2
1
(t

+ 1) .
Xt f(t) = (t + 1)

2
1
(t

+ 1), t > 0. Ta ch cn xt vi 0 < < 1


(v nu = 1 th (3.28) hin nhin ng).
Ta c f

(t)
= (t + 1)
1
t
1
2
1
=
_
(t + 1)
1
(2t)
1
_
.
f

(t)
= 0 t = 1.
T bng xt du suy ra f(t) 0, vi mi t > 0. Vy b c chng
minh.
By gi ta chng minh nh l 3.5: p dng (3.2) v b (3.5) ta
c R

1

_
r
2
c
a
+
r
3
b
a
_

2
1
_
_
r
2
c
a
_

+
_
r
3
b
a
_

_
,
www.MATHVN.com - HOANG NGOC QUANG, Yen Bai
85
Hnh 3.10 Bng xt du
tng t R

2
2
1
__
r
3
a
b
_

+
_
r
1
c
b
_

, R

3
2
1
_
_
r
1
b
c
_

+
_
r
2
a
c
_

_
.
Cng theo v ba bt ng thc trn, sau p dng bt ng thc
AM GM ta c
R

1
+R

2
+R

3
2
1
_
_
r
1
c
b
_

+
_
r
1
b
c
_

_
+ 2
1
__
r
2
c
a
_

+
_
r
2
a
c
_

_
+
+2
1
_
_
r
3
a
b
_

+
_
r
3
b
a
_

_
2

(r

1
+r

2
+r

3
) .
3.3. M rng bt ng thc Erdos-Mordell trong t gic
nh l 3.2 cho ta tng xy dng bt ng thc Erdos-Mordell m
rng trong t gic
B 3.6. Gi s x
1
, x
2
, x
3
, x
4
,
1
,
2
,
3
,
4
, l cc s thc ty v

1
+
2
+
3
+
4
= . Khi ta c bt ng thc

2 (x
1
x
2
cos
1
+x
2
x
3
cos
2
+x
3
x
4
cos
3
+x
4
x
1
cos
4
) x
2
1
+x
2
2
+x
2
3
+x
2
4
.
(3.30)
Chng minh. T nhn xt
1
+
2
+(
3
+
4
) = v
3
+
4
+(
1
+
2
) = ,
p dng b 3.1 ta c
x
1

2
x
2
cos
1
+x
2
x
3

2
cos
2
+
x
1

2
x
3

2
cos (
3
+
4
)
1
2
_
x
2
1
2
+x
2
2
+
x
2
3
2
_
,
x
3

2
x
4
cos
3
+x
4
x
1

2
cos
4
+
x
3

2
x
1

2
cos (
1
+
2
)
1
2
_
x
2
3
2
+x
2
4
+
x
2
1
2
_
.
Cng theo v hai bt ng thc trn, vi lu rng cos (
3
+
4
) +
cos (
1
+
2
) = 0, ta thu c bt ng thc cn chng minh.
www.MATHVN.com - HOANG NGOC QUANG, Yen Bai
86
nh l 3.6. T gic li ABCD, P l mt im nm trong n.
K hiu R
1
, R
2
, R
3
, R
4
ln lt l khong cch t P ti cc nh
A, B, C, D v l
1
, l
2
, l
3
, l
4
di cc ng phn gic ca cc gc

APB,

BPC,

CPD,

DPA, tng ng. Khi


R
1
+R
2
+R
3
+R
4

2 (l
1
+l
2
+l
3
+l
4
) . (3.31)
Chng minh. t

APB = 2,

BPC = 2,

CPD = 2,

DPA = 2, ta c
+ + + = .
Hnh 3.11
S dng cng thc ng phn gic
v bt ng thc AM GM ta c
l
1
=
2R
1
R
2
R
1
+R
2
cos
_
R
1
R
2
cos ,
l
2
=
2R
2
R
3
R
2
+R
3
cos
_
R
2
R
3
cos ,
l
3
=
2R
3
R
4
R
3
+R
4
cos
_
R
3
R
4
cos ,
l
4
=
2R
4
R
1
R
4
+R
1
cos
_
R
4
R
1
cos .
T 4 bt ng thc trn v p dng
b 3.6 ta c

2 (l
1
+l
2
+l
3
+l
4
)

2(
_
R
1
R
2
cos +
_
R
2
R
3
cos +
+
_
R
3
R
4
cos +
_
R
4
R
1
cos ) R
1
+R
2
+R
3
+R
4
.
Ch 3.2. Ta lun c l
1
r
1
, l
2
r
2
, l
3
r
3
, l
4
r
4
trong
r
1
, r
2
, r
3
, r
4
ln lt l khong cch t P ti cc cnh AB, BC, CD, DA
ca t gic li ABCD. Do , t bt ng thc (3.31) ta c
R
1
+R
2
+R
3
+R
4

2 (r
1
+r
2
+r
3
+r
4
) . (3.32)
Bt ng thc (3.32) c xem l bt ng thc Erdos - Mordell
trong t gic.
www.MATHVN.com - HOANG NGOC QUANG, Yen Bai
87
3.4. M rng bt ng thc Erdos-Mordell trong a gic
B 3.7. Vi mi dy s thc x
1
, x
2
, , x
n
ta lun c
cos

n
n

k=1
x
2
k

n1

k=1
x
k
x
k+1
+x
1
x
n
=
=
n2

k=1
1
2 sin
k
n
sin
(k+1)
n
_
sin
(k + 1)
n
x
k
sin
k
n
x
k+1
+ sin

n
x
n
_
2
.
Chng minh. Bng cch so snh cc h s ca x
2
k
v x
k
x
k+1
. Chng hn,
h s ca x
2
k
l
sin
(k+1)
n
2 sin
k
n
+
sin
(k1)
n
2 sin
k
n
=
2 sin
k
n
cos

n
2 sin
k
n
= cos

n
,
vi k = 2, 3, , n 2 v h s ca x
2
1
l
sin
2
n
2 sin

n
= cos

n
, h s ca x
2
n1
l
sin
(n2)
n
2 sin
(n1)
n
=
sin
2
n
2 sin

n
= cos

n
v h s ca x
2
n
l
n2

k=1
sin
2
n
2 sin
k
n
sin
(k+1)
n
=
n2

k=1
sin

n
2
_
cot
k
n
cot
(k + 1)
n
_
=
sin

n
2
_
cot

n
cot
(n 1)
n
_
= cos

n
.
H s ca x
k
x
k+1
l
sin
(k+1)
n
sin
k
n
2 sin
k
n
sin
(k+1)
n
= 1 vi k = 1, 2, , n2 v h
s ca x
n1
x
n
l
2 sin
(n2)
n
sin

n
2 sin
(n2)
n
sin
(n1)
n
=
sin

n
sin

n
= 1. H s ca x
1
x
n
l
2 sin
2
n
sin

n
2 sin

n
sin
2
n
= 1 v h s ca x
k
x
n
vi k = 2, 3, , n 2 l
n2

k=2
2 sin
(k+1)
n
sin

n
2 sin
k
n
sin
(k+1)
n

n3

k=1
2 sin
k
n
sin

n
2 sin
k
n
sin
(k+1)
n
=
n2

k=2
sin

n
sin
k
n
=
n3

k=2
sin

n
sin
(k+1)
n
= 0.
www.MATHVN.com - HOANG NGOC QUANG, Yen Bai
88
H qu 3.3. Vi mi dy s thc x
1
, x
2
, , x
n
ta lun c
cos

n
n

k=1
x
2
k

n1

k=1
x
k
x
k+1
+x
1
x
n
0. (3.33)
Chng minh. V sin
k
n
> 0, sin
(k+1)
n
> 0 vi mi k = 1, 2, , n nn
n2

k=1
1
2 sin
k
n
sin
(k+1)
n
_
sin
(k + 1)
n
x
k
sin
k
n
x
k+1
+ sin

n
x
n
_
2
0.
T bt ng thc trn v b 3.7 c bt ng thc (3.33)
B 3.8. (Wolstenholme-Lenhard) Vi mi dy s thc
R
1
, R
2
, , R
n
,R
n+1
= R
1
v mi dy
1
,
2
, ,
n
tha mn
n

k=1

k
= (2r + 1) , r N, bt ng thc sau lun ng
cos

n
n

k=1
R
2
k

n

k=1
R
k
R
k+1
cos
k
. (3.34)
Chng minh. p dng h qu 3.3, tng t ta c
cos

n
n

k=1
y
2
k

n1

k=1
y
k
y
k+1
+y
1
y
n
0. (3.35)
vi mi dy s thc y
1
, y
2
, , y
n
.
t x
k
= R
k
cos
k
v y
k
= R
k
sin
k
trong R
k
R v
k
R .
Cng theo v hai bt ng thc (3.33) v (3.35) ta c
cos

n
n

k=1
R
2
k

n1

k=1
R
k
R
k+1
[cos
k
cos
k+1
+ sin
k
sin
k+1
]
+R
1
R
n
[cos
1
cos
n
+ sin
1
sin
n
] 0
hay cos

n
n

k=1
R
2
k

n1

k=1
R
k
R
k+1
cos (
k

k+1
) +R
1
R
n
cos (
1

n
) 0.
t
k

k+1
=
k
vi k = 1, 2, , n 1 v
1

n
= (2r + 1)

n
, r N ta c bt ng thc cos

n
n

k=1
R
2
k

n

k=1
R
k
R
k+1
cos
k
, trong
R
k
R,
k
R v
n

k=1

k
= (2r + 1) , r N.
www.MATHVN.com - HOANG NGOC QUANG, Yen Bai
89
nh l 3.7. (Bt ng thc Erdos-Mordell cho a gic) Cho
A
1
A
2
A
n
, (n 3) l a gic li v P l mt im ty nm
trong n. Gi R
1
, R
2
, , R
n
ln lt l khong cch t P ti cc nh
A
1
, A
2
, , A
n
v r
1
, r
2
, , r
n
ln lt l khong cch t im P ti
cc cnh A
1
A
2
, A
2
A
3
, , A
n
A
1
. Ta lun c bt ng thc
n

k=1
R
k

1
cos

n
n

k=1
r
k
, (3.36)
ng thc xy ra khi v ch khi A
1
A
2
A
n
l a gic u v P l tm
ca n.
Chng minh. Gi l
1
, l
2
, , l
n
ln lt l di cc ng phn gic
ca cc gc

A
1
PA
2
,

A
2
PA
3
, ,

A
n
PA
1
. t

A
1
PA
2
= 2
1
,

A
2
PA
3
=
2
2
, ,

A
n
PA
1
= 2
n
, ta c
1
+
2
+ +
n
= .
Theo cng thc ng phn gic v bt ng thc AM GM ta c
l
1
=
2R
1
R
2
R
1
+R
2
cos
1

_
R
1
R
2
cos
1
,
l
2
=
2R
2
R
3
R
2
+R
3
cos
2

_
R
2
R
3
cos
2
,
........................................................
l
n
=
2R
n
R
1
R
n
+R
1
cos
n

_
R
4
R
1
cos
n
.
T n bt ng thc trn v p dng b 3.8 ta c
1
cos
1
n
n

k=1
l
k

1
cos
1
n
n

k=1
_
R
k
R
k+1
. cos
k

n

k=1
R
k
.
V l
1
r
1
, l
2
r
2
, , l
n
r
n
nn t bt ng thc trn, ta c
n

k=1
R
k

1
cos

n
n

k=1
r
k
.
ng thc xy ra khi v ch khi A
1
A
2
A
n
l a gic u v P l tm
ca n.
www.MATHVN.com - HOANG NGOC QUANG, Yen Bai
90
3.5. M rng bt ng thc Erdos-Mordell trong t din
Bi ton 3.9. Cho t din ABCD v P l mt im ty nm
trong n. Gi d
1
, d
2
, d
3
, d
4
ln lt l khong cch t P n cc nh
A, B, C, D v h
1
, h
2
, h
3
, h
4
ln lt l khong cch t im P n cc
mt (BCD), (CDA), (DAB), (ABC). Chng minh rng
d
1
+d
2
+d
3
+d
4
2
_
_
h
1
h
2
+
_
h
1
h
3
+
_
h
1
h
4
+
_
h
2
h
3
+
_
h
2
h
4
+
_
h
3
h
4
_
. (3.37)
Gii. Gi
Hnh 3.12
h
a
l di ng cao ca t
din h t nh A xung mt y
BCD. S
1
, S
2
, S
3
, S
4
ln lt l
din tch ca cc tam gic BCD,
CDA, DAB, ABC. V
1
, V
2
, V
3
, V
4
,
V ln lt l th tch ca cc khi
t din PBCD, PCDA, PDAB,
PABC, ABCD.
Ta c d
1
+ h
1
h
a
d
1
S
1
+
h
1
S
1
h
a
S
1
d
1
S
1
+ 3V
1

3V = 3 (V
1
+V
2
+V
3
+V
4
)
d
1
S
1
3V
2
+ 3V
3
+ 3V
4
= h
2
S
2
+
h
3
S
3
+h
4
S
4
d
1
h
2
S
2
S
1
+h
3
S
3
S
1
+h
4
S
4
S
1
. Tng t d
2
h
1
S
1
S
2
+h
3
S
3
S
2
+
h
4
S
4
S
2
, d
3
h
1
S
1
S
3
+h
2
S
3
S
3
+h
4
S
4
S
3
, d
4
h
1
S
1
S
4
+h
2
S
3
S
4
+h
4
S
3
S
4
.
Cng theo v 4 bt ng thc trn v p dng bt ng thc AMGM
c
d
1
+d
2
+d
3
+d
4

_
h
1
S
1
S
2
+h
2
S
2
S
1
_
+
_
h
1
S
1
S
3
+h
3
S
3
S
1
_
+
_
h
1
S
1
S
4
+h
4
S
4
S
1
_
+
_
h
2
S
2
S
3
+h
3
S
3
S
2
_
+
_
h
2
S
2
S
4
+h
4
S
4
S
2
_
+
_
h
3
S
3
S
4
+h
4
S
4
S
3
_
.
2
_
_
h
1
h
2
+
_
h
1
h
3
+
_
h
1
h
4
+
_
h
2
h
3
+
_
h
2
h
4
+
_
h
3
h
4
_
Nhn xt 3.2. c bit ha im P v t din ABCD ta thu c cc
kt qu sau:
www.MATHVN.com - HOANG NGOC QUANG, Yen Bai
91
+ Nu P I (I l tm mt cu ni tip t din ABCD) th t (3.37)
ta c
d
1
+d
2
+d
3
+d
4
12r, (3.38)
trong r l bn knh mt cu ni tip t din.
+ Nu ABCD l t din gn u (AB = CD, BC = AD, AC = BD)
c din tch mt mt l S th t (3.37) ta c
d
1
+d
2
+d
3
+d
4
3h, (3.39)
trong h l chiu cao ca t din gn u ABCD.
+ Nu ABCD l t din u, cnh a th t (3.39) ta c
d
1
+d
2
+d
3
+d
4
a

6. (3.40)
+ Nu ABCD l t din vung ti A (tc l ABAC, ABAD,
ACAD) v P I th d
1
= IA =

3r kt hp vi (3.38) ta c
d
2
+d
3
+d
4

_
12

3
_
r. (3.41)
www.MATHVN.com - HOANG NGOC QUANG, Yen Bai
92
Chng 4
Cc bt ng thc c trng
Chng ny trnh by mt s bt ng thc c trng v lm c bit
ha thu c cc bt ng thc hnh hc mi lin quan n khong
cch t mt im nm trong tam gic n cc nh v cc cnh. Ni
dung ch yu ca chng c hnh thnh t cc ti liu [9], [13] v [14].
4.1. Bt ng thc dng Hayashi v cc h qu
4.1.1. Bt ng thc Hayashi
nh l 4.1. (Bt ng thc Hayashi) Cho M l mt im ty trong
mt phng cha tam gic ABC vi di cc cnh l a, b, c. Khi
aMB.MC +bMC.MA +cMA.MB abc. (4.1)
y l mt nh l c cch chng minh hnh hc kh phc tp. Sau
y bng cch dng s phc s cho ta mt cch chng minh nh l rt
ngn gn v n tng. Ch cn dng mt ng nht thc i s quen
thuc v biu din hnh hc ca s phc.
Chng minh. Ta t tam gic ABC ln mt phng phc v gi m, , ,
tng ng l to v ca M, A, B, C. T ng nht i s
(m)(m)
( )( )
+
(m)(m)
( )( )
+
(m)(m)
( )( )
= 1, (4.2)
ng vi mi , , i mt khc nhau v vi mi m. ng nht thc
ny c th chng minh bng cch v tri l mt tam thc bc hai
theo m v c gi tr bng 1 ti ba im phn bit , , , do ng
nht 1.
T ng nht (4.2) chuyn qua modul ta c
|m| |m|
| | | |
+
|m| |m|
| | | |
+
|m| |m|
| | | |
1. (4.3)
www.MATHVN.com - HOANG NGOC QUANG, Yen Bai
93
Theo cch t trn ta c MA = |m |, MB = |m |, |MC| =
|m|, c = AB = | |, a = BC = | |, b = CA = | |. Khi
(4.3) tng ng vi
MB.MC
bc
+
MC.MA
ca
+
MA.MB
ab
1.
Ch 4.1. Trng hp xy ra du ng thc ta s xt n trong bi
ton 4.1 mc di.
nh l 4.2. Cho M l mt im ty trong mt phng cha tam gic
ABC vi di cc cnh l a, b, c. Khi
aMA
2
+bMB
2
+cMC
2
abc. (4.4)
Chng minh. Xt gc ca mt phng phc M v gi , , l ta v
cc nh A, B, C ca tam gic ABC. T ng nht thc

2
( ) ( )
+

2
( ) ( )
+

2
( ) ( )
= 1, (4.5)
ng vi mi , , i mt khc nhau. Chuyn qua modul ta c
1 =

cyc

2
( ) ( )

cyc
||
2
| | | |
. (4.6)
Theo cch t trn ta c || = PA, || = PB, || = PC v | | =
a, | | = b, | | = c, bt ng thc (4.6) tng ng vi bt
ng thc aMA
2
+bMB
2
+cMC
2
abc.
nh l 4.3. Cho M l mt im ty trong mt phng cha tam gic
ABC vi di cc cnh l a, b, c. Khi
aMA
3
+bMB
3
+cMC
3
abc.MG. (4.7)
Trong G l trng tm ca tam gic ABC.
Chng minh. T ng nht thc
x
3
(y z) +y
3
(z x) +z
3
(x y) = (x y) (y z) (z x) (x +y +z) , (4.8)
ng vi mi s phc x, y, z. Chuyn qua modul, ta c
|x|
3
|y z| +|y|
3
|z x| +|z|
3
|x y| |x y| |y z| |z x| |x +y +z| . (4.9)
Xt gc ca mt phng phc G v gi , , , m l ta v ca cc im
A, B, C, M tng ng. Trong bt ng thc (4.9) xt x = m , y =
m, z = m ta thu c bt ng thc (4.7).
www.MATHVN.com - HOANG NGOC QUANG, Yen Bai
94
4.1.2. Cc h qu ca bt ng thc hyashi
H qu 4.1. (Bt ng thc Euler) K hiu R, r ln lt l bn knh
ng trn ngoi tip v ni tip tam gic ABC. Khi R 2r.
Chng minh. Xt trng hp M O (trong O l tm ng
trn ngoi tip tam gic ABC), bt ng thc (4.1) tng ng vi
R
2
(a +b +c) abc. Do R
2

abc
a +b +c
=
abc
2p
=
4R.S
2.
S
r
= 2Rr, t
R 2r.
H qu 4.2. K hiu m
a
, m
b
, m
c
ln lt l di cc ng trung tuyn
xut pht t cc nh A, B, C ca tam gic ABC. Khi
m
a
m
b
ab
+
m
b
m
c
bc
+
m
c
m
a
ca

9
4
.
Chng minh. Xt trng hp M G (G l trng tm ca tam gic
ABC), t bt ng thc (4.1) ta suy ra bt ng thc cn chng minh.
Du ng thc xy ra khi v ch khi tam gic ABC u.
4.1.3. Bi ton p dng
Bi ton 4.1. (Olympic ton Trung Quc 1998) Cho ABC l tam gic
nhn v cho P l mt im trong n. Chng minh rng
aPB.PC +bPC.PA +bPA.PB abc. (4.10)
ng thc xy ra nu v ch nu P l trc tm ca tam gic ABC.
Gii. Gi s P l gc ca mt phng phc v t , , l ta v ca
A, B, C, tng ng. Khi vi mi , , ta c ng nht thc sau
( ) + ( ) +( ) = ( ) ( ) ( ) . (4.11)
Ly modul hai v ng nht trn ta c bt ng thc
| ( )| +| ( )| +|( )| |( ) ( ) ( )| . (4.12)
Theo cch t trn ta c PA = || , PB = || , PC = || , c =
| | , a = | | , b = | |. Do c bt ng thc
aPB.PC +bPC.PA +bPA.PB abc.
www.MATHVN.com - HOANG NGOC QUANG, Yen Bai
95
By gi, ta xc nh xem khi no ng thc xy ra. t
z
1
=

( ) ( )
, z
2
=

( ) ( )
, z
3
=

( ) ( )
,
vi cch t ny, t (4.11) v (4.12) ta c cc ng thc v bt ng
thc sau z
1
+z
2
+z
3
= 1, |z
1
| +|z
2
| +|z
3
| 1.
Vy ng thc xy ra khi v ch khi z
1
, z
2
, z
3
l cc s thc dng.
Ta chng minh z
1
, z
2
, z
3
l cc s thc dng khi v ch khi P l trc
tm ca tam gic ABC. Tht vy, gi s z
1
, z
2
, z
3
l cc s thc dng.
V

z
1
z
2
z
3
=
_


_
2
,
z
2
z
3
z
1
=
_


_
2
,
z
3
z
1
z
2
=
_


_
2
.
T s phc

c bnh phng l s mt s thc m nn l cc s thun


o; vy gc to bi gia cc vc t biu din l

PA v vc t biu din
l

BC l gc c s o 90
0
, suy ra PABC. Tng t

l s
thun o, suy ra PBCA. Vy P l trc tm ca tam gic ABC.
o li, nu P l trc tm ca tam gic nhn ABC. im P bn
trong tam gic v PABC nn gc to bi gia vc t

PA v

BC c
s o 90
0
, suy ra tn ti s thc dng r
1
sao cho

= r
1
i. Tng
t, tn ti cc s thc dng r
2
, r
3
sao cho

= r
2
i,

= r
3
i. Vy
z
1
, z
2
, z
3
l cc s thc dng. T cc kt qu trn ta kt lun ng thc
xy ra khi v ch khi P l trc tm ca tam gic ABC.
Bi ton 4.2. Cho G l trng tm ca tam gic ABC . Gi R
1
, R
2
, R
3
, R
l bn knh ng trn ngoi tip cc tam gic GBC, GCA, GAB, ABC
tng ng. Chng minh rng R
1
+R
2
+R
3
3R.
Gii. p dng nh l 4.1 vi G l trng tm ca tam gic ABC. Ta c
aGB.GC +bGC.GA +cGA.GB abc, (4.13)
trong a, b, c l di cc cnh ca tam gic ABC.
Mt khc, ta c aGB.GC = 4R
1
. [GBC] =
4
3
R
1
. [ABC]. Tng t
bGC.GA =
4
3
R
2
. [ABC] , cGA.GB =
4
3
R
3
. [ABC] . T (4.13) tng
ng vi
4
3
(R
1
+R
2
+R
3
) . [ABC] 4R. [ABC]. Suy ra R
1
+R
2
+R
3

3R.
www.MATHVN.com - HOANG NGOC QUANG, Yen Bai
96
Bi ton 4.3. (Chn i tuyn IMO Rumani 2004) Cho tam gic ABC
v P l mt im trong n. Gi R
1
, R
2
, R
3
, R l bn knh ng trn
ngoi tip cc tam gic PBC, PCA, PAB, ABC tng ng. Cc ng
thng PA, PB, PC ln lt ct cc cnh BC, CA, AB ti A
1
, B
1
, C
1
. K
hiu k
1
=
PA
1
AA
1
, k
2
=
PB
1
BB
1
, k
2
=
PC
1
CC
1
. Chng minh rng k
1
R
1
+ k
2
R
2
+
k
3
R
3
R.
Gii.
Hnh 4.1
Ta c k
1
=
PA
1
AA
1
=
[PA
1
B]
[AA
1
B]
=
[PA
1
C]
[AA
1
C]
=
[PA
1
B]+[PA
1
C]
[AA
1
B]+[AA
1
C]
=
[PBC]
[ABC]
, tng t k
2
=
[PCA]
[ABC]
, k
3
=
[PAB]
[ABC]
.
Mt khc [ABC] =
abc
4R
, [PBC] =
aPB.PC
4R
1
, [PCA] =
bPC.PA
4R
2
, [PAB] =
cPA.PB
4R
3
.
Bt ng thc Hayashi p dng cho
tam gic ABC v im P nm trong
n tng ng vi bt ng thc sau
aPB.PC
4R
1
.R
1
+
bPC.PA
4R
2
.R
2
+
cPA.PB
4R
3
.R
3
R.
abc
4R
.
Do , ta c [PBC] .R
1
+[PCA] .R
2
+[PAB] .R
3
R. [ABC] hay
[PBC]
[ABC]
.R
1
+
[PCA]
[ABC]
.R
2
+
[PAB]
[ABC]
.R
3
R. T , c bt ng thc cn
chng minh k
1
R
1
+k
2
R
2
+k
3
R
3
R.
4.2. Bt ng thc Weizenbock suy rng v cc h qu
4.2.1. Bt ng thc Weizenbock suy rng
Chng ta bit cc bt ng thc sau y: Gi s a, b, c l di
ba cnh, cn S l din tch ca mt tam gic th
a
2
+b
2
+c
2
(a b)
2
+ (b c)
2
+ (c a)
2
+ 4

3S. (4.14)
a
2
+b
2
+c
2
4

3S. (4.15)
Bt ng thc (4.14) c gi l bt ng thc Hadwiger - Finsler, cn
bt ng thc (4.15) l bt ng thc Weizenbock l h qu ca bt
www.MATHVN.com - HOANG NGOC QUANG, Yen Bai
97
ng thc Hadwiger - Finsler. Trong mc ny xt s m rng ca bt
ng thc Weizenbock.
nh l 4.4. Cho x, y, z l cc s thc tha mn cc iu kin x+y, y +
z, z +x, xy +yz +zx 0. t a, b, c l di ba cnh v S l din tch
ca tam gic ABC. Khi
xa
2
+yb
2
+zc
2
4

xy +yz +zx.S. (4.16)


Chng minh. p dng nh l hm s cosin c
2
= a
2
+ b
2
2ab cos C v
cng thc din tch S =
1
2
ab sin C, ta c
xa
2
+yb
2
+zc
2
4

xy +yz +zx.S
xa
2
+yb
2
+z
_
a
2
+b
2
2ab cos C
_

xy +yz +zx.2ab sin C


(x +z) a
2
+ (y +z) b
2
2ab
_
xy +yz +zxsin C +z cos C

(x +z)
a
b
+ (y +z)
b
a
2
_
sin C

xy +yz +zx +z cos C

. (4.17)
p dng bt ng thc Cauchy - Schwarz, ta c
_
sin C

xy +yz +zx +z cos C

_
xy +yz +zx +z
2
_ _
sin
2
C + cos
2
C
_
= xy +yz +zx +z
2
= (x +z) (y +z) .
Mt khc
_
(x +z)
a
b
+ (y +z)
b
a
_
2
4 (x +z) (y +z) .
Do (4.17) ng.
Du ng thc xy ra khi v ch khi
_

_
(x +z)
a
b
= (y +z)
b
a
cos C
z
=
sin C

xy +yz +zx

_
a

y +z
=
b

x +z
cos
2
C
z
2
=
sin
2
C
xy +yz +zx
=
sin
2
C +cos
2
C
xy +yz +zx +z
2
=
1
(x +z) (y +z)
.
www.MATHVN.com - HOANG NGOC QUANG, Yen Bai
98
Thay b v cos C tng ng vo biu thc c
2
= a
2
+ b
2
2bc cos C ta
c
c
2
= a
2
+a
2
x +z
y +z
2a
2
_
x +z
y +z
z
_
(x +z) (y +z)

_
c
a
_
2
= 1 +
x +z
y +z
2
z
y +z

c
a
=
_
x +y
y +z

a

y +z
=
c

x +y
.
Vy ng thc xy ra khi v ch khi
a

y+z
=
b

x +z
=
c

x +y
.
Bt ng thc (4.16) c gi l bt ng thc weizenbock suy rng.
Tip theo, ta s m rng bt ng thc Hadwiger - Finsler.
nh l 4.5. Cho x, y, z l cc s thc dng. Bt ng thc sau lun
ng
(y +z) a
2
+ (z +x) b
2
+ (x +y) c
2
8

xy +yz +zxS + (x +y) (a b)


2
+(y +z) (b c)
2
+ (z +x) (c a)
2
. (4.18)
Chng minh.
Hnh 4.2
Gi A

, B

, C

ln lt l
tm ng trn bng tip
cc gc A, B, C. D dng
chng minh c b ba im
(A

B, C

) , (B

, C, A

) , (C

, A, B

)
thng hng. Gi K, H l
chn ng cao h t C

xung AB v t B

xung
AC. p dng nh l 4.4
vo tam gic A

ta c
zA

B
2
+ xB

C
2
+ yC

A
2

xy +yz +zxS
A

C
.
Ta c
B

=B

A +AC

=
AH
cos
_

2

A
2
_ +
AK
cos
_

2

A
2
_ =
AH +HK
cos
_

2

A
2
_
www.MATHVN.com - HOANG NGOC QUANG, Yen Bai
99
=
p c +p b
cos
_

2

A
2
_ =
a
sin
A
2
=
2Rsin A
sin
A
2
= 4Rcos
A
2
.
Tng t A

= 4Rcos
C
2
, C

= 4Rcos
B
2
. Do
S
A

C
=
1
2
.B

.A

. sin B

=
1
2
.4Rcos
A
2
.4Rcos
C
2
. cos
B
2
=8R
2
cos
A
2
cos
C
2
cos
B
2
=
R
2
sin Asin Bsin C
sin
A
2
sin
B
2
sin
C
2
=
1
8R
.
2Rsin A.2Rsin B.2Rsin C
sin
A
2
sin
B
2
sin
C
2
=
abc
2r
=
abc
4R
.
2R
r
=
2S.R
r
.
Theo cng thc vng trn bng tip, ta c
r
a
+r
b
= p
_
tan
A
2
+ tan
B
2
_
= p
_
r
p a
+
r
p b
_
=
prc
(p a) (p b)
.
Mt khc, t cng thc din tch tam gic li c
pr.
abc
4R
= p (p a) (p b) (p c)
prc
(p a) (p b)
= 4R
p (p c)
ab
,
nn r
a
+r
b
=
prc
(p a) (p b)
= 4R
p (p c)
ab
= 4Rcos
2
C
2
.
Tng t r
b
+r
c
= 4Rcos
2
A
2
, r
c
+r
a
= 4Rcos
2
B
2
.
T cc kt qu trn ta c
zA

B
2
+xB

C
2
+yC

A
2
=z.16R
2
cos
2
C
2
+x.16R
2
cos
2
A
2
+y.16R
2
cos
2
B
2
=4R
_
z.4R. cos
2
C
2
_
+ 4R
_
x.4R. cos
2
A
2
_
+ 4R
_
y.4R. cos
2
B
2
_
=4R[z (r
a
+r
b
) +x (r
b
+r
c
) +y (r
c
+r
a
)] .
Suy ra 4R[z (r
a
+r
b
) +x (r
b
+r
c
) +y (r
c
+r
a
)] 4

xy +yz +zx.
2SR
r
.
Bit rng r
a
(p a) = r
b
(p b) = r
c
(p c) = pr = S, nn bt ng thc
trn tng ng vi
r [r
a
(y +z) +r
b
(z +x) +r
c
(z +x)] 2S

xy +yz +zx
www.MATHVN.com - HOANG NGOC QUANG, Yen Bai
100
4 (y +z)
S
2
p (p a)
+ 4 (z +x)
S
2
p (p b)
+ 4 (x +y)
S
2
p (p c)
8S

xy +yz +zx
4 (y +z) (p b) (p c) + 4 (z +x) (p c) (p a) + 4 (x +y) (p a) (p b)
8S

xy +yz +zx
(y +z) a
2
+ (z +x) b
2
+ (x +y) c
2
8S

xy +yz +zx + (x +y) (a b)


2
+ (y +z) (b c)
2
+ (z +x) (c a)
2
.
Nhn xt 4.1. bt ng thc (4.18) nu t y + z = x
1
, z + x =
y
1
, x +y = z
1
th bt ng thc s tr thnh
x
1
a
2
+y
1
b
2
+z
1
c
2
4
_
x
2
1
+y
2
1
+z
2
1
(x
1
y
1
)
2
(y
1
z
1
)
2
(z
1
x
1
)
2
S
+x
1
(b c)
2
+y
1
(c a)
2
+z
1
(a b)
2
. (4.19)
Nh vy bt ng thc (4.19) ng vi x
1
, y
1
, z
1
l di ba cnh ca
mt tam gic. Vn t ra l liu bt ng thc (4.19) c cn ng vi
mi x
1
, y
1
, z
1
l cc s thc dng hay khng ? Cu tr li l c, c
th hin thng qua nh l sau y:
nh l 4.6. Vi x, y, z l cc s thc dng. Chng minh rng
xa
2
+yb
2
+zc
2
4
_
x
2
+y
2
+z
2
(x y)
2
(y z)
2
(z x)
2
S
+x(b c)
2
+y(c a)
2
+z(a b)
2
. (4.20)
Chng minh. p dng bt ng thc (4.16) cho tam gic A

ta c
(x +y +z) B

C
2
+ (x y +z) C

A
2
+ (x +y z) A

B
2
4
_
x
2
+y
2
+z
2
(x y)
2
(y z)
2
(z x)
2
.S
A

C
.
Mt khc, ta li c
(x +y +z) B

C
2
+ (x y +z) C

A
2
+ (x +y z) A

B
2
=(x +y +z) 16R
2
cos
2
A
2
+ (x y +z) 16R
2
cos
2
B
2
+ (x +y z) 16R
2
cos
2
C
2
=4R[(x +y +z) (r
b
+r
c
) + (x y +z) (r
c
+r
a
) + (x +y z) (r
a
+r
b
)]
=8R(xr
a
+yr
b
+zr
c
) =
8R
r
(xrr
a
+yrr
b
+zrr
c
)
www.MATHVN.com - HOANG NGOC QUANG, Yen Bai
101
=
8R
r
[x(p b) (p c) +y (p c) (p a) +z (p a) (p b)] .
v
4
_
x
2
+y
2
+z
2
(x y)
2
+ (y z)
2
+ (z x)
2
.S
A

=4
_
x
2
+y
2
+z
2
(x y)
2
+ (y z)
2
+ (z x)
2
.
2S.R
r
.
Suy ra
xa
2
+yb
2
+zc
2
4
_
x
2
+y
2
+z
2
(x y)
2
(y z)
2
(z x)
2
S
+x(b c)
2
+y(c a)
2
+z(a b)
2
.

4.2.2. Cc h qu ca bt ng thc Weizenbock suy rng


H qu 4.3. Vi mi s thc dng x, y, z. Khi bt ng thc sau
lun ng
xab +ybc +zca 4

xy +yz +zx.S (4.21)


Chng minh. V bt ng thc Weizenbock suy rng (4.16) ng vi mi
s thc dng x, y, z nn thay x x
b
a
, y y
c
b
, z z
a
c
ta c
xab +ybc +zca 4
_
xy
c
a
+yz
a
b
+zx
b
c
.S
hay
(xab +ybc +zca)
2
16S
2
_
xy
c
a
+yz
a
b
+zx
b
c
_
. (4.22)
Li thay x z
c
a
, y x
a
b
, z y
b
c
vo bt ng thc Weizenbock suy
rng, sau bnh phng hai v , ta c bt ng thc
(xab +ybc +zca)
2
16S
2
_
xy
a
c
+yz
b
a
+zx
c
b
_
. (4.23)
www.MATHVN.com - HOANG NGOC QUANG, Yen Bai
102
Cng theo v 2 bt ng thc (4.22) v (4.23), ri p dng bt ng
thc AM GM, ta c
2 (xab +ybc +zca)
2
16S
2
_
xy
_
a
c
+
c
a
_
+yz
_
b
a
+
a
b
_
+zx
_
b
c
+
c
b
__
2.16S
2
(xy +yz +zx) .
Suy ra xab +ybc +zca 4

xy +yz +zx.S.
Trong cc bt ng thc (4.16) v (4.21) c xut hin nhng bin s
x, y, z dng ty . Ta ngh n vic thay cc s x, y, z bng cc bin
MA, MB, MC s thu c mt s kt qu sau
H qu 4.4. Cho M l mt im ty trong tam gic ABC. Khi
bt ng thc sau lun ng
aMA +bMB +cMC 4S. (4.24)
Chng minh. p dng bt ng thc Weizenbock suy rng vi x =
MA
a
, y =
MB
b
, z =
MC
c
, ta c aMA + bMB + cMC =
MA
a
a
2
+
MB
b
b
2
+
MC
c
c
2
4
_
MA
a
MB
b
+
MB
b
MC
c
+
MC
c
MA
a
.S. Mt khc, ta c
MA
a
MB
b
+
MB
b
MC
c
+
MC
c
MA
a
1 nn aMA +bMB +cMC 4S.
H qu 4.5. Cho M l mt im ty trong tam gic ABC. Khi
bt ng thc sau lun ng
bc
a
MA +
ca
b
MB +
ab
c
MC 4S. (4.25)
Chng minh. p dng bt ng thc (4.21) h qu 4.3 vi x =
MC
c
, y =
MA
a
, z =
MB
b
, ta c
bc
a
MA +
ca
b
MB +
ab
c
MC
4
_
MA
a
MB
b
+
MB
b
MC
c
+
MC
c
MA
a
.S 4S.
H qu 4.6. Cho tam gic ABC vi di cc cnh a, b, c din tch S
v tam gic A

vi di cc cnh a

, b

, c

din tch S

; M l mt
im ty trong mt phng. Khi bt ng thc sau lun ng
a
2
a
MA +
b
2
b
MB +
c
2
c
MC 4S

. (4.26)
www.MATHVN.com - HOANG NGOC QUANG, Yen Bai
103
Chng minh. p dng bt ng thc Weizenbock suy rng cho tam gic
A

vi x =
MA
a
, y =
MB
b
, z =
MC
c
, ta c
a
2
a
MA+
b
2
b
MB +
c
2
c
MC 4
_
MA
a
MB
b
+
MB
b
MC
c
+
MC
c
MA
a
.S

4S

.
Cc bt ng thc trn l kh kh v p. Th nhng ta rng cc
h s tng qut x, y, z khng xut hin. Cc h s c th bi cc
bin MA, MB, MC. vn xut hin cc h s x, y, z, ta s dng bt
ng thc Hyashi
MA.MB
ab
+
MB.MC
bc
+
MC.MA
ca
1 thu c kt qu sau
H qu 4.7. Cho tam gic ABC vi di cc cnh a, b, c v tam gic
A

vi di cc cnh a

, b

, c

din tch S

. Vi M l mt im ty
trong mt phng v x, y, z l cc s thc dng, ta c
x
a
2
a
2
MA
2
+y
b
2
b
2
MB
2
+z
c
2
c
2
MC
2
4
_
xyz
x +y +z
.S

.
Chng minh. p dng bt ng thc Weizenbock suy rng cho tam gic
A

, ta c
x
a
2
a
2
MA
2
+y
b
2
b
2
MB
2
+z
c
2
c
2
MC
2
=
x
a
2
MA
2
.a
2
+
y
b
2
MB
2
.b
2
+
z
c
2
MC
2
.c
2
4
_
yz
MB
2
b
2
.
MC
2
c
2
+zx
MC
2
c
2
.
MA
2
a
2
+xy
MA
2
a
2
.
MB
2
b
2
.S

. (4.27)
Mt khc p dng bt ng thc Cauchy - Schwarz, ta c
_
yz
MB
2
b
2
.
MC
2
c
2
+zx
MC
2
c
2
.
MA
2
a
2
+xy
MA
2
a
2
.
MB
2
b
2
__
1
yz
+
1
zx
+
1
xy
_

_
MB.MC
bc
+
MC.MA
ca
+
MA.MB
ab
_
2
1.
Suy ra
yz
MB
2
b
2
.
MC
2
c
2
+zx
MC
2
c
2
.
MA
2
a
2
+xy
MA
2
a
2
.
MB
2
b
2

xyz
x +y +z
. (4.28)
T (4.27) v (4.28) ta c bt ng thc cn chng minh.
www.MATHVN.com - HOANG NGOC QUANG, Yen Bai
104
H qu 4.8. Cho tam gic ABC, M l mt im ty trong mt
phng. Khi bt ng thc sau lun ng
(MA.MB +MB.MC +MC.MA)
_
a
MA
+
b
MB
+
c
MC
_
12S.
Chng minh. p dng bt ng thc (4.21), ta c
MA.MB +MB.MC +MC.MA
=
MA
a
.
MB
b
.ab +
MB
b
.
MC
c
.bc +
MC
c
.
MA
a
.ca
4

MA.MB.MC
abc
_
MA
a
+
MB
b
+
MC
c
_
.S.
chng minh bt ng thc cho ta cn chng minh
MA.MB.MC
abc
_
MA
a
+
MB
b
+
MC
c
__
a
MA
+
b
MB
+
c
MC
_
2
9.
Tht vy, ta c
_
MA
a
+
MB
b
+
MC
c
__
a
MA
+
b
MB
+
c
MC
_
9 (4.29)
v
MA.MB.MC
abc
_
a
MA
+
b
MB
+
c
MC
_
=
MB.MC
bc
+
MC.MA
ca
+
MA.MB
ab
1. (4.30)
T (4.29) v (4.30) ta c iu phi chng minh.
Hnh 4.3
Ta th m rng theo hng khc
xut hin cc h s x, y, z. Vi ch
rng: Nu gi A

, B

, C

l hnh chiu
ca M trn BC, CA, AB th
MA =
2R.B

a
, MB =
2R.C

b
,
MC =
2R.A

c
.
www.MATHVN.com - HOANG NGOC QUANG, Yen Bai
105
Nh th, ta quy cc bin MA, MB, MC v di 3 cnh ca mt
tam gic. Ta c kt qu sau
H qu 4.9. Cho tam gic ABC vi di cc cnh a, b, c ni tip
ng trn (O, R). M l mt im bt k trong tam gic. Khi vi
mi s thc dng x, y, z ta c
x
MB.MC
a
+y
MC.MA
b
+z
MA.MB
c

xy +yz +zx.
_
R
2
OM
2
_
R
.
Chng minh. Gi A

, B

, C

l hnh chiu ca M trn BC, CA, AB theo


nh l Euler ta c
[A

] =
1
4
_
1
OM
2
R
2
_
. [ABC]
p dng h qu 4.3 cho tam gic A

vi lu MA =
2R.B

a
,
MB =
2R.C

b
, MC =
2R.A

c
, ta c

xy +yz +zx.
_
R
2
OM
2
_
R
= 4R.

xy +yz +zx.
[A

]
[ABC]
R.
xA

.A

+yB

.B

+zC

.C

[ABC]
=
xMC.MB.cb +yMA.MC.ac +zMB.MA.ba
4R[ABC]
=
xMC.MB.cb +yMA.MC.ac +zMB.MA.ba
abc
=x
MB.MC
a
+y
MC.MA
b
+z
MA.MB
c
.
4.3. Bt ng thc Klamkin v cc h qu
4.3.1. Bt ng thc Klamkin
Vo nm 1975, M.S. Klamkin thit lp nh l sau y:
nh l 4.7. Cho ABC l mt tam gic ty vi di cc cnh ln
lt l a, b, c v P l im bt k trong mt phng cha tam gic. Vi
cc s thc x, y, z ta c
(x +y +z)
_
xPA
2
+yPB
2
+zPC
2
_
yza
2
+zxb
2
+xyc
2
. (4.31)
www.MATHVN.com - HOANG NGOC QUANG, Yen Bai
106
Chng minh. Ta c
x

PA +y

PB +z

PC 0
_
x
2
PA
2
+y
2
PB
2
+z
2
PC
2
_
+
+
_
2xy

PA.

PB + 2yz

PB.

PC + 2zx

PC.

PA
_
0. (4.32)
Theo nh l hm s cosin ta c
2

PA.

PB = 2PA.PBcos
_

PA,

PB
_
= PA
2
+PB
2
c
2
,
2

PB.

PC = 2PB.PC cos
_

PB,

PC
_
= PB
2
+PC
2
a
2
,
2

PC.

PA = 2PC.PAcos
_

PC,

PA
_
= PC
2
+PA
2
b
2
.
thay cc bt ng thc trn vo (4.32) ta c thu c bt ng thc
(x +y +z)
_
xPA
2
+yPB
2
+zPC
2
_
yza
2
+zxb
2
+xyc
2
.
ng thc trong (4.31) xy ra khi v ch khi x

PA+y

PB+z

PC =

0 ,
tc l P l tm t c ca h im {A, B, C}.
4.3.2. Cc h qu ca bt ng thc Klamkin
H qu 4.10. Trong tam gic ABC vi G l trng tm, ta c cc bt
ng thc sau
1. PA
2
+PB
2
+PC
2

a
2
+b
2
+c
2
3
. (4.33)
2. PA
2
+PB
2
+PC
2

4
9
_
m
2
a
+m
2
b
+m
2
c
_
. (4.34)
3. PA
2
+PB
2
+PC
2
GA
2
+GB
2
+GC
2
. (4.35)
Chng minh. Khi x = y = z, bt ng thc 4.31 tr thnh
3
_
PA
2
+PB
2
+PC
2
_
a
2
+b
2
+c
2
.
Suy ra bt ng thc (4.33). Tip theo ta bin i bt ng thc (4.33)
nh sau
PA
2
+PB
2
+PC
2

2
_
b
2
+c
2
_
a
2
9
+
2
_
c
2
+a
2
_
b
2
9
+
2
_
a
2
+b
2
_
c
2
9
.
T y suy ra cc bt ng thc (4.34) v (4.35).
www.MATHVN.com - HOANG NGOC QUANG, Yen Bai
107
H qu 4.11. Trong tam gic ABC, ta c cc bt ng thc sau
1.
PA
2
a
2
+
PB
2
b
2
+
PC
2
c
2

a
4
+b
4
+c
4
a
2
b
2
+b
2
c
2
+c
2
a
2
1. (4.36)
2.
PA
2
b
2
+
PB
2
c
2
+
PC
2
a
2
1. (4.37)
3.
PA
2
c
2
+
PB
2
a
2
+
PC
2
b
2
1. (4.38)
Chng minh. p dng bt ng thc Klamkin vi x =
1
a
2
, y =
1
b
2
, z =
1
c
2
ta thu c bt ng thc (4.36). Nu cho x =
1
b
2
, y =
1
c
2
, z =
1
a
2
th ta
c bt ng thc (4.37), cn nu cho x =
1
c
2
, y =
1
a
2
, z =
1
b
2
ta s c bt
ng thc (4.38).
H qu 4.12. Trong tam gic ABC, ta c cc bt ng thc sau
1.
m
2
a
a
2
+
m
2
b
b
2
+
m
2
c
c
2

9
4
. (4.39)
2.
m
a
m
b
ab
+
m
b
m
c
bc
+
m
c
m
a
ca

9
4
. (4.40)
Chng minh. p dng bt ng thc Klamkin vi P G, x =
1
a
2
,
y =
1
b
2
, z =
1
c
2
ta thu c bt ng thc (4.39). Nu cho P G,
x =
a
m
a
, y =
b
m
b
, z =
c
m
c
, ta s c bt ng thc (4.40).
H qu 4.13. Trong tam gic ABC, ta c cc bt ng thc sau
1.
cos
2
A
sin Bsin C
+
cos
2
B
sin C sin A
+
cos
2
C
sin Asin B
1. (4.41)
2. 4R
2

a
3
+b
3
+c
3
+abc
a +b +c
. (4.42)
3.
2R r
r

a
3
+b
3
+c
3
abc
. (4.43)
Chng minh. p dng bt ng thc Klamkin vi x = a, y = b, z = c,
P H (H l trc tm ca tam gic). rng HA = |2Rcos A| ,
www.MATHVN.com - HOANG NGOC QUANG, Yen Bai
108
HB = |2Rcos B| , HC = |2Rcos C| v s dng nh l hm s sin ta
d dng nhn c bt ng thc (4.41). S dng cc bin i HA
2
=
4R
2
cos
2
A = 4R
2
_
1 sin
2
A
_
= 4R
2
a
2
, HB
2
= 4R
2
b
2
, HC
2
=
4R
2
c
2
, sau mt s bin i ta nhn c bt ng thc (4.42). S
dng cng thc abc = 4Rrp ta bin i bt ng thc (4.42) v bt ng
thc (4.43).
4.4. Bt ng thc Jian Liu v cc h qu
4.4.1. Bt ng thc Jian Liu
Jian Liu chng minh nh l 4.8 di y. chng minh nh l
4.8 ta cn b sau:
B 4.1. Cho ABC l mt tam gic ty v P l mt im bt k
trong mt phng cha tam gic ABC. Nu bt ng thc sau y
f (a, b, c, R
1
, R
2
, R
3
) 0 (4.44)
ng, th bt ng thc sau cng ng
f (aR
1
, bR
2
, cR
3
, R
2
R
3
, R
3
R
1
, R
1
R
2
) 0, (4.45)
trong R
1
, R
2
, R
3
ln lt l khong cch t P n cc nh A, B, C.
Chng minh. Xt php nghch o N tm P, h s R
1
.R
2
.R
3
, ta c
N : A A

, B B

, C C

Khi ta c PA

= PB.PC = R
2
R
3
, tng t PB

= R
3
R
1
,
PC

= R
1
R
2
v B

= R
1
R
2
R
3
.
BC
PB.PC
= aR
1
, tng t C

=
bR
2
, A

= cR
3
. V f (a, b, c, R
1
, R
2
, R
3
) 0 ng vi mi tam
gic ABC v mi P nn p dng iu kin ny cho tam gic
A

v im P ta c f (B

, C

, A

, PA

, PB

, PC

) 0 hay
f (aR
1
, bR
2
, cR
3
, R
2
R
3
, R
3
R
1
, R
1
R
2
) 0.
nh l 4.8. Cho x, y, z l cc s dng. Khi , vi tam gic ABC
ty v P l im bt k trong mt phng cha ABC, bt ng thc
www.MATHVN.com - HOANG NGOC QUANG, Yen Bai
109
sau ng
R
2
1
x
+
R
2
2
y
+
R
2
3
z

aR
1
+bR
2
+cR
3

xy +yz +zx
, (4.46)
trong R
1
= PA, R
2
= PB, R
3
= PC.
ng thc xy ra khi v ch khi ABC l tam gic nhn, P trng vi
trc tm H v x : y : z = cot A : cot B : cot C.
Chng minh. Nu P trng vi mt trong cc nh ca ABC, chng
hn, P A, th PA = 0, PB = c, PC = b v bt ng thc (4.46) tr
thnh tm thng. Trong trng hp ny, du ng thc trong (4.46)
r rng l khng xy ra.
Tip theo, gi s P khng trng vi mt trong cc nh ca ABC.
Nu x, y, z l cc s dng th bt ng thc (4.31) tng ng vi bt
ng thc sau
_
xR
2
1
+yR
2
2
+zR
2
3
_
_
1
yz
+
1
zx
+
1
xy
_

a
2
x
+
b
2
y
+
c
2
z
.
Mt khc, theo bt ng thc Cauchy - Schwarz ta c
a
2
x
+
b
2
y
+
c
2
z

(a +b +c)
2
x +y +z
.
ng thc xy ra khi v ch khi x : y : z = a : b : c.
Kt hp hai bt ng thc trn ta c bt ng thc
_
xR
2
1
+yR
2
2
+zR
2
3
_
_
1
yz
+
1
zx
+
1
xy
_

(a +b +c)
2
x +y +z
. (4.47)
Du ng thc trong (4.47) xy ra khi v ch khi x : y : z = a : b : c v
P l tm ca ng trn ni tip tam gic ABC.
By gi, p dng php nghch o trong b 4.1 cho bt ng thc
(4.47), ta thu c
_
x(R
2
R
3
)
2
+y(R
3
R
1
)
2
+z(R
1
R
2
)
2

_
1
yz
+
1
zx
+
1
xy
_

(aR
1
+bR
2
+cR
3
)
2
x +y +z
,
hay
(R
2
R
3
)
2
yz
+
(R
3
R
1
)
2
zx
+
(R
1
R
2
)
2
xy

_
aR
1
+bR
2
+cR
3
x +y +z
_
2
. (4.48)
www.MATHVN.com - HOANG NGOC QUANG, Yen Bai
110
Thay x xR
2
1
, y yR
2
2
, z zR
2
3
, ta c
1
yz
+
1
zx
+
1
xy

_
aR
1
+bR
2
+cR
3
xR
2
1
+yR
2
2
+zR
2
3
_
2
. (4.49)
Li mt ln na thay x
1
x
, y
1
y
, z
1
z
, ta c bt ng thc
(4.46) cn chng minh.
Nu ng thc trong (4.44) ch xy ra khi P l tm ca ng trn ni
tip ABC, th ng thc trong (4.45) xy ra ch khi ABC l nhn
v P l trc tm ca n. Theo iu ny v iu kin ng thc trong
(4.47) xy ra, ta c ng thc trong (4.46) xy ra khi v ch khi ABC
l nhn, P l trc tm ca n v
R
1
xa
=
R
2
yb
=
R
3
zc
. (4.50)
Khi P l trc tm ca tam gic nhn ABC, ta c R
1
: R
2
: R
3
= cos A :
cos B : cos C. Do , trong trng hp ny, t (4.50) ta c x : y : z =
cot A : cot B : cot C. V vy, ng thc trong (4.46) xy ra khi v ch khi
ABC l nhn, P trng vi trc tm ca n v
x
cot A
=
y
cot B
=
z
cot C
.
4.4.2. Cc h qu ca bt ng thc Jian Liu
H qu 4.14. Vi P ty nm trong mt phng cha tam gic ABC
v vi mi s dng x, y, z, bt ng thc sau lun ng
R
2
1
x
+
R
2
2
y
+
R
2
3
z

4S

xy +yz +zx
. (4.51)
ng thc xy ra khi v ch khi x : y : z = cot A : cot B : cot C v P l
trc tm ca tam gic nhn ABC.
Chng minh. T bt ng thc (4.24) v bt ng thc Jian Liu (4.46)
ta thu c bt ng thc cn chng minh (4.51).
H qu 4.15. Cho tam gic ABC v P l mt im ty trong mt
phng cha tam gic. Khi bt ng thc sau ng
R
2
1
+R
2
2
+R
2
3

3

3
S. (4.52)
www.MATHVN.com - HOANG NGOC QUANG, Yen Bai
111
Chng minh. Trong bt ng thc (4.51) cho x = y = z s thu c bt
ng thc (4.52).
H qu 4.16. Vi P ty nm trong mt phng cha tam gic ABC
v vi mi s dng x, y, z, bt ng thc sau lun ng
aR
2
1
+bR
2
2
+cR
2
3
aR
1
+bR
2
+cR
3

2Rr. (4.53)
ng thc xy ra khi v ch khi tam gic ABC u v P l trc tm
ca n.
Chng minh. Trong bt ng thc Jian Liu (4.46) ta thay x =
1
a
, y =
1
b
,
z =
1
c
v s dng kt qu
1
bc
+
1
ca
+
1
ab
=
1
2Rr
, ta thu c bt ng
thc cn chng minh (4.53).
H qu 4.17. (Bt ng thc Hayashi) Nu P l mt im ty v
khng trng vi cc nh ca tam gic ABC, th
R
2
R
3
bc
+
R
3
R
1
ca
+
R
1
R
2
ab
1. (4.54)
ng thc xy ra khi v ch khi tam gic ABC nhn v P l trc tm
ca n.
Chng minh. Trong bt ng thc (4.48), thay x = aR
1
, y = bR
2
,
z = cR
3
ta c bt ng thc (4.54).
H qu 4.18. Nu P l mt im ty v khng trng vi cc nh
ca tam gic ABC, th
(R
2
R
3
+R
3
R
1
+R
1
R
2
)
2
_
1
R
2
R
3
+
1
R
3
R
1
+
1
R
1
R
2
_
4p
2
. (4.55)
Trong p l na chu vi ca tam gic ABC. ng thc xy ra khi v
ch khi tam gic ABC nhn v P l trc tm ca n.
Chng minh. Trong (4.48) thay x xaR
1
, y ybR
2
, z zcR
3
. Khi
ta c
R
2
R
3
yzbc
+
R
3
R
1
zxca
+
R
1
R
2
xyab

_
aR
1
+bR
2
+cR
3
xaR
1
+ybR
2
+zcR
3
_
2
. (4.56)
www.MATHVN.com - HOANG NGOC QUANG, Yen Bai
112
Tip tc thay x =
1
a
, y =
1
b
, z =
1
c
, ta c
(R
2
R
3
+R
3
R
1
+R
1
R
2
) (R
1
+R
2
+R
3
)
2
(aR
1
+bR
2
+cR
3
)
2
. (4.57)
p dng phng php bin i nghch o ca b 4.1 cho bt ng
thc (4.57), sau chia c hai v cho (R
1
R
2
R
3
)
2
ta c bt ng thc
(4.55).
H qu 4.19. Cho P l mt im nm trong tam gic ABC, khi bt
ng thc sau lun ng
(R
2
R
3
)
2
r
2
r
3
+
(R
3
R
1
)
2
r
3
r
1
+
(R
1
R
2
)
2
r
1
r
2

16
9
p
2
. (4.58)
Trong p l na chu vi ca tam gic ABC.
Chng minh. Vi P l mt im ty nm trong tam gic ABC, ta c
cc bt ng thc sau
aR
1
br
3
+cr
2
, bR
2
cr
1
+ar
3
, cR
3
ar
2
+br
1
.
Cng theo v 3 bt ng thc trn vi lu l a+b +c = 2p, ar
1
+br
2
+
cr
3
= 2rp, ta thu c bt ng thc
aR
1
+bR
2
+cR
3
2p (r
1
+r
2
+r
3
) 2rp.
Nhn c hai v vi bt ng thc trn vi 2, sau cng vi bt ng
thc aR
1
+bR
2
+cR
3
4S, vi lu l S = rp ta thu c
3 (aR
1
+bR
2
+cR
3
) 4p (r
1
+r
2
+r
3
) , (4.59)
hay
aR
1
+bR
2
+cR
3
r
1
+r
2
+r
3

4
3
p. (4.60)
Mt khc, trong bt ng thc (4.48) cho x = r
1
, y = r
2
, z = r
3
ta thu
c bt ng thc
(R
1
R
2
)
2
r
2
r
3
+
(R
3
R
1
)
2
r
3
r
1
+
(R
1
R
2
)
2
r
1
r
2

_
aR
1
+bR
2
+cR
3
r
1
+r
2
+r
3
_
2
. (4.61)
T hai bt ng thc (4.60) v (4.61) ta c bt ng thc (4.58).
www.MATHVN.com - HOANG NGOC QUANG, Yen Bai
113
H qu 4.20. Cho P l mt im nm trong tam gic ABC, khi bt
ng thc sau lun ng
(R
1
+R
2
+R
3
)
2
r
1
+r
2
+r
3

3
p. (4.62)
Chng minh. T bt ng thc (4.57) v (4.60) ta suy ra
(R
2
R
3
+R
3
R
1
+R
1
R
2
) (R
1
+R
2
+R
3
)
2

16
9
p
2
(r
1
+r
2
+r
3
)
2
.
Mt khc, ta c
3 (R
2
R
3
+R
3
R
1
+R
1
R
2
) (R
1
+R
2
+R
3
)
2
.
T hai bt ng thc trn ta c bt ng thc (4.62).
H qu 4.21. Cho P l mt im nm trong tam gic ABC, khi bt
ng thc sau lun ng
R
2
1
r
a
+
R
2
2
r
b
+
R
2
3
r
c

4
3
(r
1
+r
2
+r
3
) . (4.63)
Chng minh. Trong bt ng thc (4.46) thay x r
a
, y r
b
, z r
c
vi lu l r
b
r
c
+r
c
r
a
+r
a
r
b
= p
2
, ta c
R
2
1
r
a
+
R
2
2
r
b
+
R
2
3
r
c

1
p
(aR
1
+bR
2
+cR
3
) . (4.64)
T bt ng thc (4.60) v (4.64) ta c bt ng thc (4.63).
H qu 4.22. Cho P l mt im nm trong tam gic ABC, khi bt
ng thc sau lun ng
R
2
1
r
a
+
R
2
2
r
b
+
R
2
3
r
c
r
1
+r
2
+r
3
+r. (4.65)
Chng minh. Cng theo v cc bt ng thc (4.59) v aR
1
+bR
2
+cR
3

4S, sau chia c hai v cho 4, ta c
aR
1
+bR
2
+cR
3
p (r
1
+r
2
+r
3
+r) . (4.66)
T bt ng thc (4.64) v (4.66) ta c bt ng thc (4.65).
www.MATHVN.com - HOANG NGOC QUANG, Yen Bai
114
H qu 4.23. Cho P l mt im nm trong tam gic ABC. Gi
h
a
, h
b
, h
c
ln lt l di ng cao h t A, B, C. Khi
R
2
1
h
a
+
R
2
2
h
b
+
R
2
3
h
c
4r. (4.67)
Trong r l bn knh ng trn ni tip tam gic ABC.
Chng minh. Ta bit rng
1
h
a
+
1
h
b
+
1
h
c
=
1
r
hay h
a
h
b
+ h
b
h
c
+ h
c
h
a
=
h
a
h
b
h
c
r
=
8S
3
abcr
=
2S
2
rR
. V R 2r nn
h
a
h
b
+h
b
h
c
+h
c
h
a

S
2
r
2
. (4.68)
Trong (4.51) cho x = h
a
, y = h
b
, z = h
c
v s dng (4.68) ta c
R
2
1
h
a
+
R
2
2
h
b
+
R
2
3
h
c

4S
_
S
2
r
2
= 4r.
H qu 4.24. Cho P l mt im nm trong tam gic ABC. Khi
bt ng thc sau lun ng
aR
2
1
+bR
2
2
+cR
2
3

2

S
3
4

27
(4.69)
Chng minh. Theo bi ton 1.6 ta c 4

3
9abc
a +b +c
hay
1
ab
+
1
bc
+
1
ca

3

3
4S
. (4.70)
By gi trong (4.51) cho x =
1
a
, y =
1
b
, z =
1
c
v s dng (4.4.2.) ta c
aR
2
1
+bR
2
2
+cR
2
3

4S
_
3

3
4S
=
2

S
3
4

27
.
H qu 4.25. Cho P l mt im nm trong tam gic ABC. Gi
m
a
, m
b
, m
c
ln lt l di trung tuyn xut pht t A, B, C. Khi
bt ng thc sau lun ng
m
a
R
2
1
+m
b
R
2
2
+m
c
R
2
3

4S

S
4

3
. (4.71)
www.MATHVN.com - HOANG NGOC QUANG, Yen Bai
115
Chng minh. Gi A
1
, B
1
, C
1
ln lt l trung im ca BC, CA, AB.
Dng tam gic AA
1
M sao cho M i xng vi C
1
qua B
1
. D thy
m
a
, m
b
, m
c
l di cc cnh ca tam gic MAA
1
(m
a
= AA
1
, m
b
=
MA
1
, m
c
= MA) v [MAA
1
] =
3
4
S. p dng bt ng thc ta c
1
m
a
m
b
+
1
m
b
m
c
+
1
m
c
m
a

4S

S
4

3
=

3
S
. (4.72)
By gi trong (4.51) cho x =
1
m
a
, y =
1
m
b
, z =
1
m
c
v s dng bt ng
thc (4.72) ta c m
a
R
2
1
+m
b
R
2
2
+m
c
R
2
3

4S
_

3
S
=
4S

S
4

3
.
www.MATHVN.com - HOANG NGOC QUANG, Yen Bai
116
Kt lun
Lun vn trnh by v t c mt s kt qu sau:
1. Trnh by mt s bt ng thc trong tam gic v trong t gic t
c bn n nng cao.
2. Gii thiu hai bt ng thc quen thuc v c nhiu p dng trong
cc thi hc sinh gii l bt ng thc Erdos-modell v bt ng
thc Ptolemy. ng thi cng xy dng mt s m rng ca hai bt
ng thc ny.
3. Trnh by mt s bt ng thc c trng, cc m rng v p dng,
ng thi c bit ha c cc bi ton bt ng thc hnh hc mi
lin quan n khong cch t mt im n cc nh v cc cnh ca
tam gic.
4. Tc gi nu v chng minh c mt s b gip cho vic chng
minh cc bt ng quan trng trong lun vn nh bt ng thc hnh
bnh hnh, bt ng thc Erdos-Modell m rng i vi 2 im trong
tam gic, m rng trong t gic v a gic, bt ng thc Jian Liu v.v..
Ngoi ra, khai thc mt s bt ng thc tc gi c bit ha thu
c nhiu bt ng thc hay v p.
5. Lun vn chn lc gii thiu mt s thi hc sinh gii trong
nc, khu vc v quc t lin quan n cc bt ng thc trong tam
gic v trong t gic. Cc bt ng thc hnh hc p dng trc tip hai
bt ng thc ni ting l bt ng thc Erdos-Modell v bt ng
thc Ptolemy.
www.MATHVN.com - HOANG NGOC QUANG, Yen Bai
117
Ti liu tham kho
[1] B gio dc v o to-Hi ton hc Vit Nam (2009), Cc bi ton
chn lc - 45 nm Tp ch Ton hc v Tui tr, NXB Gio Dc.
[2] Nguyn Minh H, Suy ngh mi t mt bi ton quen thuc, Tuyn
tp 5 nm tp ch ton hc v tui tr, NXB Gio Dc (1995).
[3] V nh Ha (2005), Bt ng thc hnh hc, NXB Gio Dc.
[4] Phan Huy Khi, Nguyn o Phng (1994), H thc lng trong
tam gic v t gic, NXB Gio Dc.
[5] Nguyn Vn Nho (2011), Nhng nh l chn lc trong hnh hc
phng qua cc k thi Olympic, NXB i Hc S Phm.
[6] Nguyn c Tn (2000), Chuyn bt ng thc v cc tr trong
hnh hc phng, NXB Gio dc.
[7] Nguyn Thng V (1989), 200 bi ton chn lc v h thc lng
trong tam gic, NXB Gio dc.
[8] Trn Nam Dng, Ptolemys inequality and its applications, K yu
hi ngh khoa hc, Vit Tr (2011).
[9] Titu Andreescu and Dorin Andrica, Proving some geometric in-
equalities by using complex numbers, Educatia Matematica Vol.1,
N2, (2005), 19-26.
[10] Dusan Djukic, Vladimir Jankovic, Ivan Matic, Nikola Petrovic, The
IMO Compendium A Collection of Problems Suggested for the Inter-
national Mathematical Olympiads:19592004, Springer Publishers,
2004.
[11] Yu-Dong Wu, Chun-Lei Yu Zhi-Hua Zhang, A geometric inequality
of the generalized Erdos-Modell type, Journal of inequalities in pure
and applied mathematics, Vol.10, Iss.4, Ar.106, 2009.
[12] Radmila Bulajich Manfrino, Jos Antonio Gmez Ortega, Rogelio
Valdez Delgado, Inequalities A Mathematical Olympiad Approach,
Birkhauser Publishers, 2009.
www.MATHVN.com - HOANG NGOC QUANG, Yen Bai
118
[13] D.S Mitrinovic, J.E.Pecaric and V.Volenec, Recent Advances in
Geometric Inequalities, Kluwer Academic publishers, Dordrecht,
Netherlands, 1989.
[14] Jian Liu, A weighted geometric inequality and its applications, Jour-
nal of inequalities in pure and applied mathematics, Vol.9, Iss.2,
Ar.58, 2008.
[15] Jzsef Sndor, On the geometry of equilateral triangles, Forum Ge-
ometricorum, Vol.5 (2005) 107117.
www.MATHVN.com - HOANG NGOC QUANG, Yen Bai

You might also like